Female Reproductive System/Obstetric Gynaecology Flashcards
x
On examination of a 28-year-old nulliparous woman at 37 weeks gestation, her fundal height is found to be 35cm and her fetus in transverse lie. An ultrasound exam at 18th week, revealed a low-lying placenta. Which one of the following is the most likely cause of this presentation?
A. Placenta previa.
B. Ruptured membrane.
C. Small for gestational age.
D. Nulliparity.
E. A normal variation.
A. Placenta previa
Shoulder presentations, unstable lie, transverse lie and oblique lie may be detected in late pregnancy. These conditions occur in 1 in 200 pregnancies, usually in multiparous women. The etiology is varied. They may occur in a lax multiparous uterus with no other complications of pregnancy but may be associated with other contributing factors.
Contributing factors to fetal transverse lie include:
-High parity
-Pendulous abdomen
-Placenta previa
-Polyhydramnios
-Pelvic inlet contracture and / or fetal macrosomia
-Uterine abnormalities (e.g. bicornuate uterus or uterine fibroids)
-Fetal anomaly (e.g. tumors of the neck or sacrum, hydrocephaly, abdominal distension)
-Distended maternal urinary bladder
-Poorly formed lower segment
-Wrong dates i.e. more premature than appears
-Undiagnosed twins
-Preterm delivery (fetus)
When transverse lie is found, an ultrasound exam is required to exclude placenta previa as a likely cause and a contraindication to vaginal delivery.
85% of low-lying placentas found on ultrasound at 18 weeks will be normally situated at term. In 15%, placenta remains in the lower segment of the uterus. This woman still has a 15% chance of having low placentation as the main contributing factor to the transverse lie.
After 20weeks gestation, the fundal height in centimeters correlates to the gestational age in weeks. A discrepancy of >2 cm between the gestational age and the fundal height (more than 2 cm) is considered significant and can be caused by:
- Fundal height < gestational age:
-Dating errors
-Oligohydramnios
-Transverse/oblique lie Small for gestational age - Fundal height > gestational age:
-Dating errors
-Large for gestational age
-Multiple pregnancy
-Polyhydramnios
-Molar pregnancy
In this case, a discrepancy of only 2 cm is not that significant but can be caused by the transverse lie.
Over all, some reports show that extreme prematurity is the most common finding in pregnancies complicated by transverse lie, but in this case and with low-lying placenta in history, placenta previa remains the most likely cause to consider and exclude.
Transverse lie near term is not a normal finding. In early pregnancy, it is very common to see in fetus in transverse lie, but not a normal finding near term.
- South Australian Perinatal Practice Guideline - Unstable lie of the fetus
- Williams Obstetrics – McGraw Hill - 24th Edition – page 468
You are working at a busy GP clinic. Your next patient is 24 -year-old woman with irregular menstrual and a positive pregnancy test. She is keen to know the age of her baby. Which one of the following methods would be the most accurate one for estimation of gestational age?
A. Bimanual examinations at 10 weeks.
B. Ultrasound at 16 weeks.
C. Transvaginal ultrasound at 8 weeks.
D. Transvaginal ultrasound at 20 weeks.
E. Transabdominal ultrasound at 20 weeks.
C. Transvaginal ultrasound at 8 weeks
By eight weeks gestation, the fetus and its heart beat can be detected relatively easy with transvaginal ultrasound. Dating scans are usually recommended if there is doubt about the validity of the last menstrual period, such as in the following conditions:
-Patient does not know when the first day of her last period or the likely day of conception was
-Patient with irregular periods
-Patients who has become pregnant while on hormonal contraceptives
Transvaginal ultrasonography performed between weeks 8 and 12 (within the first trimester) can predict the gestational with three to five days difference and is the most accurate method of determining the gestational age.
As the pregnancy advances, sonography becomes less accurate in estimation of gestational age.
- http://sogc.org/wp-content/uploads/2014/02/gui303C
A 37-year-old woman presents to you with history of irregular periods and decreased libido for the past 6 months. Which one of the following would be the investigation of choice?
A. Serum prolactin level.
B. Ultrasound scan of the ovaries.
C. Serum FSH and LH.
D. Serum FSH and estradiol.
E. Thyroid function tests.
D. Serum FSH and estradiol
In women younger than 40 years, who present with secondary amenorrhea or menstruation irregularities and signs of estrogen deficiency such as decreased libido, atrophic vaginitis, etc, the most common cause is found to be primary ovarian failure (POF) (premature menopause) or premature ovarian insufficiency (POI), for which an elevated FSH associated with decreased estradiol level is diagnostic.
Measuring serum FSH level is the core study to establish the diagnosis of POI/POF after pregnancy has been excluded. By convention, 2 FSH levels in the menopausal range for the specific assay (>40 μIU/mL by radioimmunoassay), measured at least 1 month apart, are diagnostic of POI/POF. A parallel test of serum estradiol is necessary. As a rule, serum estradiol is low in women with POI/POF and is similar to or less than the early follicular phase estradiol of women who cycle normally. The combination of low estradiol and high gonadotropins defines POI/POF.
Measurement of serum LH is also important. In most cases of spontaneous POI/POF, FSH is higher than LH. If autoimmune oophoritis is present, FSH may be only mildly elevated, sometimes below the cutoff of 40 μIU/mL, while LH is markedly elevated.
Occasionally, women with POI/POF may have spontaneous follicular activity. In such women, if hormonal tests are performed during such episodes, FSH, LH, and estradiol levels could be in the normal range or only minimally elevated. This may lead to an erroneous exclusion of POI/POF. In these cases, persistent amenorrhea or oligomenorrhea accompanied by menopausal symptoms necessitates a repeat of the above tests in 1-2 months.
*http://emedicine.medscape.com/article/271046-worku
Which of the following is not a risk factor for isolated spontaneous abortion?
A. Age more than 35 years.
B. Cigarette smoking.
C. High doses of caffeine.
D. Uterine adhesions.
E. Retroverted uterus.
E. Retroverted uterus
The following are known risk factors for spontaneous abortion or miscarriage:
-Advanced maternal age (≥35)
-Previous miscarriage(s)
-Antiphospholipid syndrome
-Parentral chromosomal derangements
-Embryonic chromosomal abnormalities
-Congenital uterine malformations
-Cervical weakness
-Diabetes mellitus (subclinical disease excluded) and thyroid disease (subclinical disease excluded)
-Immune factors
-Infections
-Inherited thrombophilic defects
-Caffeine, smoking and alcohol use (dose dependent)
A retroverted uterus has not shown association with increased risk of spontaneous abortions. A retroverted uterus is a uterus that is tilted backwards instead of forwards.
- The royal college of obstetricians and Gynaecologists – Green-top Guideline No.17
A 24-year-old woman underwent dilation and curettage for septic abortion. Now, she has developed amenorrhea of six months duration. An office pregnancy test excludes pregnancy. She smokes 10 cigarettes and drinks two standard units of alcohol every day. Which one of the following is the most appropriate next investigation to reach a cause for this presentation?
A. Liver function tests.
B. Urine analysis and microscopy.
C. Transvaginal ultrasound.
D. Full hormone assay.
E. MRI of the brain.
C. Transvaginal ultrasound
Intrauterine adhesions are a common complication of curettage. Approximately 90% cases of severe intrauterine adhesions are related to curettage for pregnancy complications such as missed or incomplete abortion, postpartum hemorrhage, or retained placental tissue.
Intrauterine adhesions can be asymptomatic and of no clinical significance.
If there are symptoms they can include:
-Infertility
-Menstrual irregularities (amenorrhea)
-Cyclic pelvic pain
-Recurrent miscarriages
When intrauterine adhesions are suspected, transvaginal ultrasonography is the next best investigation to confirm the diagnosis. Although not always necessary, diagnostic hysteroscopy remains the gold standard diagnostic investigation.
- UpToDate - Intrauterine adhesions
A 20-year-old female presents to your practice at 18 weeks pregnancy with right iliac fossa pain that is particularly brought on by getting up from a chair, sneezing and coughing. On examination she has normal vital signs and is otherwise healthy. Abdominal examination reveals no tenderness, rebound tenderness or guarding. Which one of the following is the most likely diagnosis?
A. Round ligament pain.
B. Acute appendicitis.
C. Ovarian torsion.
D. Ruptured ectopic pregnancy.
E. Uncomplicated ectopic pregnancy.
A. Round ligament pain
This patient has signs and symptom suggestive of round ligament pain. It is considered a normal finding during pregnancy and does not require any intervention.
Round ligament pain most frequently occurs during the second trimester of pregnancy when women report sharp unilateral or bilateral pain in the iliac fossa that may radiate to the groin. The pain is often sudden-onset, sharp and spastic ,lasting few seconds. The pain is aggravated by standing, getting off chairs, sneezing, laughing or rolling in bed. Sudden change of position is a well-known trigger.
Rest and avoiding sudden changes in body position is the cornerstone of management.
OPTION B : Although appendicitis presents with pain in the right iliac fossa, absence of tenderness makes this diagnosis less likely.
OPTION C : Ovarian torsion presents with aute onset severe pain followed by signs and symptoms of peritoneal irritation such as tenderness, rebound tenderness, guarding, etc.
OPTION D and E : Presentation of ectopic pregnancy and its complications (e.g. rupture) almost always occurs in the first trimester; furthermore, absence of tenderness, rebound tenderness and other localized findings make this diagnosis even less
likely.
NOTE - It should be noted that the question asks about the most likely diagnosis, not the next best step in management. Although, round ligament strain comes top on the list, but more serious conditions should be excluded first through appropriate examination and investigations.
- Danforth’s Obstetrics and Gynaecology – 10th Edition – page 18
A 30-year-old woman presents with a 12-month history of secondary infertility. Her first child, fathered by the same partner, was born 4 years ago after she was conceived spontaneously, and through an uneventful vaginal delivery. She has always had irregular periods occurring every 2 to 4 months. On examination, she is obese (BMI>32) and otherwise normal. Ultrasonography of the pelvis reveals 12 small cyst of 3-6 mm in diameter in the left and 20 cysts of about the same size in the right ovary. A sperm analysis of the partner is normal. Which one of the following is the most appropriate next step in management?
A. Metformin.
B. Clomiphene citrate.
C. Ovulation induction with gonadotropins.
D. Laparoscopic ovarian drilling.
E. In-vitro fertilization.
B. Clomiphene citrate
The findings on ultrasonography are suggestive of polycystic ovarian syndrome (PCOS). PCOS is the most common cause of infertility due to anovulation. Infertility in women with PCOS, however, is not absolute and many women can conceive even without treatment. In women with infertility due to PCOS, different options are available:
Non-pharmacological treatment:
If a woman is younger than 35 years of age and has a BMI>25, and no other cause of infertility is suspected an intensive lifestyle program addressing weight loss, without any pharmacological treatment for the first 6 months, is recommended. Small amounts of weight loss (~5%) may restore menstrual cycle regularity and ovulation, providing benefit even if pharmacological intervention is subsequently required
`Pharmacological treatment:
1-If pharmacological treatment is required, the best first-line treatment is clomiphene citrate, which has a pregnancy rate of 30–50% after six ovulatory cycles.
2-In women with a BMI <30–32 kg/m2, metformin may have a similar efficacy to clomiphene citrate, and is the first-line treatment (with or without clomiphene citrate) if there is concomitant impaired glucose tolerance).`
3-If clomiphene citrate, metformin or a combination of the two is unsuccessful in achieving pregnancy, gonadotropins are the next pharmacological options.
4-Laparoscopy with ovarian surgery/drilling (LOS) is an appropriate second-line treatment if clomiphene citrate with metformin has failed. The pregnancy rate with LOS is as effective as 3-6 cycles of gonadotropin ovulation induction.
5-If all of the above are unsuccessful or if there are other factors contributing to infertility such as endometriosis or male factors, in vitro fertilization or intra-cytoplasmic sperm injection is recommended.
*http://www.racgp.org.au/afp/2012/october/polycysti
* Therapeutic Guidelines – Endocrinology; available from http://tg.org.au
A 30-year-old pregnant woman presents to the Emergency Department with severe right-sided throbbing head ache, nausea, and vomiting. She is 24 weeks pregnant. Her medical history is remarkable for migraine. Which one of the following is the most appropriate management of this patient?
A. Paracetamol.
B. Paracetamol and codeine.
C. Codeine and metoclopramide.
D. Codeine and promethazine.
E. Sumatriptan.
C. Codeine and metoclopramide
This woman is suffering from a migraine attack associate with nauseas and vomiting. In pregnant women with migraine paracetamol is the treatment of choice for mild attacks. For more severe attacks, codeine alone or in combination with paracetamol can be used.
Codeine is not shown to lead to miscarriage or have teratogenic effects on fetus; however, long-term use of opiates can cause withdrawal (abstinence) syndrome in the neonate.
Because paracetamol alone or with codeine would not stop vomiting, an antiemetic should be added as well to control the nausea and vomiting. Metoclopramide is the antiemetic of choice during pregnancy (category A). Promethazine is category C and should be avoided.
TOPIC REVIEW
A step-wise approach to management of an acute migraine attack in a pregnant woman is as follows:
- Paracetamol (first-line) - For acute treatment, paracetamol is safe but often inadequate to control the symptoms. Migraine that does not respond to paracetamol alone may be relieved with combination therapy such as paracetamol (650 to 1000 mg) and metoclopramide (10 mg); paracetamol-codeine.
For migraine with nausea and vomiting, metoclopramide (category A) can be safely added. Prochlorperazine can be used as an alternative to metoclopramide.
Women with migraine that has not responded to these drugs after several days should be evaluated for provoking factors and treated more aggressively with the following medications in a step-wise approach.
- NSAIDs and aspirin (second-line) - NSAIDs are second-line options, and safest in the second trimester. In the first trimester, an association with miscarriage and some birth defects (ventricular septal defect, gastroschisis) has been suggested. In the third trimester, their use should be limited to fewer than 48 hours due to concerns about premature ductus arteriosus closure, platelet inhibition, and oligohydramnios.
- Opioids (third-line) - Opiates (e.g., oxycodone, meperidine, morphine, etc.) can be given by rectal, intravenous, or intramuscular administration.
Opioids can be useful for treatment in women with nausea and vomiting.
- Triptans (fourth-line) - For moderate to severe symptoms in patients who do not respond to other drugs, triptans can be considered.
NOTE - Ergotamine is absolutely contraindicated throughout pregnancy.
Of the options, codeine plus metoclopramide is the most appropriate one for a severe headache and nausea of this woman.
- AAFP - Treatment of Acute Migraine Headache
- UpToDate - Headache in pregnant and postpartum women * Australian Prescriber
- Therapeutic Guidelines – Neurology
A 35-year-old woman pregnant woman presents to the Emergency Department with left-sided retro-orbital and occipital severe headache associated with nausea and vomiting. She has been suffering from migraine for the past 10 years, and has been on treatment with sumatriptan. Which one of the following drugs if used for treatment of migraine would lead to premature closure of fetal ductus arteriosus?
A. Codeine.
B. Paracetamol.
C. Metoclopramide.
D. Non-steroidal anti-inflammatory drugs (NSAIDs).
E. Sumatriptan.
D. Non-steroidal anti-inflammatory drugs (NSAIDs)
For treatment of migraine in a pregnant woman, NSAIDs should be used with caution and only if paracetamol with or without codeine/metoclopramide fails to control the pain. If NSAIDs are indicated, they should be used not more than 48 hours, and not in late pregnancy, because they are associated with premature closure of fetal ductus arteriosus. Aspirin has the same effect and should be avoided as well.
Other effects of NSAIDs on fetus include:
-Delayed labor and birth
-Oligohydramnios via decreasing the glomerular filtration rate in the fetus
Other mentioned drugs are not associated with premature closure of ductus arteriosus.
- UpToDate- Headache in pregnant and postpartum women
- Australian Prescriber - Treatment of nausea and vomiting in pregnancy
A pregnant woman has come to you because two days after babysitting her friend’s son, he was diagnosed with infectious mononucleosis. She is worried about herself and her baby’s well-being. Currently, she is asymptomatic. Which one of the following will be the most appropriate action for now?
A. Order an ultrasound for assessment of fetal hydrops.
B. Check serology for antibody titres.
C. Reassure her.
D. Tell her to come back if any symptoms develop.
E. Refer her to specialist.
D. Tell her to come back if any symptoms develop
Epstein-Barr virus (EBV) is a human herpes virus with variable incubation period that may cause infectious mononucleosis. This virus can remain latent in the body and become reactivated at a later time.
In Australia, EBV is more common among women aged 15 and 19 years, but in developing countries it is more common among children. The route of transmission is sharing oral secretions (saliva). EBV has an incubation period of 2-7 weeks after exposure.
Clinical features of EBV include:
-Fever
-Sore throat
-Lymphadenopathy
-Characteristic increase in the percentages of monocytes and lymphocytes (mononucleosis and lymphocytosis)
-Hepatosplenomegaly
-Rise in hepatic transaminases
Of all pregnant women, only 3.0% to 3.4% are susceptible and of those infected, only 50% develop clinical disease.
In several recent studies, EBV infection was not transmitted to the fetus and there were no adverse effects. The risk of intrauterine transmission of EBV infection is considered to be low, even when the mother is symptomatic; however, reactivation of EBV in pregnancy may carry a small risk of a shortened pregnancy duration and lower birth weight.
Recent primary EBV infection is diagnosed by the presence of viral capsid antigens (VCA) IgG and IgM antibodies in the absence of antibodies to EBV-associated nuclear antigen which develop 3 to 4 weeks after primary infection.
It is recommended that serology for IgG and IgM antibodies against viral capsid antigens (VCA) be obtained soon after symptoms of infection. About 80% of those infected form antibodies to early antigens, which usually fall to undetectable levels by six months afterwards. The presence of antibodies against early antigens at later times after acute infection indicates possible viral reactivation.
Management is supportive with rest, fluids and analgesia if required. Most pregnant women will have a gradual, uneventful recovery after an acute phase lasting several days to 3 or 4 weeks. Fever usually resolves within two weeks. Abnormal liver transaminases occur in about 10 % of cases. Nausea, anorexia and possibly vomiting can be expected. Significant organomegaly usually resolves within 1 to 3 months. Recovery from severe fatigue may occur quickly, however a full recovery to a feeling of wellbeing may take several months.
This woman has a 3.0 – 3.4% risk of infection with EBV. Even if she shows clinical infection, the potential risks to the fetus are negligible, and she can be safely reassured that no harm from EBV threatens her pregnancy. She, however, may have become infected and show clinical infection, in that case further management (including serology) would be required. For this reason she should be warned that the risk of developing the disease is small yet possible and she should come back for further management if any symptom develops.
Yasmin®, containing drospirenone 3mg and ethinylestradiol 30 mcg, has been released for use in Australia. Which one of the following has made it the preferred oral contraceptive pill among Australian women?
A. It has not weight gain as an adverse effect and maybe associated with weight loss.
B. It has a failure rate of less than other OCPs.
C. It has a protective effect against cervical cancer.
D. It causes less spotting even at the very beginning of use.
E. It has a good effect on acne.
A. It has not weight gain as an adverse effect and maybe associated with weight loss
Yasmin® has been shown to be associated with less fluid retention and weight gain as one of the complication of COCs, and this the main reason Yasmin® is preferred by most women suffering from this adverse effect. Unlike older progestogens, drospirenone is associated with no increase in weight or eve slight weight loss due to its anti-mineralocorticoid effects.
OPTION B : The failure rate Yasmin® is about the same as other COCs. There is no study showing that use of Yasmin® is associated with less incidence of cervical cancer as a long-term adverse effect of COCs (option C) . As with all COCs, Yasmin can cause irregular bleeding and spotting within the first few months of use (option D).
OPTION E : The progesterone component – drospirenone has antiadorgenic effects and is slightly more effective in treatment of acne, but compared to other COCs, the difference is not significant enough to make it preferable in terms of acne treatment of prevention.
- AMC Handbook of Multiple Choice Questions – page 533
*http://www.ncbi.nlm.nih.gov/pubmed/16203653
*http://www.uptodate.com/contents/hormonal-therapy-
A 29-year-old obese woman comes to you for prescription of oral contraceptive pill (OCP). She weighs 115kg and has a BMI of 35. She also has hirsutism and acne. She mentions that she has migraine headaches at occasions, associated with pins and needles in her left arm. Which one of the following would be the most appropriate contraception method for her?
A. Condoms.
B. Implanon®.
C. An OCP containing ethinyl estradiol and norgestrel.
D. An OCP containing ethinyl estradiol and cyproterone acetate.
E. An OCP containing ethinylestradiol and drospirenone.
A. Condoms
This woman has classic migraine associated with focal neurological findings. In such patients use of OCP preparation of any kind containing estrogen is absolutely contraindicated. Progesterone has androgenic effects such as hirsutism, acne and weight gain. For a woman of her weight, progesterone of any kind (norgestrel, drospirenone, cyproterone, etc) is better avoided; therefore, a barrier method such as male condoms will be the most appropriate advice.
When choosing a combined oral contraceptive pill (COCP), it is recommended that preparations containing 20-30 mcg ethinylestradiol be considered first. The progesterone component can be norgestrel, drospirenone, cyproterone, etc; however, the preparations containing norgestrel are cheaper and more affordable for patients.
Considerations should be given to the progesterone type for particular patients:
i) Patients with bothersome fluid retention and weight gain as a side effect of COCPs can be prescribed preparations containing drospirenone (Yaz®, Yasmin®). Drospirenone has anti-mineralocorticoid activity and does not lead to fluid retention. It may even be associated with slight weight loss.
ii) If the patient has probable polycystic ovarian syndrome (PCOS), a preparation containing cyproterone acetate is preferred.
TOPIC REVIEW
Combination oral contraceptives (COCs) with antiandrogenic progestins are a subclass of COCs. These include agents that contain cyproterone acetate, drospirenone, or dienogest plus an estrogen (ethinylestradiol).
Yasmin ® (ethinylestradiol 30mcg/drospirenone 3 mg) and Yaz ® (ethinylestradiol 20mcg/drospirenone 3mg) are the two drospirenone-containing oral contraceptives. These two COCs are shown in studies to be effective in treatment of acne and other androgenic effects such as hirsutism, PCOS, etc; however, they are only slightly superior to other COCs. The effective component of COCs against acne is estradiol and all low-dose COCs are estrogen dominant, which effectively makes all of these agents antiandrogenic and effective.
Some studies, however, suggest that their use is associated with a 2- to 3-time increase in venous thromboembolic events. Although their use is only contraindicated in the presence of active venous thromboembolism (e.g. DVT, PE, etc)
- AMC Handbook of Multiple Choice Questions – page 533
*http://www.uptodate.com/contents/hormonal-therapy-
*http://www.fpnotebook.com/mobile/gyn/pharm/Ysmn.ht
A 36-year-old woman presents to your clinic for advice regarding diabetes mellitus. She has 3 children, and was diagnosed with gestational diabetes mellitus in her second pregnancy at the age of 32 years. Which one of the following tests would be the most appropriate screening test for her?
A. Fasting blood sugar (FBS), 3yearly.
B. Oral glucose tolerance test (OGTT), 2 yearly.
C. OGTT, yearly.
D. HbA1C, now.
E. FBS, 2 yearly.
D. HbA1C, now
All women diagnosed with GDM should have a 75 g OGTT at 6-12 weeks postpartum. Additionally and due to the fact that women with GDM have a 50% risk of developing type 2 DM within 20 years, they need to be tested for DM. Based on current guidelines by the Royal Australian College of General Practitioners (RACGP) all women with GDM should undergo a fasting OGTT with 75 g glucose at weeks 6-12 postpartum and fasting blood sugar (FBS) or glycated hemoglobin (HbA1C) every 3 years thereafter. Since this woman has not been screened for diabetes until now, the most important step would be ordering an FBS or HbA1c now.
NOTE - Different guidelines mention different intervals for DM screening in women with history of GDM. For example Therapeutic Guidelines and Australian Diabetes in Pregnancy Society, recommend 2-yearly 75 g OGTT as the screening test of choice. We have chosen the correct answer based on RACGP guidelines which are important AMC MCQ references for the AMC MCQ exam.
- RACGP - Gestational diabetes mellitus
- Therapeutic Guidelines
A 28-year-old primigravida woman presents at 24 weeks gestation after she noticed vaginal bleeding of approximately 50ml. Last week, she also had a 10ml vaginal bleeding, for which ultrasonography was performed revealing placenta previa grade IV. Which one of the following would be the most appropriate next step in management?
A. Transvaginal ultrasonography.
B. Blood group and cross match.
C. Anti-D (RhoGAM).
D. Abdominal ultrasonography.
E. Induction of labor.
B. Blood group and cross match
The scenario describes vaginal bleeding in the second half of the pregnancy (antepartum hemorrhage), most likely caused by the previously diagnosed grade IV placenta previa.
The usual history includes a first episode of vaginal bleeding occurring at home. Bleeding is not often heavy. This can be followed by more episodes of bleeding.
The first priority in cases presenting with antepartum hemorrhage is assessment of vital signs, estimation of the blood loss (both clinically and by using laboratory tests), determination of blood group and cross matching.
Even an asymptomatic currently-stable placenta previa can lead to massive bleeding at anytime. This is even more likely with higher grades of placenta previa, or if the first episode of bleeding occurred prior to 30 weeks’ gestation.
The abdomen should be palpated for any uterine tenderness, contraction and fetal presentation. An abdominal ultrasonography is needed to confirm the diagnosis as well as gathering additional information regarding the situation.
Fetal well-being should be assessed by cardiotocography, and corticosteroids administered (if indicated) to promote fetal lung maturity because the feared risk of premature labor.
No vaginal examination should be made before placenta previa is excluded by ultrasound examination, because this can lead to torrential bleeding. Transvaginal ultrasonography is more accurate than transabdominal and first choice of investigation if indicated. The probe is entered into the vaginal to the extent and in an angle that prevents the probe from reaching the cervical os and is safe to perform in experienced hands.
Anti-D (RhoGAM) should be administered, but not as the first priority.
TOPIC REVIEW
Placenta previa is implantation of placenta, either partially or wholly, in the lower uterine segment below (previa) the fetal presenting part. Placenta previa is classified as:
*Grade I – placenta is in lower segment, but the lower edge does not reach the internal os.
*Grade II – Lower edge of placenta reaches the internal os, but does not cover it.
*Grade III – Placenta covers the internal os partially.
*Grade IV – placenta covers the internal os completely
Placenta previa occurs is 0.5-1.0% of pregnancies and accounts for 20% of all cases of antepartum hemorrhage. It is three times more common in multiparous women.
Risk factors for placenta previa include:
-Large placental area e.g. multiple pregnancy
-Advanced age
-High parity
-Deficient endometrium due to pre-existent uterine scar (e.g. previous cesarean section) -Endometritis
-Manual removal of placenta
-Curettage (especially for miscarriage or termination of pregnancy)
-Submucous fibroids
NOTE - if a routine ultrasound examination is made at 18 weeks, the report may show that there is a low-lying placenta, but in over 85% of cases, the placenta will be normally situated by the time of delivery, as the lower uterine segment does not develop fully until late in the third trimester. Another ultrasound examination should be performed at about the 34th week, or earlier if vaginal bleeding occurs.
Presentation
Most cases of placenta previa present with painless bright red vaginal bleeding of different amount without uterine contractions. Rarely, uterine contraction and tenderness may be present.
In approximately one-third of affected pregnancies, the initial bleeding episode occurs prior to 30 weeks of gestation; this group is more likely to require blood transfusions and is at greater risk of preterm delivery and perinatal mortality than women whose bleeding begins later in the course of pregnancy. An additional one-third of patients become symptomatic between 30 and 36 weeks, while most of the remaining patients have their first bleed after 36 weeks. About 10% of women reach term without bleeding.
For an individual patient, it is not possible to predict whether a bleed will occur, nor the gestational age, volume, or frequency of bleeding. However, higher grades of placenta previa bleed earlier and more compared with lower grades. There is often fetal malpresentation or usually high and mobile fetal presenting part.
Management :
1. Check vital signs, establish IV access and start fluids (if indicated)
2. Cross-matched blood and blood products should be readily available in anticipation of massive hemorrhage.
3. Gentle abdominal palpation to see is uterine tenderness/contractions are present, and to estimate the gestational age as well as the fetal presenting part.
4. Ultrasonography to confirm the diagnosis
5. Assessment of fetal well-being
6. Anti-D (RhoGAM) if indicated
7. Corticosteroids (if indicated)
8. Decide for outpatient versus inpatient management.
Outpatient management maybe considered:
If the patient is stable and there is no current hemorrhage, the patient can be managed in outpatient setting:
Explain the frequency and severity of recurrent bleeding is unpredictable and carries the risk of fetal and maternal complications Advise the woman to seek immediate hospital care if contractions or vaginal bleeding occurs
Ensure emergency transport access to hospital
Admit if active bleeding
If inpatient care is indicated:
Cesarean delivery should be considered where there is maternal or fetal instability despite vigorous management, regardless of gestational age; otherwise, cesarean delivery at 37 weeks should be performed. Vaginal delivery may be considered in low-risk women with low-grade placenta praevia.
*http://www.sahealth.sa.gov.au/wps/wcm/connect/b1c6
* Llewellyn-Jones – Fundamentals of Obstetrics and Gynaecology – Elsevier Mosby – 9th Edition
* Royal College of Obstetrics and Gynaecology – Green-top Guideline No.27
Which one of the following is unlikely to predispose to postpartum hemorrhage?
A. Uterine fibroids.
B. Multiple gestation.
C. Von Wille brand disease of the mother.
D. Oligohydramnios.
E. Prolonged labor.
D. Oligohydramnios
Predisposing factors to postpartum hemorrhage include:
-Uterine fibroids
-Multiple pregnancy
-Polyhydramnios
-Maternal history of bleeding disorders
-Prolonged labor
-Retained placenta
-Instrumental delivery
-Pregnancy-induced hypertension and pre-eclampsia
-Past history of postpartum hemorrhage
-Multiparity
-Obesity
Oligohydramnios does not increase the risk of postpartum hemorrhage.
A 28-year-old woman, who is 22 weeks pregnant in her second pregnancy, presents for evaluation of a vulval ulcer. The swab taken confirms the diagnosis of herpes simplex type II (HSV-II) infection. When she is informed, she becomes quite surprised as neither she, nor hes husband has ever had this infection before and insists to know the source of the infection. Furthermore, she is very concerned about her baby’s wellbeing and asks whether her condition may affect it. Which one of the following is the most appropriate advice in this regard?
A. Most of neonates with neonatal herpes present with mucocutaneous lesions.
B. Although treatment with antiviral agents such as aciclovir will reduce the rate recurrence of the disease, they cannot be used during pregnancy because of their adverse effects on the fetus.
C. The primary infection is commonly asymptomatic.
D. Unless she has had a new sexual partner recently, this problem could not have been acquired sexually.
E. The risk of neonatal herpes is much higher with recurrent maternal infection compared with primary infection.
C. The primary infection is commonly asymptomatic
Unlike what is usually thought, genital herpes infection is asymptomatic in 75% of the cases. Therefore, this woman may have contracted the infection from his asymptomatic husband.
Herpes simplex virus type II exclusively passes on through skin-to-skin contact during sexual activity; therefore having a sexual partner infected with HSV-II is essential for contracting the disease. Herpes simplex virus type I, on the other hand can be transmitted from non-sexual skin contacts e.g. kissing.
With herpetic infection during pregnancy, antiviral therapy can be considered. Studies have shown that aciclovir is the safest to use in pregnancy.
The risk of fetal infection is significantly higher in primary infection, because in recurrences the fetus has already received IgG against HSV, passively from the mother. It should be born mind that neonatal infection with recurrent infection is also possible, but far less likely compared with primary infection. If primary HSV is contracted before 30 weeks, the risk of shedding HSV during a normal birth is 7 % with an overall risk of ≤ 3 % for neonatal HSV disease.
*https://www.sahealth.sa.gov.au/wps/wcm/connect/91b
* Australian society for infectious diseases – Management of Perinatal Infections (2014)
Which one of the following is not helpful in prevention of transmission of HIV infection from a pregnant woman to her baby?
A. Maternal antiretroviral therapy.
B. Peripartum intravenous zidovudine.
C. Elective cesarean section.
D. Neonatal antiretroviral treatment.
E. Breastfeeding.
E. Breastfeeding
Interventions to prevent perinatal transmission of HIV include:
-Maternal antiretroviral therapy
-Peripartum intravenous zidovudine
-Elective cesarean section
-Neonatal antiretroviral treatment
-Bottle feeding
Breast feeding is contraindicated in HIV positive mothers because of increased chance of transmission of HIV infection.
Without intervention, the risk per cent of HIV transmission to the fetus is approximately 20-30%. With intervention this rate the risk decrease to less than 2%.
*https://www.sahealth.sa.gov.au/wps/wcm/connect/72e
A 27-year-old woman, diagnosed with gestational diabetes at 28 weeks pregnancy, has been being managed at a high-risk pregnancy clinic until 38 weeks when she vaginally deliveres a healthy 4-kg baby without any complications. Which one of the following is correct regarding follow-up?
A. 75g oral glucose tolerance test performed between weeks 6 and 12 after delivery.
B. Check fasting blood glucose level in 6 months after delivery.
C. HbA1C as soon as possible.
D. Fasting lipids.
E. No further action is required.
A. 75g oral glucose tolerance test performed between weeks 6 and 12 after delivery
Less than 10% of women with GDM remain hyperglycemic after delivery. The management of these women requires ongoing care from a diabetes or medical clinic in collaboration with their general practitioner.
The following are recommendations for women with gestational diabetes after delivery:
-Checking a random blood glucose level the day after delivery
-4-point blood glucose level (BGL) measurement on the day prior to discharge (fasting and two hours post meals for three meals)
-Cease blood glucose monitoring if BGL is within normal range.
-If there is elevated BGL (above 10 mmol/L) contact the medical registrar / diabetes educator – if BGLs are considerably elevated, after-hours contacts are justified.
-It is recommended that women who have had GDM visit their GP for a follow-up oral glucose (75 gr) tolerance test at 6-12 weeks postpartum, and every 1-2 years thereafter. -Lifestyle counselling.
-A summary letter is provided to the woman’s GP for follow-up care.
This woman should have a random BGL the day after delivery, 4-point BGL test the day before discharge, and an oral glucose tolerance test using 75g glucose between weeks 6 and 12 after delivery.
Checking HbA1C and fasting lipids would be required if this patient is found to have type 2 diabetes mellitus.
A 17-year-old girl presents to your office for consultation because she has never had a period. On examination. she has normal height for her age. She also has normally developed breasts, pubic hair, axillary hair and genitalia. Which one of the following would be the most likely cause of her amenorrhea?
A. Turner syndrome.
B. Prolactinoma.
C. Absent uterus.
D. A hypothalamic lesion.
E. Congenital adrenal hyperplasia.
C. Absent uterus
Primary amenorrhea is defined as the absence of menses by the age of 16 in the presence of normal growth and secondary sexual characteristics, or by the age of 14 in the absence of these features. Primary amenorrhea, in the presence of normal sexual development is suggestive of abnormalities such as absent or malformed uterus or absent or septated vagina or obstructed menstrual flow. Ultrasonography is the best initial investigation to find whether uterus is present or absent.
If uterus is present, amenorrhea may be due to menstruation flow obstruction caused by conditions such as imperforated hymen or vaginal septum. If uterus is absent or abnormal, chromosomal abnormalities are the most likely cause.
In the presence of normal breast development, normal pubic and axillary hair growth, normal genitalia and normal height, hypothalamic hypogonadism (option D), pituitary lesions and chromosomal abnormalities such as Turner syndrome (option A) are very unlikely to be the cause of amenorrhea in this girl.
With prolactinoma (option B), patients are expected to have galactorrhea, headache, or visual symptoms along with menstrual abnormalities.
Congenital adrenal hyperplasia (option E) has different clinical picture with manifestations much earlier in life.
- Medscape - Amenorrhea
- NSW Health - Amenorrhoea
A 38-year-old female at 33 weeks’ gestation sustains a road traffic accident at 90 km/hour. In the emergency department and on examination, she is found to be pale, with a heart rate of 112 bpm, blood pressure of 95/55 mmHg and respiratory rate of 18 breaths per minute. Her oxygen saturation is 95% on room air. Fetal heart rate is audible at 102bpm. The uterus is tense and tender. Which one of the following is the most likely diagnosis?
A. Ruptured uterus.
B. Liver laceration.
C. Placental abruption.
D. Ruptured spleen.
E. Placenta previa.
C. Placental abruption
This patient has signs and symptoms consistent with clinical diagnosis of placental abruption.
Trauma in last trimester of pregnancy could be dangerous to both the mother and the fetus. Motor vehicle accidents, by force of deceleration, cause placental separation.
Placental abruption is characterized by painful, tender uterus which is often contracting. The condition leads to maternal hypovolemic hypotension and consequent fetal distress represented by fetal bradycardia and repetitive late decelerations.
The amount of vaginal bleeding is not usually an appropriate indicator to severity of placental abruption, because bleeding could be very severe or it may be concealed between uterine wall and placenta in form of a hematoma.
OPTION A : Uterine rupture is characterized by severe abdominal pain and tenderness, cessation of contractions and loss of uterine tone. It is associated with mild to moderate vaginal bleeding and fetal bradycardia or loss of heart sound. Compared to placental abruption, there uterus is less tense and tender.
OPTION B and D : Ruptured spleen and liver laceration present may justify the low blood pressure, tachycardia and fetal bradycardia but not the tense, tender and contracting uterus.
OPTION E : Placenta previa presents with sudden, painless bleeding with bright red blood. There is no uterine tenderness. This diagnosis is not consistent with the clinical picture.
Which one of the following is the most common cause of postpartum hemorrhage?
A. Uterine atony.
B. Laceration of genital tract.
C. Uterine rupture.
D. Uterine inversion.
E. Coagulopathy.
A. Uterine atony
The traditional definition of a primary postpartum hemorrhage (PPH) is a blood loss of 500 mL or more in the first 24 hours. Postpartum hemorrhage can be minor (500-1,000 mL) or major (> 1,000 mL). A major PPH can be further described as moderate (1,000-2,000 mL) or severe (> 2,000 mL).
A widely accepted definition of postpartum hemorrhage (PPH) in many institutions is a blood loss of 600 ml for vaginal delivery and 750 ml for cesarean delivery. The classification of PPH in relation to the amount of blood loss is problematic, largely due to a well-documented underestimation of blood loss. A clinically relevant alternative is a substantial fall in the hematocrit e.g. 10 %.
PPH causes include:
a) Abnormalities of uterine contraction (Tone) 70 %
b) Genital tract trauma (Trauma) 20 %
c) Retained products of conception or invasive placenta (Tissue) 10 %
d) Abnormalities of coagulation (Thrombin) < 1 %
Postpartum uterine atony is by far the most common cause of PPH.
The most common risk factors for uterine atony include:
-Multiple pregnancy
-Polyhydramnions
-Macrosomia
-Prolonged labor
-Multiparity
A 26-year-old primigravida presented with vaginal bleeding at 16 weeks gestation. She is Rh-negative, but her partner RH-positive. On examination, cervix is dilated and products of conception are visible. Pelvic ultrasound confirms the diagnosis of spontaneous abortion. Which one of the following would be the most appropriate advice regarding Anti-D antibody (RhoGAM)?
A. Give Anti-D at 28 weeks gestation in next pregnancy.
B. Anti-D is not indicated in this situation.
C. Genetic analysis should be performed before making any decision.
D. Give anti-D now.
E. Advice that Anti-D antibodies would develop within 7 days.
D. Give anti-D now
Rhesus (Rh) negative women who deliver an Rh positive baby or who are otherwise exposed to Rh positive red blood cells are at risk of developing anti-Rh antibodies (RhD) and should receive RhD antibody (RhoGAM®) in current pregnancy.
Rh positive fetuses/neonates of these mothers are at risk of developing hemolytic disease of the fetus and newborn, which can be lethal or associated with serious morbidity.
TOPIC REVIEW
An Rh-negative mother has no antibodies against Rh (D) antigen. If she is pregnant and the fetus is Rh-positive any mixing up the fetus’ blood to the mother’s will trigger an immune response by lymphocytes and will lead to production of anti-Rh antibodies in the mother’s blood (anti-D antibody). RhoGAM® is anti-D IgG passive antibody that will eliminate the D-antigen from the mother’s blood, before the mother’s immune system start to sensitize. It is administered intramuscularly (IM).
RhoGAM is available is two forms:
1. CLS-250 IU – one dose contains 50μg of anti-D antibody
2. CLS-265 IU – one dose contains 125μg of anti-D antibody
Each 300μg of anti-D antibody neutralizes 15 mL of fetal packed red cells (30 mL of whole blood)
RhoGAM should be administered within the first 72 hours after the precipitating even, however, with much less success rate it can be given up to 9-10 days.
INDICATIONS
-
First trimester indications (up to and including 12 weeks)
-Spontaneous abortion (complete, incomplete, or missed abortion) probably excluding threatened abortion
-Induced abortion (medically or by D&C)
-Ectopic pregnancy
-Chorionic villous sampling (this procedure is performed at 10-12 weeks)
-Molar pregnancy – chorionic villi may contain D-antigen
MANAGEMENT - in the first 12 weeks the maximum amount of fetal blood that can mix with mother’s is 2.5 mL of RBC (5 mL whole blood). A single dose of CLS 250-IU is sufficient for neutralization of circulating fetal D-antigen. Multifetal pregnancies need extra doses. In a singleton pregnancy Kleihauer-Betki test is not indicated because the amount of fetomaternal blood mix is not significant and a single dose CLS – 250 will be enough.
-
Second and third trimester indications
-Spontaneous or induced abortions of all kind after 12 weeks
-Amniocentesis (this procedure is performed > 15 weeks)
-Cordocentesis (this procedure is performed >20 weeks)
-Fetal blood sampling
-Fetal death
-External cephalic version of breech presentation (successful or unsuccessful)
-Blunt abdominal trauma in pregnancy considered sufficient to cause fetomaternal haemorrhage
-Antepartum haemorrhage (revealed or concealed) in the second or third trimester (e.g. placental abruption, placenta praevia, etc)
MANAGEMENT - in the second or third trimester CLS 625 (containing 125μg anti D-antibody) should be used. A Kleihauer-Betki test is indicated in the second or third trimester events to assess the need for extradoses of CLS 625 IU. Ideally the sample for the test should be collected within 15 minutes of the precipitating event, but if not possible it can be performed up to 72 hours.
*RhoGAM will remain in maternal circulation for up to 6 weeks.
**RhoGAM should not be given to women in which anti-D antibody has already formed.
Routine administration of RhoGAM - in the absence of a precipitating event, RhoGAM (CLS – 625 IU) is routinely given to all Rh-negative pregnant women at 28 weeks and 34 weeks of pregnancy.
Within the first 72 hours postpartum all R-negative women whose baby is Rh-positive should receive a dose of CLS-625 IU and undergo quantification of fetomaternal blood mix using Kleihauer-Betki test to evaluate whether extra doses of CLS-625 IU are indicated.
*http://www.australianprescriber.com/magazine/23/2/
*https://www.nhmrc.gov.au/_files_nhmrc/publications
* Royal Australian and New Zealand College of Obstetricians and Gynaecologists – College Statement C-Obs 6 (2015)
A 31-year-old G2P1 woman presented to the maternity unit at 38 weeks gestation and in labour. Her previous pregnancy led to caesarean section and delivery of a healthy baby. The current pregnancy had been uneventful without any remarkable problems in antenatal visits except first trimster nausea and vomiting. On arrival, she had a cervical dilation of 4 cm and the fetal head was at -1 station. After 5 hours, the cervical length and fetal head station are still the same despite regular uterine contractions. Suddenly, there is sudden gush of blood of approximately 1000 mL and the fetal heart rate drops to 80 bpm on CTG. Which one of the following could be the most likely cause?
A. Lower genital tract lacerations.
B. Placenta previa.
C. Placenta accreta.
D. Uterine atony.
E. Ruptured uterus.
E. Ruptured uterus
With previous caesarean section in history and prolonged active phase of the first stage of the labour, uterine rupture would be the most likely cause of the bleeding.
Maternal manifestations of uterine rupture are variable and include the following :
1- Constant abdominal pain – pain may not be present in sufficient amount, character, or location to suggest uterine rupture and pain may be partially or completely masked by regional analgesia.
2- Signs of intraabdominal hemorrhage (a strong indicator) - Although hemorrhage is common, the signs and symptoms of intraabdominal bleeding in cases of uterine rupture, especially those cases not associated with prior surgery, may be subtle.
3- Vaginal bleeding – Vaginal bleeding is not a cardinal symptom, because it may be modest, despite major intraabdominal hemorrhage.
4- Maternal tachycardia and hypotension
5- Cessation of uterine contractions
6- Loss of station of the fetal presenting part
7- Uterine tenderness
Fetal bradycardia (as seen in this case) is the most common and characteristic clinical manifestation of uterine rupture. Variable or late decelerations may precede the bradycardia, but there is no fetal heart rate pattern pathognomonic of rupture. Furthermore, fetal heart rate changes alone have low sensitivity and specificity for diagnosing uterine rupture.
Postpartum uterine rupture is characterized by pain and persistent vaginal bleeding despite use of uterotonic agents. Hematuria may occur if the rupture extends into the bladder.
NOTE - A history of previous uterine surgery (e.g. cesarean section) typically alerts the obstetrician to the possibility of uterine rupture in symptomatic women.
When uterine rupture is suspected, immediate cesarean section should be performed to save both the mother and the baby. Definite diagnosis of uterine rupture is always made after laparotomy.
*http://www.sahealth.sa.gov.au/wps/wcm/connect/0f2c
*http://www.uptodate.com/contents/rupture-of-the-un
A 50-year-old woman presents to your GP practice because of persistent vaginal bleeding for the last 3 days. Her last menstrual period occurred one year and a half ago. She mentions that just before the bleeding she felt reduction in hot flushes she has been experiencing for the past year. She is sexually active and has had regular pap smears with a normal one 6 months ago. Which one of the following is the most likely diagnosis?
A. Endometrial cancer.
B. Cystic glandular endometrial hyperplasia.
C. Atrophic vaginitis.
D. Cervical cancer.
E. An episode of ovarian follicular activity.
E. An episode of ovarian follicular activity
When menopause occurs, estrogen levels fall. Decreased estrogen levels result in increased production of follicular stimulating hormone (FSH) and very high levels of circulating FSH. At times, these high levels of FSH cause a remaining follicle to become active, resulting in menstruation. At such occasions, produced estrogen by the active follicle leads to decreased symptoms of menopause such as hot flushes, vaginal dryness, etc.
OPTION A : Endometrial cancer could be a possibility but it is very uncommon in premenopausal women or those women within the first 2-3 years of their menopause.
OPTION B : Cystic glandular hyperplasia is a result of long-term exposure of endometrium to unopposed estrogen such as in women with anovulatory cycles (e.g., in PCOS). There is no clue in history to suggest endometrial hyperplasia.
OPTION C : Atrophic vaginitis presents differently with decreased vaginal lubrication and dyspareunia, vaginal discharge or bleeding, dysuria, etc. Although not impossible, it is uncommon for vaginal atrophy to occur within the first 2-3 years of menopause.
OPTION D : With a normal cervical screening 6 months ago, the cervical cancer is very unlikely.
- AMC Handbook of Multiple Choice Questions – page 528
Which one of the following is highly suggestive of polycystic ovarian syndrome (PCOS)?
A. Elevated FSH.
B. Elevated LH.
C. Elevated testosterone levels.
D. Decreased free testosterone.
E. Hyperprolactinemia.
C. Elevated testosterone levels
Polycystic ovarian syndrome (PCOS) is associated with the following:
- Clinical or biochemical hyperandrogenism. Clinical features suggestive of PCOS include hirsutism, acne, deepened voice, etc
- Menstrual dysfunction, irregularity or lack of ovulation.
- Polycystic ovaries on the ultrasound.
Only the first two mentioned features will suffice for establishing the diagnosis of PCOS, because cystic structures on ultrasound are often not seen, particularly in women who have been on treatment.
The following hormonal changes are seen in PCOS:
-Serum FSH (follicle stimulating hormone) is either normal or low.
-Elevation of LH (luteinizing hormone)
-A normal LH/FSH ratio in premenopausal women is about 1:1, while in PCOS this ration may be 2:1 or 3:1. It should be noted that approximately 30% of patients with established diagnosis of PCOS will have a LH/FSH ration of 1:1.
-Serum estrogen level is either decreased or normal. Overall, estrogen level is very nonspecific for diagnosis of PCOS.
-Serum free testosterone is the first-line investigation in patients suspected of having PCOS. Hyperandrogenism is one of the essential criteria to establish the diagnosis of PCOS.
-Prolactin level is usually normal or mildly elevated, possibly due to chronic estrogen exposure.
Of the options, an elevation of testosterone is diagnostic for PCOS.
*https://www.mja.com.au/journal/2011/195/6/assessme
*https://www.nhmrc.gov.au/_files_nhmrc/publications
*http://emedicine.medscape.com/article/256806-wor
A 21-year-old woman, who underwent dilation and curettage for termination of an unwanted pregnancy 4 days ago, presents to the emergency department with fever and purulent vaginal discharge. Her pregnancy was the outcome of sex with an stranger, while intoxicated in a party. Which one of the following organisms is most likely to be the cause of her problem?
A. Mycoplasma hominis.
B. E.coli.
C. Chlamydia trachomatis.
D. Chlamydia trachomatis and vaginal pathogens.
E. Neisseria gonorrhea.
D. Chlamydia trachomatis and vaginal pathogens
With the presence of fever and purulent vaginal discharge, this woman has inflammatory pelvic disease. The predominance of microbial etiology depends on whether the PID is sexually-acquired or not:
- Non-sexually acquired PID – This type is usually caused by mixed pathogens originating from vaginal flora, including anaerobes, facultative bacteria, Mycoplasma hominis and sometimes E.coli. This type of PID often occurs postpartum or following instrumentation or gynecological surgeries.
- Sexually acquired PID – Chlamydia trachomatis and Neisseria gonorrhea are the two most common causes of pelvic inflammatory disease (PID) in sexually active women. In Australia chalmydia is much more common.
In patients presenting for termination of pregnancy, especially those with risk factors for Chlamydia, such as not having a constant sexual partner, preoperative swabs have shown that Chlamydia infection is present in up to 15%. For this patient combination of vaginal pathogens and Chlamydia trachomatis would be the most likely etiology.
If the symptom was purulent vaginal discharge, and no fever was present, the diagnosis would have been cervicitis (and not PID). Chlamydia trachomatis is the most common cause of cervicitis. Vaginal pathogens do not cause cervicitis.
NOTE - for the exam purpose remember the following:
- The most common cause of post-procedural PID is vaginal pathogens
- The most common cause of post- procedural PID in women, who are high risk for STDs as well is Vaginal pathogens PLUS Chlamydia
- The most common cause of PID, unrelated to surgical procedures is Chlamydia trachomatis
—
* Therapeutic Guidelines – Antibiotic; available from: http://tg.org.au * AMC Handbook of Multiple Choice Questions – page 526
*http://emedicine.medscape.com/article/256448-overv
A 40-year-old woman with history of tubal ligation presents with complaint of irregular heavy menstrual bleeding. She is known to have multiple uterine fibroids. Other significant point in history is dilation and curettage 6 months ago with no improvement in her symptoms, or no diagnosis for a cause. On examination, the uterus has the size of a 12-week pregnant uterus. Laboratory studies are significant for a hemoglobin level of 85 g/L. Which one of the following is the next best step in management?
A. Total abdominal hysterectomy.
B. Endometrial ablation.
C. Myomectomy.
D. Combined oral contraceptive pills.
E. NSAIDs.
A. Total abdominal hysterectomy
Uterine leiomyomas (fibroids) are benign tumors of uterus. Since histological confirmation of the clinical diagnosis in not necessary in most cases, asymptomatic uterine leiomyomas only require follow-up without any specific intervention.
Symptoms and complications demand intervention. Hysterectomy is the definitive treatment. Other alternatives include myomectomy with various techniques, endometrial ablation, uterine artery embolization and myolysis.
Hysterectomy is indicated in the following situations:
a) Women with acute hemorrhage who do not respond to other therapies
b) Women who have completed childbearing and have current or increased future risk of other diseases (cervical intraepithelial neoplasia, endometriosis, adenomyosis, endometrial hyperplasia, or increased risk of uterine or ovarian cancer) that would be eliminated or decreased by hysterectomy
c) Women who have failed prior minimally invasive therapy for leiomyomas
d) Women who have completed childbearing and have significant symptoms, multiple leiomyomas, and a desire for a definitive treatment
Total abdominal hysterectomy is the procedure of choice if hysterectomy is planned.
For women, who desire to preserve the ability of child bearing, a course of gonadotropin releasing hormone (GnRH) agonists, followed by myomectomy is the treatment of choice.
NOTE - Leiomyomas are the most common indication for hysterectomy, accounting for 30% of hysterectomies in white women and over 50% of hysterectomies in black women.
This woman, who does not wish to have more children and has undergone tubal ligation, and has bothersome symptoms and anemia, total abdominal hysterectomy is the most appropriate treatment option.
OPTION B : Endometrial ablation is inferior to hysterectomy. With endometrial ablation, amenorrhea is achieved; however, leiomyomas remain untreated.
OPTION C : Myomectomy is not advisable, unless further fertility is desired. There is a likelihood of recurrence that may necessitate further surgeries. Furthermore, when the uterus is enlarged with multiple leiomyomas, it will be very difficult to remove them all. Remaining leiomyomas can grow over time and produce the symptoms again.
OPTION D : Hormonal methods including combined contraceptive pills, progesterone-only method, or diazole, has been used with anecdotal effects on some symptoms such as menorrhagia. Some have controversial efficacy and some with proven efficacy have undesirable adverse effect profiles that limit their use.
OPTION E : NSAIDs have not been extensively studied in leiomyomas-related menorrhagia.
NSAIDs do not appear to reduce blood loss in women with leiomyomas, but they decrease painful menses and can be useful for this purpose.
- UpToDate - Uterine fibroids (leiomyomas): Treatment overview
You are giving advice to a young couple about contraception. After explanation of different methods, they choose to use ‘rhythm method’ (calendar calculation). The woman has menstrual cycles of between 26 and 29 days. Which one of the following abstinence periods would be the correct advice?
A. From day 4 to day 17.
B. From day 6 today 17.
C. From day 8 today 19.
D. From day 9 today 19.
E. From day 12 to day 17.
B. From day 6 today 17
Regardless of the length of the menstrual cycle, ovulation occurs 14 days (2 weeks) before menstruation. In other words, luteal phase has always a fixed duration of 14 days; therefore, in this woman ovulation occurs on day 12 at earliest and on day 15 at latest. After ovulation, there is a window of 24-36 hours, during which the egg can be fertilzied by sperm; therefore, intercourse should not be resumed until the latest day when ovulation is possible plus an additional 48 hours (15 + 2=17).
In the presence of adequate and normal cervical mucus, sperm can survive up to 6 days. To calculate the day from which abstinence should be started, 6 should be subtracted from the earliest date of possible ovulation (12 here) (12-6=6).
The appropriate advice for this couple will be abstinence between days 6 and 17.
NOTE - calculation of the abstinence period is by the following formula:
-Beginning of the abstinence = subtract 6 days from the earliest day of the cycle on which ovulation can occur
-End of abstinence = Add 2 days to the latest day of the cycle on which ovulation may occur
For example, in a patient with menstrual cycles between 26 and 32 days:
-Possible days of ovulation: earliest=26-14=12th day of menstrual cycle; latest=32-14=18th day of menstrual cycle
-Beginning of the abstinence = 12-6=6th day of menstrual cycle
-End of abstinence =18 +2 =20th day of menstrual cycle
- AMC Handbook of Multiple Choice Questions – pages 525-526
A 32-year-old woman presents with complaint of mucopurulent greenish vaginal discharge. Which one of the following organisms is the most likely cause of this presentation?
A. Candida albicans.
B. Chlamydia trachomatis.
C. Gardnerella vaginalis.
D. Mycoplasma hominis.
E. E.coli.
B. Chlamydia trachomatis
Of the given options, only Chlamydia trachomatis can cause a thick mucopurulent greenish yellow vaginal discharge. Trichomonas vaginalis (not an option here) can also present with thin frothy yellow-green vaginal discharge that is offensive. Neisseria gonorrhea can also present with mucopurulent vaginal discharge.
OPTION A : The discharge in candida vaginitis is thick, white and non-offensive.
OPTION C and D : Gardnerella vaginalis is the most common cause of bacterial vaginosis. Vaginal discharge of bacterial vaginosis is thin, grey-white and has an offensive (fishy) odor. Mycoplasma hominis is another organism found in bacterial vaginosis.
OPTION E : E.coli may colonize vagina but does not result in infections presenting with green vaginal discharge.
- Australian STI management guideline - Vaginal Discharge * Medscape - Cervicitis
A 65-year-old woman presents to your clinic with complaints of a brownish vaginal discharge for the past 2 months. On examination, atrophic vagina is evident. Which one of the following is the most like diagnosis?
A. Vaginal atrophy.
B. Endometrial cancer.
C. Cervical cancer.
D. Chlamydia infection.
E. Endometrial hyperplasia.
A. Vaginal atrophy
With vaginal discharge of brownish color in a 65-year-old woman, endometrial cancer should always be the first diagnosis to exclude; however, the question asks about the most likely cause of the presentation and not the most important diagnosis to consider.
The brown color of vaginal discharge is often caused by blood. The source of bleeding can be either the uterine cavity or the vagina. Of all women with postmenopausal vaginal bleeding, only 5-10% were found to have endometrial cancer, while approximately 60% had atrophic vaginitis. Of the options, vaginal atrophy due to estrogen-deficient status of menopausal women is the most likely explanation for this presentation.
Estrogen deficiency in postmenopausal years results in urogenital atrophy.
Symptoms of urogenital atrophy include:
-Vaginal dryness
-Vaginal burning or irritation
-Decreased vaginal lubrication during sexual activity
-Dyspareunia, including vulvar or vaginal pain (at the introitus or within the vagina)
-Vulvar or vaginal bleeding (e.g. postcoital bleeding, fissures)
-Vaginal discharge (leucorrhea or yellow and malodorous)
-Pelvic pressure or a vaginal bulge
-Urinary tract symptoms (e.g. urinary frequency, dysuria, urethral discomfort, hematuria)
TOPIC REVIEW
The differential diagnoses of vaginal bleeding in postmenopausal women is less broad than that for abnormal bleeding in premenopausal women. A series of 1138 women aged 41 to 91 years with postmenopausal bleeding reported the following etiologies and prevalence:
**-Vaginal/endometrial atrophy: 50% **
-Polyps: 12%
-Endometrial cancer: 10%
-Endometrial hyperplasia: 9.8%
-Hormonal effects: 7%
-Cervical cancer: < 1%
-Other (e.g., hydrometra, pyometra, hematometra): 2%
- Family Practice Notebook - Postmenopausal Bleeding
- UpTodate - Post menopausal uterine bleeding
Which one of the following options is the most common presenting symptom in women with atrophic vaginitis?
A. Painless vaginal bleeding.
B. Endometritis.
C. Difficult intercourse.
D. Postcoital bleeding.
E. Scanty purulent discharge.
C. Difficult intercourse
Atrophic vaginitis, also referred to as vulvovaginal atrophy, urogenital atrophy, or vaginal atrophy, is characterized by dryness, inflammation, and thinning of the epithelial lining of the vagina and lower urinary tract due to estrogen deficiency. It typically occurs in menopausal women, but can occur in women of any age who experience a decrease in estrogenic stimulation of urogenital tissue. Premenopausal woman are in hypo-estrogenic state in postpartum period, lactation, and when on anti-estrogenic medication. In such conditions, atrophic vaginitis can also occur.
Symptoms of urogenital atrophy include:
-Vaginal dryness
-Vaginal burning or irritation
-Decreased vaginal lubrication during sexual activity
-Dyspareunia, including vulvar or vaginal pain (at the introitus or within the vagina)
-Vulvar or vaginal bleeding (e.g., postcoital bleeding , fissures)
-Vaginal discharge (leukorrhea or yellow or brown discharge that can be malodorous)
-Pelvic pressure or a vaginal bulge
-Urinary tract symptoms (e.g. urinary frequency, dysuria, urethral discomfort, hematuria)
Symptoms of vaginal atrophy are usually progressive and worsen with time. Vaginal dryness and lack of lubrication on sexual arousal is the earliest and the most common symptom of atrophic vaginitis. This is the first signs of estrogen insufficiency in urogenital system. Urinary symptoms present later as the estrogen deficiency continues.
Option A and B: Painless vaginal bleeding and endometritis are not clinical features associated with atrophic vaginitis.
- AAFP - Diagnosis and Treatment of Atrophic Vaginitis
Which one of the following is the most common site of endometriosis?
A. Round ligament.
B. Uterosacral ligament.
C. Ovary.
D. Bladder.
E. Broad ligament.
C. Ovary
Endometriosis is defined as the presence of endometrial glands and stroma at extrauterine sites. These ectopic endometrial implants are usually located in the pelvis, but can occur nearly anywhere in the body.
The most common sites of endometriosis, in decreasing order of frequency, are:
-Ovaries
-Posterior cul-de-sac
-Broad ligament
-Uterosacral ligament
-Rectosigmoid colon
-Bladder
-Distal ureter
Less common, yet possible, sites are vagina, cervix, rectovaginal septum, caecum, ileum, inguinal canals, abdominal or perineal scars, and umbilicus. Rare sites include breast, pancreas, liver, gallbladder, kidney, urethra, extremities, vertebrae, peripheral nerves, lungs, spleen, diaphragm and central nervous system.
Ovaries are the most common site of endometriosis.
Which one of the following is the least likely site of endometriosis?
A. Ovaries.
B. Cervix.
C. Uterine wall.
D. Pouch of Douglas.
E. Bladder.
B. Cervix
Endometriosis is defined as the presence of endometrial glands and stroma at extrauterine sites. These ectopic endometrial implants are usually located in the pelvis, but can occur nearly anywhere in the body.
The most common sites of endometriosis, in decreasing order of frequency, are:
-Ovaries
-Posterior cul-de-sac (AKA Pouch of Douglas/Rectouterine Pouch)
-Broad ligament
-Uterosacral ligament
-Rectosigmoid colon
-Bladder
-Distal ureter
Less common, yet possible, sites are vagina, cervix, rectovaginal septum, caecum, ileum, inguinal canals, abdominal or perineal scars, and umbilicus. Rare sites include breast, pancreas, liver, gallbladder, kidney, urethra, extremities, vertebrae, peripheral nerves, lungs, spleen, diaphragm and central nervous system.
Ovaries are the most common site of endometriosis.
A 47-year-old woman presents to your clinic with heavy irregular periods every 2 to 4 months for the past 12 months. Before that, her periods were normal, occurring every 32 days. Physical examination is unremarkable. Blood tests show a hemoglobin level of 110 g/L (normal: 115-165). Which one of the following is the most likely cause of this presentation?
A. Endometrial cancer.
B. Endometrial polyps.
C. Endometriosis.
D. Anovulatory cycles.
E. Uterine fibroids.
D. Anovulatory cycles
By far, anovulatory cycles are the most common cause irregular heavy periods. After ovulation, the follicle turns into the progesterone-secreting corpus luteum. The progesterone stabilizes the endometrium. If pregnancy fails to take place, corpus luteum regresses and the progesterone declines. When progesterone falls, endometrial shedding and menstrual flow occurs at the predicted time.
Without ovulation, unopposed estrogen continues to cause endometrial growth. This results in unpredictable shedding of a hypertrophied endometrium and irregular, often heavy vaginal bleeding.
OPTION A : Anovulatory cycles have different etiologies, but polycystic ovary syndrome and approaching menopause are the most common causes. Endometrial cancer in premenopausal woman is not common.
OPTION B, C, and E : Endometrial polyps, endometriosis and uterine fibroids can cause heavy menstrual bleeding (menorrhagia) in a predictable fashion. Menstrual irregularities are not a common feature of these conditions.
- Clinical Care Standards - Heavy Menstrual Bleeding Clinical Care Standard
- AAFP - Evaluation and Management of Abnormal Uterine Bleeding in Premenopausal Women
- UpToDate - Approach to abnormal uterine bleeding in nonpregnant reproductive-age women
A30-year-old woman on oral contraceptive pills (Microgynon 30®) presents to your clinic for a repeat prescription. On examination, you find out that she has a blood pressure of 160/100mmHg, confirmed by a second reading 20 minutes later. She mentions that her husband and she are planning to become pregnant in 6 months. Which one of the following is the best advice for her?
A. She can continue the same OCP but should use methyldopa for hypertension.
B. She should change to an OCP preparation with lower dose of estrogen.
C. She should stop OCP, use condoms for contraception and reassess her blood pressure in 3 months.
D. She should stop the OCP and be started on methyldopa for hypertension.
E. She should stop the OCP and be started on a diuretic or ACE inhibitor for hypertension.
C. She should stop OCP, use condoms for contraception and reassess her blood pressure in 3 months
Eestrogen-containing hormonal contraception methods are NOT absolutely contraindicated in hypertensive women, whether the hypertension has existed before or developed as result of the hormonal contraception use. With hypertension, the patient should be started on appropriate antihypertensive agent if she insists on using the method.
This woman, however, is willing to stop the OCP; hence, cessation of the estrogen-containing method and using an alternative is the best advice. Since this woman is planning to conceive, using condoms for contraception in the next 6 month, during which she does not want to become pregnant, would be the best advice among the options.
If OCPs are ceased, she does not need to be started on antihypertensive drugs at this stage because with cessation of estrogen, there is a good chance that her blood pressure returns to normal. Antihypertensive medications will be indicated if blood pressure remains high despite stopping the OCP.
If use of antihypertensive medications is inevitable, methyldopa should be considered as first line because it is safe to use during pregnancy.
ACE inhibitors and diuretics are contraindicated during pregnancy and should be avoided.
*http://www.aafp.org/afp/2010/0915/p621.html
*http://www.gpnotebook.co.uk/simplepage.cfm?ID=7721
A 51-year-old menopausal woman presents with frequent hot flushes disturbing her sleep and occupational performance. She has the history of breast cancer diagnosed 10 years ago. The tumor was positive for estrogen receptors on pathology. Which one of the following would be the most appropriate management?
A. Cyclical estrogen and progesterone.
B. Progesterone-only HRT.
C. Paroxetine.
D. Black cohoch.
E. Mefenamic acid.
C. Paroxetine
Hot flush is a sudden feeling of warmth that is generally most intense over the face, neck, and chest. The duration is variable but on average lasts about 3-4 minutes. It is often accompanied by sweating that can be profuse and followed by a chill. Hot flushes are seen in perimenopausal women as one of the most common complaints, and is the only indication to start hormone replacement therapy (HRT). HRT is the most effective treatment for severe hot flushes and is a reasonable choice in the absence of contraindications.
OPTION A and B : Hormonal therapy including estrogen and progesterone is avoided in patients with history of breast cancer, although the effect of progesterone on breast cancer is not established.
Women with hot flushes and contraindications to hormonal therapy may benefit from selective serotonin reuptake inhibitor (first-line) such as paroxetine, or other medications such as gabapentin or venlafaxine.
SSRI are widely used in Australia for management of hot flushes when there is contraindication to hormone therapy. It also improves mood symptoms. Paroxetine is the most common prescribed SSRI (and the only one approved by FDA in US). SSRIs are not as effective as HRT, but definitely more effective than placebo. It is important to note that paroxetine, fluoxetine, duloxetine, and bupropion should not be used in patients who are currently on tamoxifen because it interferes with the conversion of tamoxifen to its active metabolite endoxifen.
OPTION D : Black Cohosh acts by unknown mechanism and is used for hot flushes. Many studies have found no significant benefit over placebo.
OPTION E : Mefenamic acid may help for short-term control of menopausal symptoms; however, its long term use is controversial and not recommended.
A 28-year old woman presents with her husband complaining of inability to conceive after 18 months of regular sexual activity. During the discussion on an etiology, further work-up and the treatment, she asks about the possible outcome for treatment of different etiologies. Which one of the following conditions has the most favorable treatment outcome?
A. Azoospermia.
B. Tubal obstruction.
C. Polycystic ovarian syndrome.
D. Pelvic tuberculosis.
E. Turner syndrome.
C. Polycystic ovarian syndrome
Treatment of infertility due to ovulation problems has had the most satisfactory outcomes by far. Polycystic ovarian syndrome (PCOS), also known as Stein-Leventhal syndrome is characterized by hyperandrogenism, small cysts in ovaries, acne and hirsutism as well as problems of ovulation leading to anovulatory cycles due to hormonal imbalance.
70% of females with anovulatory cycles start ovulation on medical management and of those, 50% conceive within 6 to 9 months.
OPTION A : Low or zero sperm counts, poor sperm motility and dysmorphic sperms can all cause infertility. Fertility drugs for promotion of sperm production, artificial insemination with donor sperm and intracytoplasmic injection of sperm can be used as treatment modalities, but success rate is low (less than 25%).
OPTION B and D : Obstruction of or damage to the Fallopian tubes follows pelvic inflammatory disease, prior surgery, or tuberculosis and prevents the sperm from reaching to the ovum. Laparoscopic surgery is used to treat tubal obstructions or adhesions. Success rates of conception vary widely from a minimum of 10% to a maximum of 70% depending on the severity of the blockage.
OPTION E : Turner syndrome is associated with ovarian dysgenesis and infertility. There is no option for treatment of gonadal dysgenesis and reversal of infertility. Success rate is very low even after egg donation due to high rate of miscarriages.
*http://www.ncbi.nlm.nih.gov/pmc/articles/PMC270066 *http://emedicine.medscape.com/article/27
A 28-year-old unmarried woman presents to your clinic with slight left-sided lower abdominal pain for the past 2 days. On examination, she has a blood pressure of 125/90 mmHg, pulse rate of 90 bpm and temperature of 37.3°C. She is otherwise healthy. Abdominal exam elicits no tenderness, rebound or guarding. Ultrasonographic assessment shows a 6 cm solid mass lateral to the uterus on the left side. Which one of the following is the most likely diagnosis?
A. Mucinous cystadenoma.
B. Ovarian teratoma.
C. Endometriosis.
D. Corpus luteum cyst.
E. Ovarian cancer.
B. Ovarian teratoma
Ovarian cystic teratomas and ovarian dermoid cysts are encapsulated tumors with mature tissue or organ component. They are composed of well-differentiated derivations from at least two of the three germ cell layers, namely ectoderm, mesoderm, and endoderm; therefore, they may contain developmentally mature skin components such as hair follicles, sweat glands, pockets of sebum, blood, fat, bone, nail, teeth, eyes, cartilage, thyroid tissue, etc.
Real organoid structures such as teeth, fragments of bone, etc. may be present in approximately 30% of cases.
The diameter of an ovarian teratoma is typically less than 10 cm and very rarely more than 15 cm.
Uncomplicated ovarian dermoids are often asymptomatic and discovered incidentally. There is, however, an increased risk of ovarian torsion that may present with acute pelvic pain.
Ultrasound shows a solid mass in the adnexal (next to the uterus). Teratomas are common in young women around the age of 30 years and the most common pelvic tumors in women younger than 20 years.
OPTION A : Mucinoid cystadenoma, as the name implies, has cystic nature and is not solid. This tumor is at the benign end of the spectrum of mucin-containing epithelial ovarian tumors. On ultrasonography, large cystic adnexal mass that is multilocular with numerous septations is seen.
OPTION C : Ovarian endometriosis has a different appearance on ultrasonography including the acoustic enhancement of ovaries with homogenous low-level internal echoes as a result of the hemorrhagic debris. On the other hand, the predominant feature of endometriosis is dysmenorrhea.
OPTION D : Corpus luteal cyst is a type of functional ovarian cyst that forms when a corpus luteum does not regress, either normally due to pregnancy or without the pregnancy taking place. Corpus luteum cyst is the most common pelvic mass within the first trimester. When corpus luteal cysts are associated with pregnancy, most of them will involute by the end of the second trimester.
On ultrasonography, they might have a range of sonographic appearances depending on the stage of evolution and the age of the associated intracystic hemorrhage, but general characteristics are as follows:
-Diffusely thick wall
-Peripheral vascularity
-Diameter less than 3 cm (with the wall)
OPTION E : Although ovarian cancer is seen as an adnexal solid tumor, it is a remote possibility in a 28-year-old woman.
(PICTURES OF ULTRASOUNDS of TERATOMA, MUCINOID CYSTADENOMA, CORUS LUTEUM CYST in PAGE 934-935)
A 26-year-old woman presents to your clinic on the 6th day of her menstrual cycle with mild lower abdominal pain. She lives with her male partner and has no children. On examination, she has a blood pressure of 110/70 mmHg, pulse rate of 90bpm, and temperature of 37.5°C. No adnexal mass is palpated; however, on vaginal exam cervical motion tenderness is noted. Which one of the following would be the next best step in management?
A. Transvaginal ultrasonography.
B. Cervical swabs for culture.
C. Urine analysis and culture.
D. Thyroid stimulating hormone.
E. Abdominal CT scan.
B. Cervical swabs for culture
Cervical motion tenderness (also called cervical excitation or chandelier sign) suggests a pelvic pathology. Classically, it is present in pelvic inflammatory disease (PID) and ectopic pregnancy (EP). Sometimes the test is used to differentiate PID from appendicitis.
This patient is menstruating and less likely to have an EP; therefore, PID remains the most likely diagnosis. EP is less likely.
Because of the relatively poor specificity and sensitivity of clinical findings in PID, the Center for Disease Control and (CDC) has established minimal criteria for diagnosis of PID.
In a female patient PID is the diagnosis if:
-She is at risk for sexually transmitted disease (STD) AND
-She has lower abdominal or pelvic pain without no other identifiable cause for her illness other than PID, AND
-on pelvic examination there is (1) cervical motion tenderness or (2) uterine tenderness or (3) adnexal tenderness
In such patients, based on diagnosis of PID, empiric antibiotics should be started after cervical swabs are taken for culture and DNA probe.
Since EP is an important differential diagnosis in a patient with cervical motion tenderness, EP should be excluded. This is correct that in the presence of normal menstruation EP is very unlikely, but excluding pregnancy would be a good choice if it was among the options.
Ultrasonography is the next best step in management if ectopic pregnancy is suspected based on history, clinical findings and a positive pregnancy test.
*http://emedicine.medscape.com/article/256448-diffe
*http://emedicine.medscape.com/article/256448-worku
A 31-year-old woman comes to your clinic complaining of unpredictable vaginal bleeding for the past 6 months. She also mentions that recently the amount of bleeding has increased. Which one of the following is the most appropriate next step in management?
A. Full blood exam (FBE).
B. Transvaginal ultrasonography.
C. Serum beta HCG.
D. Thyroid stimulating hormone (TSH).
E. Endometrial sampling.
A. Full blood exam (FBE)
Abnormal uterine bleeding (AUB) affects 9-14% of women. AUB generally can be divided into anovulatory and ovulatory patterns. In ovulatory pattern, the bleeding is regular but heavy, whereas in anovulatory pattern the bleeding is irregular and unpredictable.
Ovulatory pattern is usually caused by uterine problems such as leiomyomas, endometriosis, adenomyosis, polyps, etc. Anovulatory pattern is a result of hormonal problems such as polycystic ovarian syndrome (PCOS), hypothyroidism, hyperthyroidism, hyperprolactinemia, and Cushing syndrome.
The approach to women with ovulatory versus anovulatory pattern is different. It is recommended that if one pattern of bleeding cannot be told from the other, the patient be assessed as having irregular bleeding because this pattern includes investigation for endometrial hyperplasia/cancer for more diagnostic safety.
In approach to abnormal uterine bleeding in reproductive years, the first thing to exclude is pregnancy. It is done using an office urine pregnancy test and not serum beta HCG) (option C)
The next step, after exclusion of pregnancy is checking whether the patient has developed anemia due to blood loss with a full blood exam (FBE). Transvaginal ultrasound (option B) is also a very important investigation in this patient because she has anovulatory pattern of AUB, bringing up endometrial hyperplasia or even cancer as a possibility for which transvaginal ultrasound can be used as a screening tool. Endometrial sampling (option E) using curettage follows if any endometrial abnormalities are found on ultrasound.
A thyroid function test (option D) should only be ordered if there are indicators of thyroid disease. Testing for coagulation disorders such as Von Willebrand disease is recommended if indicated.
- Cancer Australia - Abnormal Vaginal Bleeding in Pre- and Peri-menopausal Women
- AAFP - Evaluation and Management of Abnormal Uterine Bleeding in Premenopausal Women * Approach to diagnosis and management of abnormal uterine bleeding
A 29-year-old woman with past history of pregnancy with an anencephalic fetus presents to your clinic for preconception counselling. Which one of the following is the most appropriate advice regarding folic acid supplementation for her?
A. She should use supplemental folic acid 0.4 to 0.5 mg per day.
B. She should use supplemental folic acid 10 mg per day.
C. She should increase consuming more green vegetables.
D. She should use supplemental folic acid 1 mg per day.
E. She should use supplemental folic acid 4 to 5 mg per day.
E. She should use supplemental folic acid 4 to 5 mg per day
Folic acid deficiency of mother can result in fetal neural tube defects and for this reason every woman should be started on folic acid from at least one month before conception and continue during the first trimester. In addition, all women should be encourage to include green vegetables in their diets, at least during this period.
Current recommendation is daily use of folic acid 0.5 mg daily or a multivitamin containing at least 0.4 mg of folic acid.
Women should take 10 times the minimal dose (4 to 5 mg per day) if:
- There is a family, or personal history of neural tube defects (NTDs) or a previous pregnancy with NTD
- Use of medications that affect folic acid absorption e.g. antiepileptic medications
- Diabetes (type I/II)
- Mother’s BMI>35 kg/m2
According to National Health and Medical Research Council (NHMRC), the father’s personal history of NTDs is another condition requiring prescription if high dose folic acid.
This woman has previous history of a pregnancy associated with NTD; hence, she should be started on folic acid 4 to 5 mg per day from at least one month before conception and 3 months into the pregnancy.
NOTE - the above recommendations apply for women with all current nutritional status. For example, if a woman is vegetarian, just assuming that she is receiving adequate amount of folic acid through green vegetables should not result in altered management of folic acid supplementation for her.
*http://www.kemh.health.wa.gov.au/development/manua
*https://www.nhmrc.gov.au/_files_nhmrc/file/nics/ma
A 27-year-old woman presents for a pre-pregnancy consultation. She is vegetarian. Which one of the following would be the best recommendation with regard to folic acid supplementation?
A. She does not need folic acid supplementation because she is receiving adequate amount through her diet.
B. She does not need folic acid supplementation but she should increase consumption of green vegetables.
C. She should be started on folic acid 0.5 mg per day.
D. She should be started on folic acid 0.1 mg per day.
E. She should be started on folic acid 5 mg per day
C. She should be started on folic acid 0.5 mg per day
Folic acid deficiency of mother can result in fetal neural tube defects and for this reason every woman should be started on folic acid from at least one month before conception and continue during the first trimester. In addition, all women should be encourage to include green vegetables in their diets, at least during this period.
Current recommendation is daily use of folic acid 0.5 mg daily or a multivitamin containing at least 0.4 mg of folic acid.
Women should take 10 times the minimal dose (4 to 5 mg per day) if:
- There is a family, or personal history of neural tube defects (NTDs) or a previous pregnancy with NTD
- Use of medications that affect folic acid absorption e.g. antiepileptic medications
- Diabetes (type I/II)
- Mother’s BMI>35 kg/m2
According to National Health and Medical Research Council (NHMRC), the father’s personal history of NTDs is another condition requiring prescription if high dose folic acid.
The above recommendations apply for women with all current nutritional status, and being a vegetarian does not exclude her. Therefore, she should be started on folic acid supplementation 0.4 to 0.5 mg per day. She should also be advised to include more green vegetables in her diet if she already has not.
*http://www.kemh.health.wa.gov.au/development/manua
*https://www.nhmrc.gov.au/_files_nhmrc/file/nics/ma
A 32-year-old vegetarian woman presents for a pre-pregnancy consultation. She is epileptic and has been on phenytoin for the past 3 years. Which one of the following would be the best recommendation regarding folic acid supplementation?
A. She does not need folic acid supplementation because she is receiving adequate amount through her diet.
B. She does not need folic acid supplementation but she should increase consumption of green vegetables.
C. She should be started on folic acid 0.5 mg per day.
D. She should be started on folic acid 5 mg per day.
E. She should be started on folic acid 0.1 mg per day.
D. She should be started on folic acid 5 mg per day
Folic acid deficiency of mother can result in fetal neural tube defects and for this reason every woman should be started on folic acid from at least one month before conception and continue during the first trimester. In addition, all women should be encourage to include green vegetables in their diets, at least during this period.
Current recommendation is daily use of folic acid 0.5 mg daily or a multivitamin containing at least 0.4 mg of folic acid.
Women should take 10 times the minimal dose (4 to 5 mg per day) if:
- There is a family, or personal history of neural tube defects (NTDs) or a previous pregnancy with NTD
- Use of medications that affect folic acid absorption e.g. antiepileptic medications
- Diabetes (type I/II)
- Mother’s BMI>35 kg/m2
According to National Health and Medical Research Council (NHMRC), the father’s personal history of NTDs is another condition requiring prescription if high dose folic acid.
The above recommendations apply to women with all current nutritional status. Being a vegetarian does not exclude her or change the recommendations. Since she is on antiepileptic drugs that decrease the absorption of folic acid, she should be started on folic acid supplementation 4 to 5 mg per day. She should also be advised to include more green vegetables in her diet if she already has not.
*http://www.kemh.health.wa.gov.au/development/manua
*https://www.nhmrc.gov.au/_files_nhmrc/file/nics/ma
A 41-year-old woman presents with her cervical screening results reporting CIN2. Colposcopic examination is arranged and performed but reveals no gross abnormality. Which one of the following would be the next best step in management?
A. Cone biopsy.
B. LEEP.
C. Repeat pap smear in 12 months.
D. Reassure the patient as it could be a false positive result.
E. Repeat pap smear in 6 months.
A. Cone biopsy
Patients with CIN2 cervical screening result (equivalent to HSIL in modified Bethesda system) should be referred for colposcopy. If the lesion cannot be visualized on colposcopy, performing a cone biopsy would be the next best step in management.
Cervical squamous cell abnormalities in different nomenclatures, i.e. CIN (Cervical Intraepithelial Neoplasia) and Australian Modified Bethesda system are defined and compared in the following table:
(see photo below)
CIN 2 and 3 are equivalents of HSIL.
Women with HSIL on cervical screening should be referred to specialist for colposcopy. If colposcopy is unsatisfactory, the next step to consider is cone biopsy.
Cone biopsy is an excisional technique in which a significant length of tissue along the cervical canal (from 1.5–3 cm) is removed. Traditionally, cone biopsy has been performed using the ‘cold-knife’ technique.
The specific indications for cone biopsy are:
- Failure to visualize the upper limit of the cervical transitional zone (TZ) in a woman with a HSIL on her referral cervical smear i.e. unsatisfactory colposcopy
- Suspicion of an early invasive cancer on cytology, biopsy or colposcopic assessment
- The suspected presence of an additional significant glandular abnormality (i.e. AIS) on cytology or biopsy (i.e., a mixed lesion)
- Australian Government, Department of Health – Cancer screening
- AAFP - Management of Histologic Abnormalities of the Cervix
A 26-year-old woman comes to you with a cervical screening test result, reporting high-grade squamous intraepithelial lesion (HSIL). She is 34 weeks’ pregnant. Which one of the following is the most appropriate management at this stage?
A. Loop excision.
B. Cone biopsy.
C. Continuation of the pregnancy.
D. Offer termination of the pregnancy now.
E. Refer for colposcopy.
E. Refer for colposcopy
Based on recommendations from National Health and Medical Research Council (NHMRC), management of cervical neoplasia in pregnancy is as follows:
LSIL (CIN 1)
Women with low-grade cytologic lesions should be managed in the same way as for non-pregnant women with such lesions, with a repeat smear after 12 months, if they have had a normal cervical screening in the past 2-3 years. Otherwise, they should be referred to a specialist for colposcopy.
HSIL (CIN 2,3)
With a high-grade squamous intraepithelial lesion (HSIL) (CIN 2,3) the woman should be referred for colposcopy.
Colposcopy is safe during pregnancy. The colposcopic evaluation of the cervix may be more difficult due to vaginal laxity preventing complete visualization of the transformation zone. The increased vascularity due to pregnancy may also be difficult to interpret.
Experienced colposcopists will not usually perform a biopsy if they are confident that they have excluded an invasive cancer. If no lesion is identified at colposcopy, it is advisable to request a review of all the cytological slides. If the diagnosis of a HSIL is confirmed, a second opinion from another colposcopist with wide experience in the colposcopic evaluation of pregnant women is recommended.
With HSIL, it will be prudent to review the woman at approximately 20–24 weeks with cytology and colposcopy to determine, as far as possible, that she does not have an invasive lesion.
Definitive treatment of a high-grade lesion, with the exception of invasive cancer, may be deferred safely until after pregnancy.
Invasive carcinoma
A diagnosis of cervical cancer in pregnancy poses major challenges to the patient, her family, and the treating physician(s). The general principles in management of invasive cervical carcinoma are as follows:
- If the fetal lungs are mature at the time of diagnosis, or if the fetus is at/near the gestational age of expected lung maturity, immediate delivery and definitive treatment of the mother is the preferred approach (antenatal corticosteroid therapy is given if indicated).
- If the pregnancy is previable and the patient decides not to continue the pregnancy, immediate therapy of the mother should be initiated.
- In all other patients, decisions regarding timing of treatment and delivery require careful consideration of the stage of disease, the trimester in which the diagnosis is made, and the preferences of the affected woman and her family regarding the pregnancy.
- National cervical screening program
- UpToDate - Cervical cancer in pregnancy
A 31-year-old woman gave birth to a baby of normal weight through vaginal delivery. The delivery was complicated by a small perianal tear that was taken care of without stitching. Today on the fifth day postpartum, she presents with heavy bright red vaginal bleeding. She mentions that the lochia was in scant amounts after delivery compared to her previous pregnancy. On examination, she has a temperature of 38.8°C. The uterus is mildly tender to palpation. Which one of the following could be the most likely diagnosis?
A. Endometritis.
B. Infection of the perianal tear.
C. Retained products of conception.
D. Cervical tear.
E. Uterine rupture.
C. Retained products of conception
The presence of secondary postpartum hemorrhage (between 24 hours and 12 weeks postpartum) that is bright red and fever suggests retained products of conception (RPOC).
After separation of placenta, the basal portion of the decidua remains. This decidua divides into two layers: the superficial layer that is shed and the deep layer regenerates new endometrium, which covers the entire endometrial cavity by the 16th postpartum day. Normal shedding of blood and decidua is referred to as lochia rubra (red, red brown), and lasts for the first few days following delivery. Vaginal discharge then becomes increasingly watery, called lochia serosa (pinkish brown) lasting for 2 to 3 weeks. Ultimately, the discharge becoms yellowish-white, the lochia alba.
Scant amount of lochia in the first few days after delivery suggests that the placental site may have not undergone involution probably due the RPOC. Usually, retained products undergo necrosis with fibrin deposition and may eventually form a so-called placental polyp. As the scar of the polyp detaches from the myometrium, brisk hemorrhage may occur.
The necrotic products may also become infected and cause uterine infection presented by fever, lower abdominal pain and uterine tenderness.
OPTION A : Endometritis can cause fever, offensive-odor lochia and abdominal pain/tenderness. Endometritis is the most common cause of postpartum fever. It occurs within the first 5 days postpartum with the peak incidence between days 2 and 3 postpartum. Although vaginal bleeding can be a feature at occasions, bright red bleeding makes endometritis less likely.
OPTION B : Genital lacerations are another cause of postpartum fever. The peak incidence of wound infection is between days 4 and 5. Although the temporal appearance of the fever favors wound infection as a cause, this diagnosis is less likely because wound infection do not affect the normal course of lochia, and does not have heavy bright red bleeding as an expected feature. Moreover, there are no clues to wound infection such as erythema, tenderness or discharge in history.
OPTION D : Cervical tear can be another cause of bleeding or fever, but cervical tear tends to present with primary rather than secondary postpartum hemorrhage (occurring after the first 24 hours postpartum). An infected overlooked minor cervical laceration can cause fever but is not very likely to cause bright red bleeding. Furthermore, genital tract lacerations do not affect lochia.
OPTION E : Uterine rupture is very unlikely to occur after 24 hours of delivery.
*http://www.uptodate.com/contents/retained-products
*http://www.sahealth.sa.gov.au/wps/wcm/connect/dc33
A 21-year-old woman presents to your clinic asking for emergency contraception. Which one of the following is the most important initial question to ask her before further management?
A. Previous use of OCP.
B. History of headaches.
C. The date of the unprotected sex.
D. The date of the last menstrual period (LMP).
E. Family history of hypercoagulable state.
C. The date of the unprotected sex
Emergency contraception (also known as postcoital contraception, the morning-after pill, or plan B) refers to the use of drugs or a device as an emergency measure to prevent pregnancy in women who have had recent unprotected intercourse, including those who have had a failure of another method of contraception.
Although conception is possible on only a few days of the menstrual cycle, emergency contraception is offered regardless of the timing of the menstrual cycle due to uncertainty in timing of ovulation; therefore, LMP is not a significant determining factor for considering emergency contraception.
Of the options, the most important question to ask is the date of unprotected sex. Most methods are useful within a short time of unprotected sex. The window to use an emergency contraception method is simplified to up to 5 days (120 hours) post-coitus. There is a linear relationship between efficacy and the time from intercourse. The sooner a method is used the lesser the chance of pregnancy would be.
There is no medical contraindication to using pharmacological emergency contraception methods. Contraindications to use of IUCD are the same for routine use of intrauterine devices.
*http://www.uptodate.com/contents/emergency-contrac
*http://kemh.health.wa.gov.au/development/manuals/O
The anomaly of the child, illustrated in the following photograph, is more likely to be due to which one of the following drugs taken by the mother during the pregnancy?
A. Phenytoin.
B. Sodium valproate.
C. Levodopa.
D. Carbamazepine.
E. Lamotrigine.
B. Sodium valproate
Previously, women with epilepsy were informed that their risk for fetal malformations was increased 2- to 3 folds, but more recent data suggest that the risk may not be as great as once thought, particularly for newer agents. More recent studies showed that the risk of fetal malformation following intrauterine exposure to anticonvulsants that are currently in use is approximately the same or only slightly elevated compared to children without exposure.
Sodium valproate is one important exception to these findings. Women treated with sodium valproate are at significantly increased risk of fetal malformations.
The most frequently reported anomalies are:
-Orofacial clefts
-Cardiac anomalies
-Neural tube defects
One study showed that major malformations occurred in 9% of fetuses with first-trimester valproate exposure; however, different data suggest the rate to be as high as 16%.
- PubMed - Teratogenic effects of antiepileptic drugs
- William’s Obstetrics – 2014 - McGraw Hill – pages 246-7
Which one of the following is unlikely to cause fetal growth restriction?
A. Cytomegalovirus infection.
B. Trisomy 13.
C. Thalassemia minor with hemoglobin of 80 g/L.
D. Mother with essential hypertension on treatment with methyldopa.
E. Lupus nephritis.
C. Thalassemia minor with hemoglobin of 80 g/L
Of the options, thalassemia minor has not been shown to be associated with negative pregnancy outcomes such as fetal growth restriction. Although a hemoglobin level of 80 g/L may appear low at the first look, it should be noted that in pregnant women the anemia is defined as a hemoglobin level of less than 100 g/L (not 120 g/L in non-pregnant women). The reason is that plasma volume and hematocrit increase by 50% and 30% respectively, resulting in diluted plasma.
OPTION A : Cytomegalovirus (CMV) is the most common prenatal viral infection. Fetal growth restriction is a known complication of intrauterine infection with CMV.
OPTION B : Trisomy 21 (Down syndrome), trisomy 18 (Edward syndrome), Trisomy 13 (Patau syndrome) and Turner’s syndrome are congenital causes of fetal growth restriction. Trisomy 13 is characterized by profound mental retardation, cyclopia, proboscis, holoprosencephaly and severe orofacial clefts.
OPTION D : Maternal hypertension is another important cause of fetal growth restriction. Studies suggest that treatment of hypertension during pregnancy and consequent drop in maternal mean arterial pressure is associated with a lower birth weight. In fact, it is postulated that mild to moderate hypertension of the mother should be left untreated to prevent treatment-induced fetal growth restriction.
OPTION E : Hypertension can be seen in 10-20% of pregnant women with systemic lupus erythematosus, especially if the kidneys are involved (lupus nephritis). Maternal hypertension can result in fetal growth restriction.
TOPIC REVIEW
Causes of intrauterine growth restriction:
Medical
-Chronic hypertension (!)
-Preeclampsia
-Diabetes mellitus
-Systemic lupus erythematous
-Chronic renal disease
-Inflammatory bowel disease
-Severe hypoxic lung disease
Maternal
-Smoking
-Alcohol use
-Cocaine use
-Warfarin
-Phenytoin
-Malnutrition
-Prior history of pregnancy with intrauterine growth restriction
-Residing at altitude above 5000 feet
Infections
-Syphilis
-CMV
-Toxoplasmosis
-Rubella
-Hepatitis B
-Herpes simplex 1,2
-HIV
Congenital
-Trisomy 21
-Trisomy 18
-Trisomy 13
-Turner’s syndrome
- RACGP - Intrauterine growth restriction: Diagnosis and management
- IJRM - The pregnancy outcome in patients with minor β-thalassemia
- CMR - Lupus and Pregnancy: Complex Yet Manageable
A 58-year-old woman presents to your practice with complaints of hot flushes and mood swings for the past 12 months. Her menstrual periods ceased 2 years ago. She has the history of breast cancer at the age of 35 that was treated successfully. Which one of the following is be the most appropriate management for her symptoms?
A. Clonidine.
B. Fluoxetine.
C. Hormone replacement therapy.
D. Estrogen patch.
E. No treatment at this age.
B. Fluoxetine
Menaopausal hormone replacement therapy (HRT) is the most effective treatment for vasomotor symptoms (e.g. hot flushes) of menopause. Menopause is associated with several complications, but vasomotor symptoms are the only indication to start HRT in a menopausal woman. However, personal history of a estrogen-dependent cancer makes HRT of any form an unfavorable treatment option. For those in whom HRT is not an appropriate option either due to a medical condition or the patient’s preferences, the selective serotonin reuptake inhibitors (SSRIs) and selective serotonin norepinephrine reuptake inhibitors (SNRIs) are first choices for treatment of vasomotor symptoms.
The following are recommendations by Royal Australian and New Zealand College of Obstetrics and Gynaecology (RANZCOG) regarding menopausal hormone therapy (MHT):
Recommendation1 - The primary indication for the use of MHT is the alleviation of distressing menopausal vasomotor symptoms.
Recommendation 2 - In women with primary ovarian insufficiency, MHT should be continued until the normal age of the menopause.
Recommendation 3 - MHT is also effective and appropriate for the prevention of osteoporosis related fracture in at risk women within 10 years of the menopause.
Recommendation 4 - The risk of VTE and stroke increases with oral MHT but the absolute risk is very small before age 60 years.
Recommendation 5 - In women within 10 years of the menopause MHT does not increase the risk of coronary heart disease.
Recommendation 6 - Combined MHT use for more than 5 years may be associated with an increased risk of breast cancer. This risk appears to be related to the use of a progestogen and duration of therapy.
Recommendation 7 - Estrogen-only MHT does not increase risk of breast cancer.
Recommendation 8 - Current safety data do not support the use of MHT in breast cancer survivors.
Recommendation 9 - Estrogen-only therapy is appropriate for women who have undergone hysterectomy.
Recommendation 10 - Eestrogen plus progestogen should be used in women with an intact uterus.
Recommendation 11 - The dose and duration of therapy should be consistent with treatment goals.
*http://www.uptodate.com/contents/menopausal-hot-fl
*https://www.ranzcog.edu.au/doc/hormone-replacement
A 32-year-old woman is referred to you at 36 weeks gestation for management of eclampsia. While examining, the patient develops a seizure in front of you. Which one of the following is the most appropriate immediate management?
A. Phenytoin.
B. Intravenous diazepam.
C. Ensuring airway patency.
D. Magnesium sulphate.
E. Induction of labor.
C. Ensuring airway patency
In approaching a patient, especially in an emergency setting such as this one, the most immediate management is checking and stabilizing airway, breathing and circulation (ABC), regardless of the underlying etiology.
Ensuring and maintaining airway patency should always be the first priority, followed by sufficient breathing and oxygenation, and circulation.
Magnesium sulphate is the anticonvulsant of choice for eclampsia. Intravenous diazepam can be given while magnesium sulphate is prepared and the seizure is prolonged.
Arrangements for delivery should be decided once the woman’s condition is stable. In the meantime, close fetal monitoring should be maintained.
- http://www.ranzcog.edu.au/doc/somanz-hypertensive-
*http://www.kemh.health.wa.gov.au/development/manua
A 29-year-old woman presents with two episodes of bleeding, one hour apart at 39 weeks gestation. She has no abdominal pain or any other complaints. Physical exam is inconclusive. An ultrasound scan reveals placenta previa. Which one of the following is the most appropriate next step in management?
A. Admit her for supervision.
B. Discharge her home.
C. Induction of labor.
D. Cesarean section.
E. Vaginal examination.
D. Cesarean section
Most women who initially present with symptomatic placenta previa respond to supportive therapy, and do not require immediate delivery. Even a large bleed does not preclude conservative management. Management of placenta previa when mother and fetus are stable is conservative until 38 weeks pregnancy when delivery should be performed by cesarean section. Since this woman is already at 39 weeks gestation, cesarean section without further need to document fetal lung maturity would be the most appropriate option.
Admission for supervision (option A) was the appropriate option at earlier gestational ages (< 37 weeks) where expectant management (if mother and fetus were stable) was considered until adequate fetal maturity is ensured.
Vaginal delivery is contraindicated in placenta previa most of the time, so induction of labor (option C) is inappropriate. Digital vaginal examination (option E) should be avoided in patients with placenta previa.
*http://www.sahealth.sa.gov.au/wps/wcm/connect/b1c6
* https://www.ranzcog.edu.au/doc/rcog-placenta-praev
A 37-year-old woman presents to your practice for contraception advice. She has otosclerosis and uses hearing aids. She had 3 pregnancies, during which her hearing deteriorated rapidly. Which one of the following is the most appropriate method to recommend?
A. Low-dose combined oral contraceptive pills.
B. High-dose combined oral contraceptive pills.
C. Progestin-only pills.
D. Condoms.
E. Intrauterine contraceptive device (IUCD).
E. Intrauterine contraceptive device (IUCD)
Otosclerosis is an often inherited condition, which is the most common cause of progressive deafness in young adults. It involves hardening of the bones of the middle ear resulting in progressive hearing loss that is often irreversible. In most cases of otosclerosis, the smallest bone (stapes) is involved. The bone becomes calcified or hardened by overgrowth of new bone. Treatment for otosclerosis is by a surgical procedure called stapedectomy, duirng which the top portion of the stapes is removed and a small prosthesis is inserted instead.
Although not supported by solid evidence, there have been suggestions that the combined oral contraceptives (COCs) can worsen otosclerosis, especially if the patient has experienced onset or deterioration during pregnancy or steroid use. For this reason and depsite lack of evidence, avoiding systemic hormonal methods may be appropriate. Of the given non-hormonal methods, IUCD provides more contraception benefit and less failure rates and can be advised as an acceptable method for her.
*https://www.ncbi.nlm.nih.gov/pubmed/19121641
A 27-year-old woman presents with severe nausea and vomiting at 8 weeks pregnancy. She cannot take solid foods but is able to drink small sips of liquids. She is concerned that she might have gastroenteritis, because her partner has recently had it. Which one of the following is the next best investigation in this situation?
A. Pelvic ultrasound.
B. Urinalysis and culture.
C. Stool exam and culture.
D. Quantitative β-HCG.
E. Abdominal ultrasound.
B. Urinalysis and culture
Although this woman might have gastroenteritis, the first thing to consider, given the gestational age, is hyperemesis gravidarum/nausea and vomiting of pregnancy. The mean onset of hyperemesis gravidarum/nausea and vomiting of pregnancy symptoms is at 5 to 6 weeks of gestation, peaking at about 9 weeks, and usually abating by 16 to 20 weeks of gestation; however, symptoms may continue until the third trimester in 15-20% of pregnant women and until delivery in 5%. Sixty percent of women are asymptomatic six weeks after onset of nausea. If vomiting persists beyond a few days postpartum, other etiologies should be investigated.
Diagnosis of hyperemesis gravidarum is mainly clinical; however, investigations are needed both for assessment of the patient’s status and excluding other possible causes of nausea and vomiting.
The standard initial evaluation of pregnant women with persistent nausea and vomiting includes measurement of weight, vital signs, orthostatic blood pressure, serum electrolytes, and urine ketones and specific gravity and urine microscopy and culture.
Attending to hydration status and electrolyte imbalance is of paramount importance before proceeding to investigations for and underlying cause. A spot urine dipstick at the office/ED or a formal urinalysis, microscopy, and culture is as well as electrolyte assays is the first step while arrangements for other investigations are made. An obstetric ultrasound exam (pelvic ultrasound) should be performed to look for gestational trophoblastic disease or multiple gestation as possible underlying causes. At this gestational age, transvaginal ultrasound is the preferred method. Pelvic ultrasound allows quick visualization of the female pelvic organs and structures including the uterus, cervix, vagina, fallopian tubes, ovaries, and in pregnant women, the fetus and the gestational sac.
OPTION C : Stool exam and culture may be considered for evaluation of the selected patients if gastroenteritis is suspected as the cause of the clinical picture.
OPTION D : With conditions such as multiple or molar pregnancy, quantitative serum beta HCG is higher than normal pregnancies. However, the most common cause for a β-HCG level unproportionate to gestational age is dating errors. If such discrepancy is found on quantitative β-HCG test, ultrasound will be required for accurate estimation of gestational date; therefore, this test is neither necessary nor useful as initial investigation.
OPTION E : Abdominal ultrasound is inferior to transvaginal ultrasound for evaluation of female reproductive organs and the fetus.
- South Australian Perinatal Practice Guideline Nausea and Vomiting in Pregnancy and Hyperemesis Gravidaru
- UpToDate - Nausea and vomiting of pregnancy: Clinical findings and evaluation
A 24-year-old woman, who underwent an episiotomy during labor four days ago, has presented with severe vaginal pain. On examination, an 8-cm hematoma is noted at the site of the episiotomy. The woman is hemodynamically stable. Which one of the following is the most appropriate next step in management?
A. Aspirate the mass.
B. Explore the hematoma.
C. Reassure the patient as this will resolve spontaneously.
D. Antibiotics.
E. Oral analgesics and review in one week.
B. Explore the hematoma
Most puerperal hematomas arise from bleeding lacerations related to operative deliveries or episiotomy, but it is also possible for a hematoma to result from spontaneous injury to a blood vessel in the absence a of laceration/incision of the surrounding tissue.
The most common locations for puerperal hematomas are vulva, vagina/paravaginal area, and retroperitoneum.
The diagnosis of most puerperal hematomas is based upon the presence of characteristic symptoms and findings on physical examination including:
- Vulvar hematomas - usually present with rapid development of a severely painful, tense, compressible mass covered by skin with purplish discoloration. A vulvar hematoma may be an extension of a vaginal hematoma that has dissected through loose subcutaneous tissue into the vulva.
- Vaginal hematomas - often present with rectal pressure; however, hemodynamic instability due to bleeding into the ischiorectal fossa and paravaginal space may be the first sign of a vaginal hematoma, and can result in hypovolemic shock. On physical examination, a large mass protruding into the vagina is often seen.
- Retroperitoneal hematomas - they extend between the folds of the broad ligament and may be asymptomatic initially. Due to the significant amount of blood that can accumulate in the retroperitoneal space, these patients often present with tachycardia, hypotension, or even shock. Patients with a retroperitoneal hematoma usually do not present with pain unless the hematoma is associated with trauma. Palpation of an abdominal mass or fever can also be signs of a retroperitoneal hematoma.
Treatment of a hematoma depends on the size and location:
Non-expanding hematomas that are <3cm can be managed conservatively with analgesics and ice packs.
Management of an expanding hematoma or one greater than 3cm is with surgical exploration under anesthesia. An incision is made and the hematoma is evacuated. The surgical site should not be sutured. Vagina is often packed for 12-24 hours. An indwelling urinary catheter may be indicated.
This patient has a large haematoma (>3cm) that needs surgical excision and evacuation.
Aspiration of the hematoma is not an appropriate treatment. If surgical intervention is indicated excision and evacuation is the preferred option, followed by vaginal packing for 12- 24 hours.
*https://www.sahealth.sa.gov.au/wps/wcm/connect/149
A 28-year-old primigravida woman presents to the maternity unit at 38 weeks gestation after spontaneous rupture of membrane. On vaginal examination, the cervix is 5 cm dilated and the membranes are ruptured. Meconium-stained liquor is noted in the vagina. A cardiotocography (CTG) unveils a fetal heart rate of 130 bpm, variability of 5 bpm and variable decelerations. Which one of the following would be the most appropriate next step in management?
A. Urgent cesarean section.
B. Fetal Scalp blood sampling.
C. Continue CTG monitoring.
D. Start infusion of oxytocin to facilitate the labor.
E. High forceps delivery.
B. Fetal Scalp blood sampling
Meconium can be found in the gastrointestinal tract of the fetus from as early as 10 -16 weeks gestation. Although 75% of meconium is water, the remaining 25% consists of gastric secretions, bile salts, mucous, vernix, lanugo, blood, pancreatic enzymes, free fatty acids and squamous cells. Meconium staining of the amniotic fluid (MSAF) occurs in approximately 10-15% of labors. MSAF rarely occurs before 30 weeks gestation. In a preterm fetus, MSAF can suggest possible infection or hypoxia. Neonatal aspiration of MSAF can become a serious problem if pulmonary hypertension and severe hypoxemia develop. Such conditions require expert neonatal intensive care.
Meconium stained liquor is one of the conditions considered as intrapartum risk factors for the fetus’ wellbeing. With meconium present, continuous cardiotocography (CTG) should follow for ongoing fetal assessment. Further Actions depend on CTG results:
Reassuring CTG: a CTG is reassuring if the following criteria are met:
a. Baseline fetal heart rate (FHR) is between 110-160 bpm
b. Variability of FHR is between 5-25 bpm
c. Decelerations are absent or early
The significance of the presence or absence of accelerations is unclear and are exclude for interpretation.
With a reassuring CTG, watchful waiting for labor to progress is recommended, unless there are other contraindications. Continuous CTG and close monitoring of the mother and fetus is essential.
Non-reassuring CTG: a CTG is non-reassuring if any of the following is present:
a. Baseline FHR is between 100-109 bpm or between 161-170 bpm
b. Variability of FHR is reduced (3-5 bpm for >40 minutes)
c. Decelerations are variable without complicating features
Abnormal CTG: a CTG is abnormal if at least two of the features described in non-reassuring CTG are present OR one or more of the following features exists:
a. Baseline FHR is <100 bpm or >170bpm
b. Variability is absent or <3 bpm
c. Decelerations are prolonged for >3 minutes OR late OR have complicated variables
With a non-reassuring or abnormal CTG, fetal blood sampling should follow unless contraindicated.
Contraindications to fetal blood sampling include the following:
-Clear evidence of serious fetal compromise (e.g., complete absence of reassuring features) – in such conditions urgent preparation to expedite birth should be made.
-Prolonged deceleration
-Mobile presenting part
-Unknown presentation
-Face presentation
-Undilated cervix (< 3 cm)
-Active second stage of labor
-Fetal hereditary bleeding disorders (e.g., suspected fetal thrombocytopenia, hemophilia)
-Maternal infection (e.g., HIV, hepatitis, herpes simplex virus, suspected intrauterine sepsis)
-Prematurity (gestation < 34 weeks) as delayed birth due to the procedure may be associated with an increase in adverse outcomes because the small “at risk” fetus will have a lower threshold than a term infant for fetal compromise.
This fetus has a non-reassuring CTG and no contraindications to fetal scalp blood sampling; therefore, this procedure is the most appropriate next step in management.
Proceeding to delivery is indicated if fetal blood sampling shows genuine fetal compromise.
- WA Health Department – Obstetrics and Gynaecology Guidelines
- RANZCOG Intrapartum Fetal Surveillance Clinical Guideline
A 31-year-old woman is admitted to the Maternity Unit for labor. She has had regular antenatal visits and her pregnancy has been uneventful thus far. During the labor, meconium liquor passage is noted. Cardiotocography shows a fetal heart rate (FHR) of 149 bpm. There are no decelerations or accelerations. There is beat-to-beat variability of 15 bpm. Vaginal exam is normal. Which one of the following is the most appropriate next step in management?
A. Fetal scalp blood sampling as there is a 10% chance of hypoxia.
B. Fetal scalp blood sampling as there is a 50% chance of hypoxia.
C. Fetal scalp blood sampling as there is a 75% chance of hypoxia.
D. Proceeding to immediate cesarean section.
E. The CTG is normal and close monitoring until delivery is all required for now.
E. The CTG is normal and close monitoring until delivery is all required for now
Meconium can be found in the gastrointestinal tract of the fetus from as early as 10 -16 weeks gestation. Although 75% of meconium is water, the remaining 25% consists of gastric secretions, bile salts, mucous, vernix, lanugo, blood, pancreatic enzymes, free fatty acids and squamous cells. Meconium staining of the amniotic fluid (MSAF) occurs in approximately 10-15% of labors. MSAF rarely occurs before 30 weeks gestation. In a preterm fetus, MSAF can suggest possible infection or hypoxia. Neonatal aspiration of MSAF can become a serious problem if pulmonary hypertension and severe hypoxemia develop. Such conditions require expert neonatal intensive care.
Meconium stained liquor is one of the conditions considered as intrapartum risk factors for the fetus’ wellbeing. With meconium present, continuous cardiotocography (CTG) should follow for ongoing fetal assessment. Further Actions depend on CTG results:
Reassuring CTG: a CTG is reassuring if the following criteria are met:
a. Baseline fetal heart rate (FHR) is between 110-160 bpm
b. Variability of FHR is between 5-25 bpm
c. Decelerations are absent or early
The significance of the presence or absence of accelerations is unclear and are exclude for interpretation.
With a reassuring CTG, watchful waiting for labor to progress is recommended, unless there are other contraindications. Continuous CTG and close monitoring of the mother and fetus is essential.
Non- reassuring CTG: a CTG is non-reassuring if any of the following is present:
a. Baseline FHR is between 100-109 bpm or between 161-170 bpm
b. Variability of FHR is reduced (3-5 bpm for >40 minutes)
c. Decelerations are variable without complicating features
Abnormal CTG: a CTG is abnormal if at least two of the features described in non-reassuring CTG are present OR one or more of the following features exists:
a. Baseline FHR is <100 bpm or >170bpm
b. Variability is absent or <3 bpm
c. Decelerations are prolonged for >3 minutes OR late OR have complicated variables
With a non-reassuring or abnormal CTG, fetal blood sampling should follow unless contraindicated.
Contraindications to fetal blood sampling include the following:
-Clear evidence of serious fetal compromise (e.g., complete absence of reassuring features) – in such conditions urgent preparation to expedite birth should be made.
-Prolonged deceleration
-Mobile presenting part
-Unknown presentation
-Face presentation
-Undilated cervix (<3 cm)
-Active second stage of labor
-Fetal hereditary bleeding disorders (e.g., suspected fetal thrombocytopenia, hemophilia)
-Maternal infection (e.g., HIV, hepatitis, herpes simplex virus, suspected intrauterine sepsis)
-Prematurity (gestation < 34 weeks) as delayed birth due to the procedure may be associated with an increase in adverse outcomes because the small “at risk” fetus will have a lower threshold than a term infant for fetal compromise.
A 25-year-old woman presents to the emergency department at 14 weeks pregnancy with complaint of urine retention for the past 18 hours. She had an obstetric ultrasound at 10 weeks pregnancy confirming the date. In the emergency department, a transvaginal ultrasound is performed confirming the gestational age and revealing a retroverted uterus. On abdominal examination, there is a tender mass midway below the umbilicus. Which one of the following is the most likely diagnosis?
A. Ectopic pregnancy.
B. Ovarian fibroid.
C. Red degeneration of uterine fibroid.
D. Ovarian cyst.
E. Incarcerated uterus.
E. Incarcerated uterus
The presentation is classic for incarcerated gravid uterus.
The term incarcerated gravid uterus refers to a pregnant uterus entrapped in the pelvis by the sub-promontory sacrum.
In very early pregnancy, the uterus is retroverted (retroflexed) in up to 20% of women. As the uterus enlarges during the first trimester, the fundus normally rises from the hollow of the sacrum to an anterior ventral position and corrects the retroversion. However, in rare cases, the fundus becomes wedged below the sacral promontory, where it continues to enlarge. Concomitantly, the cervix becomes displaced upwards against or above the symphysis pubis and pushes against the urethra and bladder, interfering with normal voiding.
Eventually, the posterior pelvis becomes too small to accommodate the increasing size of the fundus; this typically occurs when the pregnancy progresses beyond 20 weeks of gestation. At this point, the anterior lower uterine wall begins to thin and balloon into the upper abdomen to form a sacculation. Concomitantly, both the bladder and the cervix are pulled up into the abdominal cavity, towards the umbilicus. The cervix can stretch to 10 cm or more in length, such that the internal os becomes located above the symphysis pubis, and occasionally above the bladder.
History and symptoms:
Typically, the patient presents at 14 to 16 weeks of pregnancy with symptoms related to pressure on the anatomic structures adjacent to the entrapped enlarging uterus. The most common symptoms are pain and progressive difficulty voiding. The pain could be abdominal, suprapubic, or felt in the back; or may be just a discomfort or feeling of fullness in the pelvis.
Urinary symptoms include dysuria, frequency, sensation of incomplete emptying, stress incontinence, and urinary retention (most common). Pressure on rectum can cause rectal pressure, tenesmus and worsening constipation. Vaginal bleeding can infrequently occur.
Symptoms can be intermittent, resolving for a period of time and then returning weeks later.
Exam finding:
1. Severe anterior displacement of the cervix behind the pubic symphysis, making the physician unable to visualize the cervix with a speculum or palpate the external os on pelvic examination
2. The vagina is angulated anteriorly and a large, soft, smooth, nontender mass (the incarcerated uterus) fills the cul-de-sac
3. Initially, the uterus may be difficult to palpate abdominally, and later in pregnancy the fundal height may lag behind that expected for gestational age. The abdomen may appear full or distended due to an overdistended bladder.
Ultrasonography is the most appropriate initial investigation when the condition is suspected.
*http://www.uptodate.com/contents/incarcerated-grav
Which one of the following drugs has the highest rate of congenital malformations if used in pregnancy?
A. Carbamazepine.
B. Phenytoin.
C. Sodium valproate.
D. Levetiracetam.
E. Lamotrigine.
C. Sodium valproate
Sodium valproate carries the highest teratogenicity rate among all antiepileptic drugs. The potential congenital defects caused by sodium valproate are shown in the following table: (see photo)
- Neural tube defects - Spina bifida, anencephaly
- Heart defects - Congenital ventricular septal defect, aortic stenosis, patent ductus arteriosus, aberrant pulmonary artery
- Limb defects - Radial ray defect, polydactyly (more than 5 fingers), oligodactyly (less than 5 fingers), absent fingers,overlapping toes, camptodactyly (fixed flexion deformity of one or more proximal interphalangeal joints), split
hand, ulnar or tibial hypoplasia, - Genitourinary defects - Hypospadias, renal hypoplasia, hydronephrosis, duplication of calyceal system
- Brain anomalies - Hydranencephal (1), porencephaly(2), arachnoid cysts, cerebral atrophy, partial agenesis of corpus callosum,agenesis of septum pellucidum, lissencephaly (3) of medial sides of occipital lobes, Dandy-Walker anomaly (4)
- Eye anomalies - Bilateral congenital cataract, optic nerve hypoplasia, tear duct anomalies, microphthalmia, bilateral iris defects, corneal opacities
- Respiratory tract defects - Tracheomalacia , lung hypoplasia, severe laryngeal hypoplasia, abnormal lobulation of the right lung, right oligemic (less blood flow) lung
- Abdominal wall defects - Omphalocele
- Skin abnormalities - Capillary hemangioma, aplasia cutis congenital of the scalp.
All other medications mentioned carry the risk of congenital malformations at a lower rate compared to valproic acid.
(1) Also called hydrancephaly is absence of cerebral hemispheres and replacement of the vacant parts of the cranium with cerebrespinal fluid.
(2) A congenital disorder characterised by presence of cysts or cavities within the cerebral hemisphere.
(3) Literally meaning ‘smooth brain’ is caused by defective central nervous migration during 12th and 24th weeks of gestation resulting in lack in development of the brain folds (gyro) and grooves (sulci)
(4) Dandy-Walker syndrome is a congenital malformation of the cerebellum with the key feature being complete absence of the cerebellar vermis.
*http://www.medscape.com/viewarticle/724671
*http://www.australianprescriber.com/magazine/31/3/
*http://www.gpnotebook.co.uk/simplepage.cfm?ID=6511
Sarah, 18 years old, is in your office for a visit because she is concerned about her delayed puberty and absence of menses. She has never had a period before. On examination, she has no pubic or axillary hair, and her breasts are at Tanner I stage. Which one of the following is the most appropriate next step in management?
A. Pelvic ultrasonography.
B. Serum follicular stimulating hormone (FSH) and luteinizing hormone (LH).
C. Bone age assay.
D. Serum thyroid stimulating hormone (TSH).
E. Karyotype.
B. Serum follicular stimulating hormone (FSH) and luteinizing hormone (LH)
This scenario represents a case a primary amenorrhea as wells as delayed puberty evident by complete lack of breast development and axillary and pubic hair.
The most important step in assessment of primary amenorrhea is always a thorough history and complete physical exam to check if secondary sexual characteristics are present. Breast development reflects estrogen exposure and is the best indicator that puberty has begun. In fact, the diagnostic approach to primary amenorrhea starts with breast assessment. Breasts at Tanner III stage or higher are equivalent to presence of secondary sexual characteristics (Tanner III or higher in females older than 13 years). With developing or developed breasts, the next thing to check is whether a normal uterus is present. This is achieved by physical exam (if possible) and a pelvic ultrasound.
With Tanner I breasts, secondary sexual characteristics are absent. In such cases, investigations start with measurement of FSH either alone or with LH for assessment of hypothalamus – pituitary – ovary (HPO) axis.
This patient has not started breast development; therefore, the next step in management is measuring the FSH (and LH) for evaluation of HPO axis. TSH and prolactin are also routinely assessed as a part of hormonal assay in delayed puberty. Exclusion of pregnancy should always be considered as well.
See photo below
OPTION A : Pelvic ultrasonography is considered first choice of investigation in females with primary amenorrhea and developed secondary sexual characteristics.
OPTION C : Bone age assay is used to establish a diagnosis of constitutional growth delay (CGD). CGD is a diagnosis of exclusion, meaning that all possible causes of delayed puberty should be excluded by thorough history, physical examination, and investigation. Some conditions to exclude are hormonal deficiencies, occult systemic illness, or syndromes associated with impaired growth and development. After exclusion of all possible causes of CGD, radiographic study of the left hand and wrist to assess skeletal maturation is critical in diagnosing constitutional growth delay. Typically, in children with CGD, the bone age begins to lag behind chronologic age during early childhood and is delayed in adolescence by an average of 2-4 years.
OPTION E : Karyotyping is considered in patients with syndromic features such as short stature and amenorrhea or where hypergonadotropic hypogonadism (e.g., Turner syndrome) is suspected in the presence of an elevated FSH above the prepubertal reference range.
TOPIC REVIEW
Primary amenorrhea is defined as the absence of menses at age 15-16 in the presence of normal growth and secondary sexual characteristics (with a breast of at least Tanner II) or 13-14 in the absence of secondary sexual characteristics.
Primary amenorrhea is usually the result of a genetic or anatomical abnormality. In a large case series of primary amenorrhea, the most common etiologies were:
*Gonadal dysgenesis including turner syndrome - 43%
*Mullerian dysgenesis (absence of vagina, sometimes with absence of uterus) – 15%
*Physiological delay of puberty (constitutional delay of puberty, chronic systemic disease, acute illness) – 14% *Polycystic ovarian syndrome (PCOS) – 7%
*Isolated gonadotropin-releasing hormone (GnRH) deficiency – 5%
*Transverse vaginal septum – 3%
*Weight loss / anorexia nervosa / exercise induced – 2%
*Hypopituitarism – 2%
*Rare causes - < 1% (imperforated hymen, complete androgen insensitivity syndrome, hyperprolactinemia/prolactinoma), other pituitary tumors, congenital adrenal hyperplasia, hypothyroidism, central nervous system defects, craniopharyngioma, and Cushing’s disease)
Investigation:
According to latest guidelines, indications for investigating a cause of primary amenorrhea include:
-An adolescent who has not had menarche by age 15-16 years
-Adolescent who has not had menarche and more than 3 years have elapsed since thelarche
-An adolescent who has not had a menarche by age 13-14 years and no secondary sexual development
-An adolescent who has not had menarche by age 14 years and:
(1)There is a suspicion of an eating disorder or excessive exercise, or
(2) There are signs of hirsutism, or
(3) there is suspicion of genital outflow obstruction
The following tests are considered basic for all patients with primary amenorrhea:
-Pregnancy test (to exclude pregnancy prior to first menstruation)
-Pelvic ultrasound (complementary to physical exam)
-FSH (and LH)
-Thyroid stimulating hormone (TSH)
-Prolactin
For the exam purposes though, one should know how to prioritize these tests.
In assessment of the female patients with primary amenorrhea two critical questions should be answer:
- Are secondary sexual characteristics present or absent?
A physical examination will evaluate secondary sexual characteristics such as breast development, axillary and pubic hair, and growth. Breasts are an endogenous assay for estrogen. Breast development consistent with a Tanner stage of II or greater indicates the presence of estrogen and ovarian function (although it could be insufficient or prematurely failed). With breasts present, the next step is always checking for the presence or absence of a uterus, its anatomy and possible defects, vagina, and hymen. If there is no breast development, FHS (and LH) comes first.
- Is a uterus absent or present?
Ultrasonography is the best method to reach an answer for this question. Both transvaginal (if the patient not virgin) and transabdominal ultrasonography should be performed to assess whether mullerian structures are absent or present.
EXAM TIP
The following are characteristic features of some important conditions associated with primary amenorrhea:
Gonadal dysgenesis (including Turner)
1. No or abnormally developed breasts
2. Present uterus (but streak ovaries)
3. Often normally-developed axillary and pubic hair
4. Increased serum FSH
Androgen insensitivity
1. Normally-developed breasts (Tanner II or higher)
2. Absent uterus
3. Absent axillary and pubic hair
Hypothalamic-pituitary failure
1. No or abnormally developed breasts (Tanner stage inconsistent with predicted age)
2. Present uterus
3. Decreased serum FSH
Mullerian dysgenesis
1. Normal breast development
2. Normal axillary and pubic hair
3. Normal hormonal assay (including FSH, LH, prolactin, TSH)
4. Absent uterus (and upper part of vagina)
Transverse vaginal septum/imperforated hymen
1. Normal breast development (Tanner II or higher)
2. Normal axillary and pubic hair
3. Normal hormonal assay (including FHS, LH, prolactin, TSH)
4. Cyclical pelvic pain and/or suprapubic mass
- UpToDate - Evaluation and management of primary amenorrhea
- American Family Physician - Disorders of Puberty: An Approach to Diagnosis and Management * MSD Manual - Amenorrhea
A 16-year-old girl is brought to your clinic by her mother for evaluation because she has not started menstruation yet. She is a dedicated ballet dancer. On examination, she has a body mass index (BMI) of 16. Of all secondary sexual characteristics, only breast buds consistent with a Tanner stage II are present. You order serum FSH, LH, TSH and prolactin levels all of which are in normal range. Which one of the following is the most appropriate next step in management?
A. Pelvic ultrasonography.
B. Refer to an adolescent health center.
C. Reassure that it could be a normal variant of puberty.
D. Perform a Pap smear.
E. Karyotyping.
A. Pelvic ultrasonography
The case scenario describes a girl with primary amenorrhea. Primary amenorrhea is defined as the absence of menses by the age of 15-16 years in the presence of normal growth and secondary sexual characteristics or by the age of 3-14 years in the absence of secondary sexual characteristics. According to this definition, this girl has primary amenorrhea and should be thoroughly investigated.
Etiologies of primary amenorrhea diverse but generally can be considered as anatomical or hormonal.
In a large case series of primary amenorrhea, the most common etiologies were:
*Gonadal dysgenesis including turner syndrome - 43%
*Mullerian dysgenesis (absence of vagina, sometimes with absence of uterus) – 15%
*Physiological delay of puberty (constitutional delay of puberty, chronic systemic disease, acute illness) – 14%
*Polycystic ovarian syndrome (PCOS) – 7%
*Isolated gonadotropin-releasing hormone (GnRH) deficiency – 5%
*Transverse vaginal septum – 3%
*Weight loss / anorexia nervosa / exercise induced – 2%
*Hypopituitarism – 2%
*Rare causes - < 1% (imperforated hymen, complete androgen insensitivity syndrome, hyperprolactinaemia/prolactinoma), other pituitary tumours, congenital adrenal hyperplasia, hypothyroidism, central nervous system defects, craniopharyngioma, and Cushing’s disease)
There are a few points in the history that can narrow down the diagnosis. Firstly, this girl has Tanner II breast development indicating that she has been exposed to estrogen; however, at 16 years age, more developed breasts with higher Tanner stages would be expected.
Puberty in girls starts with breast development. Growth spurt then occurs and is followed by axillary and pubic hair development. Menstruation often does not take place until two years from the onset of puberty.
Another point to consider is the fact that this girl has no pubic or axillary hair. This finding can be either due to arrested or delayed progression of puberty which once has been started (as evident by breast buds), or incomplete androgen insensitivity in which breast development occurs but the uterus and axillary and pubic hair are absent. In this girl who is a professional ballet dancer with a BMI of 16, the former is more likely.
Low body weight, excessive physical exercise, and starvation are associated with hypogonadotropic hypogonadism (central hypogonadism). This is a very common finding among patients with anorexia nervosa or bulimia nervosa, but is also seen in female athletes as a cause of primary or secondary amenorrhea. If it occurs prior to complete puberty, it could impede puberty from normal progression.
In hypogonadotropic hypogonadism, the characteristic finding is a decreased serum FSH (and LH). Since this patient has normal hormonal assays, an ultrasonography should be performed for assessment of possible uterine anomalies or those of genital tract, resulting in outflow obstruction, some of which include: mullerian agenesis, transverse vaginal septum, Asherman’s syndrome, and imperforated hymen.
One interesting point in the question is that, in fact, with breasts at Tanner II stage, ultrasound should have been preceded the hormonal assays because, for instance, if this girl was found on ultrasound to have no uterus, there was no need to measure FSH and other hormones, and the diagnostic pathway then should be continued with karyotyping (option E) and testosterone assays to investigate complete or incomplete androgen insensitivity as the most likely diagnosis.
Referral to adolescent health centre (option B) may be indicated somewhere down the line once other causes of primary amenorrhea other than those related to excessive exercise, weight loss and eating disorders are excluded with high certainty. It is not appropriate at this stage when other probable causes of primary amenorrhea are not yet fully investigated.
This girl lags behind normal puberty and cannot be reassured (option C) unless the cause is ensured to be constitutional delay of puberty. Interestingly and unlike the common belief, constitutional delay of puberty is not that common in females compared to males.
OPTION D : Screening with Pap smear starts at 25 years of age. It is not indicated for this girl now.
A mother has brought her 16-year-old daughter to your practice for evaluation with the complaint that she has not started her periods yet. On examination, her height is 140 cm. She has breast buds consistent with stage II Tanner. There is axillary and pubic hair growth appropriate for her age. She gives no history of galactorrhea. Which one of the following is the most likely cause to her presentation?
A. A normal variant in sexual development.
B. Hypothalamic hypogonadism.
C. Mullerian agenesis.
D. Ovarian dysgenesis.
E. Pituitary tumour.
D. Ovarian dysgenesis
The main complaint presented in the scenario is primary amenorrhea. Primary amenorrhea is defined as the absence of menses at age 15-16 in the presence of normal growth and secondary sexual characteristics, or 13-14 in the absence of secondary sexual characteristics. According to this definition, this girl has primary amenorrhea and should be thoroughly investigated.
Etiology of primary amenorrhea is diverse but generally can be considered as anatomical or hormonal.
In a large case series of primary amenorrhea, the most common etiologies were:
- Gonadal dysgenesis including turner syndrome - 43%
- Mullerian dysgenesis (absence of vagina, sometimes with absence of uterus) – 15%
- Physiological delay of puberty (constitutional delay of puberty, chronic systemic disease, acute illness) – 14%
- Polycystic ovarian syndrome (PCOS) – 7%
- Isolated gonadotropin-releasing hormone (GnRH) deficiency – 5%
- Transverse vaginal septum – 3%
- Weight loss / anorexia nervosa / exercise induced – 2%
- Hypopituitarism – 2%
- Rare causes - < 1% (imperforated hymen, complete androgen insensitivity syndrome, hyperprolactinaemia/prolactinoma), other pituitary tumors, congenital adrenal hyperplasia, hypothyroidism, central nervous system defects, craniopharyngioma, and Cushing’s disease)
By far, the gonadal dysgenesis (ovarian dysgenesis) including Turner syndrome is the most common cause of primary amenorrhea in female patients.
Gonadal dysgenesis is caused by complete or partial deletion of one X chromosome with the classic Karyotype being (45,OX). The female may also have a mosaic genotype – (45,Ox/46,xx). Short stature and a square appearance is the cardinal common clinical feature of Turner syndrome.
In older adolescents and adults, presenting symptoms of the syndrome usually involves problems of puberty and fertility as well as short stature. Pubic and axillary hair growth occurs at a normal age, but is not an indicator that puberty will progress normally. Breast development may be absent if ovarian failure occurs before puberty, or partially developed if it occurs at some time after puberty has begun. Up to 13% of girls with Turner syndrome have spontaneous breast development or menses; some of these are XO/XX mosaics, with normal gonadotropin responses to luteinizing hormone–releasing hormone (LHRH). Overall, approximately 30% of girls with Turner syndrome have some spontaneous pubertal development.
Turner syndrome should be considered in individuals with primary or secondary amenorrhea and in adult women with unexplained infertility, particularly when such individuals also have a short stature.
Considering the fact that gonadal dysgenesis is the most common cause of primary infertility, and more importantly, with the short stature, gonadal (ovarian) dysgenesis is most likely to underlie the condition. A Tanner II stage breast advocates the diagnosis; however, even more developed breasts do not exclude the diagnosis. Primary amenorrhea and short stature remain the most important pointers towards ovarian dysgenesis.
OPTION A: A 17-year-old amenorrheic girl with a height of 140 is far beyond parameters of normal puberty and cannot be reassured as just having a normal variation in puberty.
OPTION B and C : Mullerian agenesis and hypothalamic hypogonadism are not associated with short stature.
OPTION E : Pituitary tumors are the cause of primary amenorrhea in less than 0.5% of cases. Short statue is not a characteristic feature.
- Medscape - Turner Syndrome
- UpToDate - Clinical Manifestations and diagnosis of Turner syndrome
- UpToDate - Evaluation and management of primary amenorrhea
A 16-year-old girl presents with primary amenorrhea. On examination, she has satisfactory development of breasts and axillary and pubic hair for her age. She is virgin and does not allow you to perform a vaginal exam for further assessments to exclude an imperforated hymen. Which one of the following would be the most appropriate next step in management?
A. CT scan of the pelvis.
B. Ultrasound scan of the pelvis.
C. Convince her to a vaginal exam.
D. Refer her to another doctor.
E. Serum follicle stimulating hormone (FSH).
B. Ultrasound scan of the pelvis
Physical exam is an important part in assessment of females with primary amenorrhea. It should include examination for assessment of presence and development of secondary sexual characteristics, genitalia and clitoris, and a vaginal exam to check for anomalies of the vagina and hymen if feasible. A vaginal exam may reveal an imperforated hymen, absence of vagina (in vaginal agenesis), or short vagina (in mullerian agenesis).
Ultrasonography of the pelvis is another very important part in assessment of every patient with primary amenorrhea, particularly if breasts are over Tanner II stage.
With ultrasound the following are checked:
-Presence or absence of the uterus
-Possible mullerian anomalies
-Ovaries
-Presence of a transverse vaginal septum
-Evidence of menstrual flow obstruction
Since this patient does not consent to vaginal examination, pelvic ultrasonography can provide details as a surrogate, and additionally, further information regarding pelvic structures. Therefore, in this situation, a pelvic ultrasound is the next best in management.
Ideally, both transvaginal and transabdominal sonography should be performed unless the patient’s virginity or refusal precludes the former. It should be noted that if the patient had consented to vaginal exam, the need for ultrasonography was not eliminated.
NOTE - vaginal examination should be restricted to women who are or have been sexually active.
With satisfactory development of breasts and pubic and axillary hair, it is very likely that the cause of this girl’s amenorrhea is mullerian agenesis. Vaginal transverse septum and imperforated hymen are also possible; however, with these conditions cyclical pelvic pain and/or discomfort and/or suprapubic mass would be expected.
OPTION A : CT scan of the pelvis adds nothing to ultrasound findings at this stage. It is more expensive and unnecessarily poses the patient to radiation exposure.
OPTION C : Convincing the patient to the examination as long as ultrasound can provide almost the same efficacy is not appropriate.
OPTION D : Referral to another doctor just because the patient does not consent to vagial exam is not an approriate action.
OPTION E : Serum FSH is unlikely to provide further clarification. With reassuring development of secondary sexual characteristics, FSH is very likely to be within the normal range. Measuring FHS might be indicated somewhere down the road but not at this stage.
- UpToDate - Evaluation and management of primary amenorrhea
- NSW Health – Amenorrhoea
A 16-year-old ballet dancer girl is brought to your office by her mother because she has not had any periods until now. On examination, the girl has a body mass index (BMI) of 16, and breasts at Tanner II stage. No axillary or pubic hair is present. Which one of the following should come next in management?
A. Reassurance.
B. Hormonal assays.
C. Abdominal pelvic ultrasound.
D. Combined oral contraceptive pills (COCPs).
E. Referral to adolescent health center.
B. Hormonal assays
Puberty in girls starts with breast development. Growth spurt then occurs and is followed by axillary and pubic hair development. Menstruation often does not take place until two years after the onset of puberty.
There are a few points in the history that can narrow down the diagnosis. Firstly, this girl has Tanner II breast at the age of 16 which shows lack of breast development as the first sign of puberty (Tanner II breasts after the age of 13 means absence of breast development).
Another point to consider is the fact that this girl has no pubic or axillary hair. This finding can be either due to arrested or delayed progression of puberty, or androgen insensitivity presented with breast development, but absence of a uterus and axillary and pubic hair. In this girl, who is a professional ballet dancer with a BMI of 16, the former is more likely.
In approaching the female patients with primary amenorrhea, the diagnostic approach depends on the presence or absence of secondary sexual characteristics. In patients with absence of such characteristics, hormonal studies including FSH, LH, TSH, and prolactin is the most appropriate next step in investigations, while in the presence of such characteristics, a pelvic ultrasound for uterine or menstrual outflow abnormalities come first.
This girl has amenorrhea and absence of secondary sexual characteristics; therefore, hormonal assay is the most appropriate next step in management.
Low body weight, excessive physical exercise, and starvation are associated with hypogonadotropic hypogonadism (central hypogonadism). This is frequently observed in patients with anorexia nervosa or bulimia nervosa, but also can be seen in female athletes. Hypogonadotropic hypogonadism may result in primary or secondary amenorrhea. If it occurs during puberty, it could impede puberty from normal progression.
Although a decreased serum FSH will be the expected finding associated with the condition, with breast buds present, the most appropriate next step in management is a pelvic ultrasound. Along with physical examination, a pelvic ultrasound is always the very first initial assessment of patients with primary amenorrhea who have secondary sexual characteristics (evident by Tanner II or higher stages of breast development). Ultrasound helps to check for the presence of a uterus, mullerian agenesis, imperforated hymen, etc. as the potential causes of primary amenorrhea. Ideally, both transvaginal and transabdominal pelvic ultrasound should be performed unless virginity precludes transvaginal ultrasound.
OPTION A : This girl lags behind normal puberty and cannot be reassured unless the cause is ensured to be constitutional delay of puberty.
OPTION C : Pelvic ultrasound would be the correct answer if secondary sexual characteristics were present.
OPTION D : Combined oral contraceptives (COCs) have no role in diagnosing or treatment of this girl.
OPTION E : Referral to adolescent health center may be indicated later once other causes of primary amenorrhea other than those related to excessive exercise, weight loss, and eating disorders are excluded with high certainty. It is not appropriate at this stage when other probable causes of primary amenorrhea are not yet fully investigated.
- American Family Physicians - Amenorrhea: An Approach to Diagnosis and Management
- NSW Health - Amenorrhoea
A 32-year-old woman presents to the emergency department with vaginal bleeding at 28 weeks pregnancy. After prompt investigations, the diagnosis of placental abruption is made. Of the following options, which is not expected to be caused by the condition?
A. Tense and tender uterus.
B. Blood pressure of 180/110mmHg.
C. Shock out of proportion to the amount of vaginal bleeding.
D. Fetal head mobile above the pelvic brim.
E. Fetal demise.
B. Blood pressure of 180/110mmHg
Placental abruption occurs when a normally implanted placenta (not in the lower uterine segment) separates from the uterine wall before delivery of the fetus. Separation can be partial or complete. The condition is associated with vaginal bleeding of varying extent and a tense tender uterus (option A) .
Commonly, the bleeding is overt and external (80% of cases); however, at occasions the bleeding may be concealed with blood accumulating between the uterine wall and the placenta. This can lead to uterine hematoma and an increased size of the fundal height. Concealed bleeding, if severe, can result in hemorrhagic shock not justified by the amount of external bleeding (option C) . Fetal head can be felt in the pelvic brim or above it (option D) , depending on the gestational age. An already engaged head can lose its position if placental abruption occurs.
Placental abruption is a serious condition capable of putting both the mother and the fetus in jeopardy. In utero fetal demise (option E) is a major concern.
There are several conditions associated with increased risk of placental abruption, with hypertension being one, but placental abruption does not result in hypertension.
*http://www.uptodate.com/contents/placental-abrupti
*http://www.aafp.org/afp/2004/1001/p1303.html
A 25-year-old woman, on oral contraception pills, presents to your office because she is concerned about 3 episodes of painless vaginal bleeding in the past week. She has had regular cervical screening tests with normal results to date, with the last one taken 12 months ago. Which one of the following is the most likely cause of her postcoital bleeding?
A. Chlamydia infection.
B. Cervical cancer.
C. Endometrial cancer.
D. A cervical polyp.
E. Cervical ectropion.
E. Cervical ectropion
Postcoital bleeding can certainly be caused by Chlamydia cervicitis, a cervical polyp or cervical carcinoma. However, the most likely cause in this instance is a cervical ectropion, where the single layer of columnar epithelium of the endocervix has extended onto the ectrocervix and is exposed to trauma during coitus.
Cervicitis and ectropion are the two most common causes of postcoital bleeding in women younger than 30 years. Although Chlamydia infection (option B) and cervicitis can be a cause as well, absence of other symptoms of Chlamydia infection such as discharge makes such diagnosis less likely than an ectropion cervix.
Cervical polyps (option D) are more common in women of reproductive age, especially those over 30-40 years.
Cervical cancer (option B) and endometrial cancer (option C) are far less likely at this age. Cervical intraepithelial neoplasia does not cause postcoital bleeding per se. However, such lesions often reside in ectropion and can bleed with contact during sexual intercourse.
Other possible causes are pregnancy and its complications such as abortion, HRT, and hormonal contraception.
TOPIC REVIEW
TABLE age-specific common causes irregular uterine bleeding
- Cancer Australia - Abnormal vaginal bleeding in pre- and peri-menopausal women
- AMC Handbook of Multiple Choice Questions – page 530
An 18-year-old girl presents to your office with painful menses for the past 12 months. She has been prescribed NSAIDs initially and then OCPs after the NSAIDs failed to control her symptoms. OCP has been unable to control the painful menses as well. Which one of the following would be the most appropriate next step in management?
A. Transabdominal ultrasound.
B. Transvaginal ultrasound.
C. CT scan of the pelvis and abdomen.
D. Dilation and curettage.
E. Laparoscopy.
B. Transvaginal ultrasound
The main complaint in this patient is with painful periods, also termed dysmenorrhea.
Dysmenorrhea can be divided into two broad categories of primary and secondary.
Primary dysmenorrhea is defined as recurrent, crampy pain occurring with menses in the absence of identifiable pelvic pathology.
Secondary dysmenorrhea is menstrual pain associated with underlying pelvic pathology such as endometriosis.
Primary dysmenorrhea usually begins in adolescence after the establishment of ovulatory cycles. Primary dysmenorrhea is caused by myometrial activity and consequent uterine ischemia. This myometrial activity is modulated and augmented by prostaglandin synthesis; therefore, non-steroidal anti-inflammatory drugs (NSAIDs), by inhibition of prostaglandin synthesis, are the first-line treatment option.
Oral contraceptive pills (OCPs) are used as second-line if NSAIDs fail. OCPs can also be used as the treatment of choice (first-line) if contraception is desired as well. OCPs prevent menstrual pain by suppressing ovulation and decreasing uterine prostaglandin levels. An additional mechanism is the reduction of menstrual flow after several months of use.
If treatment with either of these medications fails after 2 or 3 menstrual cycles, a course of treatment with the other modality should be used. Treatment with both hormonal contraceptives and NSAIDs may be effective in women who do not respond to either drug alone.
Adolescents who fail to respond to first- or second-line treatment, have recurrent symptoms, or have worsening symptoms should be re-evaluated for the possible causes of secondary dysmenorrhea such as endometriosis, uterine leiomyomas, polyps, or other pelvic pathologies.
In this patient, treatment has failed despite use of both NSAIDs and OCP; therefore, secondary dysmenorrhea should be considered and investigated. When pelvic pathology is suspected, abdominal and transvaginal ultrasonography should be used as first-line investigation. They are inexpensive, effective, and readily available. Transvaginal ultrasound is more accurate and the preferred option if possible.
OPTION A : Transabdominal ultrasound is inferior to transvaginal ultrasound for evaluation of dysmenorrhea; but it can be used instead of transvaginal ultrasound if transvaginal is not possible e.g., in virgin girls.
OPTION C : CT scan and MRI are not routinely used for assessment of secondary dysmenorrhea.
OPTION D and E : Dilation and curettage or laparoscopy might be indicated at some stage after initial evaluation points towards a specific pathology such as endometriosis or endometrial problems, but not as an initial step.
- MedScape - Dysmenorrhea
- UpToDate - Primary dysmenorrhea in adolescents
- AAFP - Diagnosis and Initial Management of Dysmenorrhea
A 34-year-old primigravida woman presents to the emergency department at 36 weeks gestation after she passed approximately 200ml of clotted blood per vagina at home 45 minutes ago. Currently, there is no active vaginal bleeding. On examination, she has a pulse rate of 120 bpm and blood pressure of 90/60 mmHg. Abdominal examination reveals a very tense uterus and moderate tenderness on palpation. Fetal heart sound is audible at a rate of 140 bpm. Which one of the following is the most appropriate next step in management?
A. Intravenous fluids.
B. Ultrasound.
C. Cardiotocography (CTG).
D. Induction of labor.
E. Immediate cesarean section.
A. Intravenous fluids
The clinical picture of a tense and tender uterus in the presence of vaginal bleeding is suggestive of placental abruption as the most likely diagnosis. The important clue is tachycardia and hypotension unproportionate to the amount of vaginal bleeding indicating that the bleeding is concealed. In concealed bleeding blood accumulates between the placenta and the uterine wall.
In the presence of hypotension and tachycardia, fluid resuscitation with intravenous isotonic fluids is the most appropriate next step in management.
Blood typing and cross match should also be performed and packed cells reserved in case of severe bleeding requiring blood transfusion.
An ultrasound scan (option B) is then indicated for assessment of the placental and fetal status. Continuous CTG (option C) should be performed for fetal monitoring. Labor induction option D) or cesarean section (option E) are indicated if there is maternal or fetal jeopardy.
*http://www.sahealth.sa.gov.au/wps/wcm/connect/b1c6
*http://3centres.com.au/guidelines/complications-in
*http://www.kemh.health.wa.gov.au/development/manua
A 29-year-old woman with history of schizophrenia for the past nine years presents to your practice, accompanied by her husband, because of amenorrhea of 2 months duration. She is currently stable on clozapine. Which one the following is most important step in management?
A. Ultrasound.
B. Measurement of serum follicle stimulating hormone (FSH).
C. Measurement of serum luteinizing hormone (LH).
D. Urine pregnancy test.
E. Full blood count (FBC).
D. Urine pregnancy test
Universally, the most common cause of secondary amenorrhea is pregnancy. Hence, in every woman of reproductive age presenting with amenorrhea, the first thing to check is a urine pregnancy test.
An ultrasound or measuring FSH and LH may be used later (if indicated) for assessment of the condition, once pregnancy has been excluded.
FBC is used to monitor adverse outcomes of clozapine such as neutropenia or agranulocytosis. It is not indicated for evaluation of amenorrhea.
- NSW Health – Amenorrhoea
A 20-year-old woman is planning to conceive and has presented for your advice. She has the history of juvenile myoclonic epilepsy, and has been stable on sodium valproate. Despite your full explanations about the risks of the teratogenicity of antiepileptic drugs, she insists that she wants to become pregnant and asks you to show her a way. Which one of the following would be the most appropriate advice?
A. Continue sodium valproate.
B. Discontinue sodium valproate.
C. Switch to carbamazepine.
D. Switch to lamotrigine.
E. Switch to phenytoin.
A. Continue sodium valproate
When approaching a woman, who wishes to become pregnant while on antiepileptic drugs, two major groups of epilepsies should be distinguished because they typically respond differently to different drugs.
Partial epilepsies respond to most antiepileptic drugs, but for idiopathic generalized epilepsies, especially in juvenile myoclonic epilepsy, seizure can be controlled with a reasonably low dose of sodium valproate.
Option D: Although lamotrigine may be helpful, it is not as effective as sodium valproate, and even sometimes worsens the myoclonic seizures of juvenile myoclonic epilepsy. Therefore, substitution of sodium valproate with lamotrigine is NOT the right advice.
Topiramate and levetiracetam may be effective in idiopathic generalized epilepsy, while carbamazepine , phenytoin and gabapentin may worsen some seizure types, especially myoclonic and absence seizures. For some women with idiopathic generalized epilepsies, there may be no effective alternative to sodium valproate. Cessation of sodium valproate is associated with recurrence of the generalized seizures, especially juvenile myoclonic epilepsy; therefore, it is not advisable to discontinue the drug.
On the other hand, sodium valproate has the highest teratogenicity potential among antiepileptic drugs. The Australian Pregnancy Register has reported the risk to be as high as 16% for the first trimester. Sodium valproate should therefore be avoided in women of reproductive age. If a patient is willing to become pregnant, she should be fully informed of the risk of teratogenicity and the decision is left to her. If she decides to accept the risks, and pregnancy is unavoidable, the lowest effective dose should be used.
If sodium valproate dose has been lowered to a minimum during pregnancy to reduce teratogenesis, the prepartum effective dose may need to be re-established before the onset of labor. This is a time of increased seizure risk, especially in patients with idiopathic generalized epilepsy who are very sensitive to sleep deprivation. Breastfeeding is safe during valproate therapy.
- Australian Prescriber - Antiepileptic drugs in pregnancy and lactation
A pregnant elementary school teacher, aged 24 years, presents to your GP office after she found that one of the students in her class has been recently diagnosed with rubella by his GP; the diagnosis however was not confirmed by serologic studies. Her last vaccination against rubella was when she was 12 years. Which one of the following would be the next best step in management?
A. No action is required; reassure her.
B. Check rubella serology.
C. Confirm the diagnosis of rubella in the sick child with rubella serology testing.
D. Give her a booster dose of MMR vaccine.
E. Advise her to terminate the pregnancy.
B. Check rubella serology
All pregnant women with contact with a person, who has confirmed diagnosis of rubella infection or is suspicious of having the disease, should be offered serologic testing (both IgM and IgG) for rubella as the most appropriate next step in management. However, if the patient had been screened for rubella IgG in the first trimester and had a positive IgG with titers >10 IU/ml, no further serologic testing would have been indicated.
OPTION A : Taking no action is incorrect as serologic testing is indicated in all pregnant patients after contact to a rubella or rubella-like infection, unless the woman has been documented to be immune in the current pregnancy using IgG titers as mentioned earlier.
OPTION C : There is no need for serologic testing in the index case (the child in this scenario).
OPTION D : Rubella-containing vaccines (MMR, MMRV) are live attenuated vaccines and contraindicated throughout pregnancy.
Advising the patient to terminate the pregnancy is recommended if there is a with definite diagnosis of rubella infection (confirmed by serology) in the first trimester, as it is associate with fetal anomalies in almost 85% of cases.
TOPIC REVIEW
Rubella, also called German measles, is usually a mild infectious disease in children and adults. It is clinically difficult to diagnose due to transient clinical features that are also common to some other viral infections.
Rubella has an incubation period of 14-23 days. The infectivity period starts from one week before until 4 days after onset of rash.
Rubella is asymptomatic in 25 to 50 % of cases. If there are symptms, they may include:
-Low grade fever
-Transient erythematous rash
-Lymphadenopathy involving post-auricular and sub-occipital nodes
-Occasionally arthritis and arthralgia (commonly observed in women of child-bearing age)
-Rarely neurological disorders and thrombocytopenia
-Rash - the rash characteristically begins on the face and spreads to the trunk and extremities. It will usually resolve within three days in the same order in which it appeared (face first and then body)
During pregnancy, maternal viremia may occur 5 to 7 days after exposure with spread of the virus throughout the body as well as transplacental infection of the fetus.
Vertical transmission (transplacental) from mother to fetus can only occur during viremia of a primary infection. The effect of maternal infection on the fetus depends on the gestational age:
- <8 weeks: up to 85% of fetuses will be infected and all will have clinical manifestations of the congenital rubella syndrome.
- 8≤ but <12 weeks: 50-80% of fetuses will be infected and of those 65-85% will be clinically affected.
- 13-16 weeks: 30% of the fetuses will be infected and of those, 1/3 will have sensorineural deafness.
- 16-19 weeks: 10% of the fetuses will be infected. Clinical features are rare, yet deafness can be a possibility.
- > 19 weeks: there is no apparent risk.
NOTE - reinfection is associated with far less risk of fetal infection (~5%.)
Abnormalities associated with congenital rubella syndrome include:
-Central nervous system dysfunction (10-25%, intellectual impairment, developmental delay, microcephaly)
-Eye abnormalities (10-25%, cataracts, retinopathy, glaucoma, strabismus, micropthalmos)
-Sensorineural deafness (60-75%)
-Cardiac abnormalities (10-20%, PDA, PA stenosis)
-Intrauterine growth restriction, short stature
-Inflammatory lesions of the brain, liver, lungs and bone marrow
All pregnant women, who have contact with rubella or clinical features consistent with rubella – like illness should be screened for the presence of rising antibody titre (IgG) and rubella specific IgM, regardless of previous history of vaccination or infection. Interpretation of the results and corresponding action is as follows:
SEE TABLE BELOW
If maternal infection is confirmed, antenatal testing is recommended at least 6 weeks after known maternal infection. Rubella PCR, rubella culture and fetal IgM can be performed following chorionic villus sampling (CVS) / amniocentesis or cordocentesis; however, due to the very high risk of fetal infection and consequent fetal anomalies, if maternal infection occurs in the first trimester, termination of pregnancy should be recommended.
NOTE - In all pregnant women, serologic testing for rubella (IgG only) should be offered and performed in the first antenatal visit. With titers ≥10 IU/ml, risk of reinfection is minimal. However, if the titers are ≤15, vaccination after delivery should be performed.
*https://www.sahealth.sa.gov.au/wps/wcm/connect/d81
*https://www.asid.net.au/documents/item/368
A kindergarten teacher presents to your clinic at 10 weeks pregnancy. Recently, she has found that one of the children in her class has rubella infection. Which one of the following is the most appropriate next step in management?
A. Exclude the sick child and reassure her.
B. Terminate the pregnancy before 13 weeks.
C. Check anti-rubella IgM and IgG levels.
D. Tell her to come back if she becomes symptomatic.
E. Give her MMR vaccine.
C. Check anti-rubella IgM and IgG levels
All pregnant women should be offered serologic testing for rubella as the most appropriate next step in management if there is exposure to rubella infection through contact with an infectious case. This is true for all pregnant women regardless of the past history of immunization or infection with rubella. However, if the patient was screened for rubella IgG in the first trimester and had a positive IgG with titers >10 IU/ml no further serologic testing would have been indicated.
OPTION A : Exclusion of the child until full recovery or at least four days after the onset of the rash is recommended, but this will not affect the management plan for the woman. She is at risk of contracting rubella and its detrimental impacts on her fetus, and cannot be reassured until investigations exclude rubella infection.
OPTION B : Termination of the pregnancy is recommended for pregnant woman with established diagnosis of rubella within the first trimester of pregnancy, confirmed with serology studies. Rubella infection in the first trimester is associated with severe fetal anomalies in 85% of cases.
OPTION D : Telling her to come back if she is symptomatic is inappropriate. She should undergo serologic studies now.
OPTION E : Rubella-containing vaccines (MMR, MMRV) are live attenuated vaccines and are contraindicated during pregnancy.
TOPIC REVIEW
Rubella, also called German measles, is usually a mild infectious disease in children and adults. It is clinically difficult to diagnose due to transient clinical features that are also common to some other viral infections.
Rubella has an incubation period of 14-23 days. The infectivity period starts from one week before until 4 days after onset of rash.
Rubella is asymptomatic in 25 to 50 % of cases. If there are symptms, they may include:
-Low grade fever
-Transient erythematous rash
-Lymphadenopathy involving post-auricular and sub-occipital nodes
-Occasionally arthritis and arthralgia (commonly observed in women of child-bearing age)
-Rarely neurological disorders and thrombocytopenia
-Rash - the rash characteristically begins on the face and spreads to the trunk and extremities. It will usually resolve within three days in the same order in which it appeared (face first and then body)
During pregnancy, maternal viremia may occur 5 to 7 days after exposure with spread of the virus throughout the body as well as transplacental infection of the fetus.
Vertical transmission (transplacental) from mother to fetus can only occur during viremia of a primary infection. The effect of maternal infection on the fetus depends on the gestational age:
- <8 weeks: up to 85% of fetuses will be infected and all will have clinical manifestations of the congenital rubella syndrome.
- 8≤ but <12 weeks: 50-80% of fetuses will be infected and of those 65-85% will be clinically affected.
- 13-16 weeks: 30% of the fetuses will be infected and of those, 1/3 will have sensorineural deafness.
- 16-19 weeks: 10% of the fetuses will be infected. Clinical features are rare, yet deafness can be a possibility.
- > 19 weeks: there is no apparent risk.
NOTE - reinfection is associated with far less risk of fetal infection (~5%.)
Abnormalities associated with congenital rubella syndrome include:
-Central nervous system dysfunction (10-25%, intellectual impairment, developmental delay, microcephaly)
-Eye abnormalities (10-25%, cataracts, retinopathy, glaucoma, strabismus, micropthalmos)
-Sensorineural deafness (60-75%)
-Cardiac abnormalities (10-20%, PDA, PA stenosis)
-Intrauterine growth restriction, short stature
-Inflammatory lesions of the brain, liver, lungs and bone marrow
All pregnant women, who have contact with rubella or clinical features consistent with rubella – like illness should be screened for the presence of rising antibody titre (IgG) and rubella specific IgM, regardless of previous history of vaccination or infection. Interpretation of the results and corresponding action is as follows:
(SEE TABLE BELOW)
If maternal infection is confirmed, antenatal testing is recommended at least 6 weeks after known maternal infection. Rubella PCR, rubella culture and fetal IgM can be performed following chorionic villus sampling (CVS) / amniocentesis or cordocentesis; however, due to the very high risk of fetal infection and consequent fetal anomalies, if maternal infection occurs in the first trimester, termination of pregnancy should be recommended.
NOTE - In all pregnant women, serologic testing for rubella (IgG only) should be offered and performed in the first antenatal visit. With titers ≥10 IU/ml, risk of reinfection is minimal. However, if the titers are ≤15, vaccination after delivery should be performed.
*https://www.sahealth.sa.gov.au/wps/wcm/connect/d81
*https://www.asid.net.au/documents/item/368
A 24-year-old woman presents at 39 weeks gestationy with complaint of intermittent watery vaginal discharge, which started last night after she had sex with her husband. On speculum examination, pooling of straw-colored fluid in the posterior vaginal fornix is noted. The cervical os is closed. Furthermore, there is leak of liquor from cervical os when she is asked to cough or strain. Which one of the following best explains these clinical findings?
A. Retained semen from the last night sex.
B. Vaginal infection.
C. Premature rupture of membranes (PROM).
D. Urine leakage.
E. Cervical insufficiency.
C. Premature rupture of membranes (PROM)
The presentation is classic for premature rupture of membranes (PROM), probably caused by trauma during intercourse.
By definition, premature rupture of membranes (PROM) is defined as rupture of embryonic membranes before the labor regardless of the age of pregnancy. If PROM occurs before 37 weeks gestation, it is termed as premature PROM (PPROM).
The classic presentation of rupture of the membranes (ROM), regardless of gestational age, is with a sudden gush of watery fluid per vagina; however, many women, such as in this scenario may complain of continuous or intermittent leakage of fluid or the sensation of wetness within the vagina or on the perineum. Presence of liquor flow from the cervical os or pooling in the posterior vaginal fornix is pathognomonic.
If there is doubt, further tests can be conducted to clinch the diagnosis:
Nitrazine test – the normal vaginal pH is between 4.5 and 6.0, whereas amniotic fluid is more alkaline, with a pH of 7.1 to 7.3. Nitrazine paper will turn blue when the pH is above 6.0; however, the presence of contaminating substances such as blood, semen, alkaline antiseptics can also cause nitrazine paper to turn blue, giving a a false-positive result. Bacterial vaginosis alkalinizes the vagina as well.
Amnisure test - a non-invasve and approximately 99% accurate for definite diagnosis of PROM. It is based on immunoassay, can be done at any gestational age and does not need a speculum exam.
NOTE - The above tests are currently used in practice in Australia.
Ferning on microscopy – using a swab, fluid is obtained from the posterior fornix or vaginal sidewalls. Once the fluid is dried on the slide, the slide is checked for ferning (arborization) under low-power microscope. The presence of ferning indicates PROM. It is noteworthy that vaginal blood can obscure presence of ferns, and that cervical mucus can result in false-positive result if the cervical os has been swabbed. Ferning test is no more performed in Australia.
NOTE - ultrasonography is not used merely for diagnosis of PROM; however, in rare cases when the diagnosis cannot be certain using history, physical exam, and mentioned tests ultrasonography may help. An example is when the patient’s history suggests PROM, but physical findings findings are not consistent with the diagnosis. Ultrasonography, however, is a part of investigation plan for assessment of fetal well-being, the position of the fetus, placental location, estimated fetal weight and presence of any anomalies in PROM and PPROM.
Retained semen will have a different appearance and will not result in findings in the clinical scenario.
OPTION B : Infections will be associated with characteristic features including purulent cervical discharge, malodorous vaginal discharge, etc. Pooling of clear fluid in the posterior fornix is pathognomonic for ROM.
OPTION E : Urine leakage is common during the pregnancy, but it is inconsistent with such a clinical scenario.
OPTION E : Although cervical insufficiency is associated with increased vaginal discharge, absence of other findings on speculum exam such as cervical dilation, and bulging membranes makes this diagnosis unlikely.
- https://www.ranzcog.edu.au/Statements-Guidelines/Obstetrics/Preterm-Prelabour-Rupture-of-Membranes
In which one of the following women groups, endometrial hyperplasia is most likely to be found?
A. An ovulating woman.
B. An obese diabetic woman.
C. A woman on cyclic combined oral contraceptive pills.
D. A woman on Depo-Provera® for treatment of endometriosis.
E. A woman with an intrauterine device.
B. An obese diabetic woman
Endometrial hyperplasia is characterized by proliferation of endometrial glands. The condition may progress to or coexist with endometrial carcinoma. Endometrial hyperplasia virtually always results from chronic exposure of endometrial tissue to estrogen (sometimes in excess amounts) unopposed by the counterbalancing effects of progesterone.
Excess estrogen exposure can be exogenous or endogenous. Exogenous estrogen exposure includes postmenopausal estrogen therapy and tamoxifen, while endogenous exposure may result from obesity anovulatory menstrual cycle, or estrogen-secreting tumors.
Of the options, diabetes and obesity are associated with increased risk of endometrial hyperplasia. Diabetes mellitus type 2 is associated with increased levels of insulin-like growth factor (IGF). IGF can cause endometrial proliferation. Increased body mass index (BMI) causes excessive peripheral conversion of androgens to estrogen in adipose tissue.
NOTE - Risk factors for endometrial hyperplasia are essentially the same.
Risk factors for endometrial cancer/hyperplasia and their relative risk are summarized in the following table:
Risk factor (Relative Risk, %)
-Unopposed estrogen therapy (2-10%)
-Increasing age (1.4%) (50-70 years)
-Late menopause (>55 years) (2%)
-Nulliparity (2%)
-Chronic anovulation (e.g. in PCOS) (3%)
-Obesity (2-4%)
-Diabetes mellitus (2%)
-Tamoxifen therapy (2%)
-Lynch syndrome (hereditary nonpolyposis colorectal cancer) (22-50%) (lifetime risk)
-BRCA1/BRCA2 gene mutation (20%)
-Early menarche (Not specified)
-Estrogen secreting tumor (Not specified)
-Family history of endometrial, ovarian, breast, or colon cancer (Not specified)
OPTION A : Ovulating women has cyclical exposure to progesterone. This leads to reduces risk of endometrial hyperplasia. For the same reason, women on cyclical combined oral contraceptive are unlikely to be at increased risk of endometrial hyperplasia.
OPTION C : Combined oral contraceptive pills (COCs) contain progesterone that counterbalances the effects of estrogen. Women on COCs do not have an increased risk of endometrial hyperplasia/cancer.
OPTION D : Depo-Provera® is injectable progesterone. Progesterone counterbalances the effect of estrogen on endometrial hyperplasia. It is a protective factor rather than risk factor.
OPTION E : Intrauterine devices have not been shown to be associated with increased risk of endometrial hyperplasia.
- UpToDate - Classification and diagnosis of endometrial hyperplasia
- RCOG - Management of Endometrial Hyperplasia
Which one of the following is the most likely finding in a pregnant woman with placental abruption?
A. Decreased fetal movements.
B. Preterm labor.
C. Vaginal bleeding.
D. Uterine tenderness.
E. Shock.
C. Vaginal bleeding
Placental abruption is defined as the premature separation of the placenta from the uterus and presents with bleeding, uterine contractions, and fetal distress. Placental abruption is a significant cause of third-trimester bleeding and is associated with fetal and maternal mortality and morbidity. The condition should be considered as a differential diagnoses in all pregnant women with vaginal bleeding in the second half of the pregnancy.
Vaginal bleeding is the most common presenting symptom reported by 80% of women with placental abruption. In 20% of women with placental abruption, vaginal bleeding is concealed; therefore, absence of vaginal bleeding does not exclude placental abruption.
Frequency of symptoms in women with placental abruption is as follows:
-Vaginal bleeding – 80%
-Abdominal or back pain and uterine tenderness – 70%
-Fetal distress – 60%
-Abnormal uterine contractions (e.g. hypertonic, high frequency) – 35%
-Idiopathic premature labor – 25%
-Fetal death – 15%
Exam findings may include:
-Vaginal bleeding
-Uterine contractions and/or tenderness
-Shock
-Absence of fetal heart sounds
-Increased fundal height do to an expanding hematoma
Shock is seen in class 3 placental abruption that represents 24% of all cases.
TOPIC REVIEW
Placental abruption can be classified clinically, based on the extent of separation (partial versus complete) or location of the separation (marginal versus central). Clinical classification of placental abruption, the prevalence of each class and its clinical features of each class is as follows:
Class 0 – asymptomatic Presentation
A diagnosis of class 0 is made retrospectively by finding an organized blood clot or a depressed area on a delivered placenta. Class 1 – Mild (represents approximately 48% of all cases)
Presentation
-No vaginal bleeding to mild vaginal bleeding
-Slightly tender uterus
-Normal maternal blood pressure and heart rate
-Absence of coagulopathy
-No fetal distress
Class 2 – moderate (represents approximately 27% of all cases)
Presentation
-No vaginal bleeding to moderate vaginal bleeding
-Moderate to severe uterine tenderness with possible tetanic contractions
-Maternal tachycardia with orthostatic changes in blood pressure and heart rate
-Fetal distress
-Hypofibrinogenemia (i.e. 50-250 mg/dL)
Class 3 - Severe (represents approximately 24% of all cases)
Presentation
-No vaginal bleeding to heavy vaginal bleeding
-Very painful tetanic uterus
-Maternal shock
-Hypofibrinogenemia (ie, < 150 mg/dL)
-Coagulopathy
-Fetal death
- Medscape - Abruptio Placentae Clinical Presentation
A 28-year-old 10-week pregnant woman presents 24 hours after eruption of a rash, characteristic of measles. The history and exam findings establish the diagnosis with certainty. Which one of the following is the most appropriate next step in management?
A. Give MMR vaccine.
B. Administer immunoglobulin.
C. Contact tracing.
D. Check her antibody status.
E. No action is required.
C. Contact tracing
The measles (rubeola) virus is a single-stranded RNA virus of the family Paramyxoviridae.
Measles is highly infectious. The main route of transmission is by respiratory airborne droplets; however, it can rarely be passed on to others by means of articles soiled with respiratory droplets.
Measles has an incubation period of 10-14 days.
Initially, the illness presents with the following:
-Fever
-Malaise
-Cough
-Coryza (inflammation of the mucous membranes of the nose)
-Conjunctivitis
-Koplik’s spots (white spots, each surrounded by a red ring, found on the buccal mucosa)
Two to 4 days after the initial symptoms, a maculopapular rash develops. The rash typically starts from face and upper neck and later spreads to involve the whole body. The infectivity periods starts 2 days before the onset of the rash and continues for 4 days after the eruption.
Measles is often a severe disease. Complication may occur, and include otitis media in 7% and bronchopneumonia in 6% of immunocompetent patients. Acute encephalitis is a rare complication occurring in between 2 and 10 per 10,000 reported cases with measles in the general population, with an associated mortality rate of 10 -15 %. Around 15-40 % of survivors will have permanent brain damage.
In pregnant women measles can be associated with increased risk of the following:
-Preterm labor
-Spontaneous abortion
-Fetal / neonatal loss
-Maternal mortality
This woman has established diagnosis of measles and is already at increased risk of above-mentioned complications. There is no active treatment for her except conservative management; however, since measles is a ‘notifiable’ disease, notification should be made as soon as possible as the most appropriate next step in management.
OPTION A : MMR is vaccine has no role in management of established measles. Furthermore, MMR is a live attenuated vaccine and is contraindicated throughout the pregnancy.
OPTION B : Normal human immunoglobulin (NHIG) is used for prophylaxis in contacts of an index case with measles in whom MMR vaccine is contraindicated, e.g., pregnant woman.
OPTION D : Checking antibody status in this patient with established measles is of no use, and will not change the management plan.
OPTION E : Taking no action while measles is a notifiable disease is not appropriate. A mentioned before all cases of measles should be reported.
- SA Health - Measles and measles contacts in pregnancy
A 27-year-old pregnant woman presents to your practice at 17 weeks gestation with complaint of eruption of a rash, following a 2-day history of malaise, low grade fever and rhinorrhea. You suspect measles and order serology tests for her. IgM against measles is positive; IgG is negative. Which one of the following is the most appropriate next step in management?
A. MMR vaccine.
B. Serologic testing of the fetus for measles.
C. Administration of natural human immunoglobulin (NHIG) within 6 days.
D. Contact tracing.
E. Repeat the serology in 2 weeks.
D. Contact tracing
The scenario describes a typical case of measles infection in a pregnant woman, confirmed by serologic studies. Positive IgM suggests acute infection, while negative IgG shows that the infection is in its early phase and seroconversion has not occurred yet.
After contracting measles, conservative management of the symptoms and its potential complications is the only management plan therapeutically.
Another very important issue is ‘notification’ and contact tracing. Measles is a notifiable disease and healthcare professionals are mandated to report cases of measles to the authorized public health units. Contact tracing is the main objective of this notification.
Control of measles relies on early diagnosis and notification of the cases, prompt isolation of infectious cases, and timely and effective identification of contacts, with provision of advice and post-exposure prophylaxis, as appropriate.
Since measles is primarily transmitted by air-borne means, a contact is defined as anyone, who has or may have shared the same air-space (enclosed area) for any length of time with an infectious case. Contact tracing should aim to identify those most susceptible to measles or at greater risk from infection, including infants, immunocompromised people and pregnant women.
In general, contacts may be prioritized in the following order, recognizing that it may not be feasible for the PHU to identify and arrange post-exposure prophylaxis for all susceptible contacts, given the constraints of time, resources and logistics:
*The household and other settings where people share communal facilities (for example, in a hospital, boarding school or military barracks)
*Early childhood education and care services, school or other educational settings where people share a classroom with the case
*People who shared a waiting area at the same time as the case (for example, patients in a healthcare facility’s waiting room and any people accompanying these patients) and people who used the waiting area or who were seen in the same consulting room for up to 30 minutes after the case left. The latter period is based on the recognition that normal room ventilation systems ensure that levels of airborne viruses and virus survival diminish rapidly.
*Work settings where work colleagues have shared the same work area or communal facilities at the same time as a case.
OPTION A : MMR vaccine is used for prevention of measles including post-exposure prophylaxis. It is not useful once measles is contracted. Even for prophylaxis, MMR was contraindicated throughout pregnancy. For those with contact to a case of measles, MMR vaccine within 72 hours of contact may have a protective effect. All measles-containing vaccines (MMR, MMRV) are contraindicated throughout pregnancy.
OPTION B : Serologic testing of the fetus is not useful because circulating maternal antibodies cross the placenta into fetal ciculation. A positive test does not mean that the fetus is infected.
OPTION C : NHIG is used as a post-measles exposure prophylactic means for those persons with contraindications to MMR vaccine such as pregnant women, premature babies, etc. It is not useful for treatment of established measles.
OPTION E : Repeating measles-specific serologic test is not necessary in this case because both symptoms and IgM levels indicate measles as the diagnosis. In general no test is indicated, unless the result has an impact on further management or is of prognostic value. Repeating serologic tests in a patient with established measles is futile.
*http://www.health.gov.au/internet/main/publishing.
A 27-year-old woman presents to the primary care center where you work at 26 weeks gestation after she has been noticing intermittent leakage of watery liquor per vagina for the past eight hours, especially after straining, coughing or sneezing. You perform a speculum vaginal exam which reveals clear fluid in the posterior vaginal fornix, as well as flow of liquid through the cervical os. Further evaluation, establishes preterm premature rupture of the membranes (PPROM) as the diagnosis. No uterine contraction is felt. There is a tertiary hospital 50 km away. Which one of the following is the most appropriate next step in management?
A. Administration corticosteroids.
B. Commencement of tocolysis.
C. Transferring her to the tertiary hospital.
D. Sending her home on oral antibiotics and warning signs.
E. Admission for observation.
A. Administration corticosteroids
The scenario is a classic presentation of preterm premature rupture of membranes (PPROM). Rupture of membrane harbingers labor in term or near term women. If ROM does not end up in established labor in 4 hours, it is termed premature ROM (PROM). In other words, PROM is defined by ROM befire the laor onset. If it occurs before 37 + 0 days weeks, PPROM is the preferred entity, as approach to PROM and PPROM are different.
The classic presentation of rupture of the membranes (ROM), regardless of the gestational age, is with a sudden gush of watery fluid per vagina or continuous or intermittent leakage of fluid or the sensation of wetness within the vagina or on the perineum. Presence of liquor flow from the cervical os or pooling in the posterior vaginal fornix is pathognomonic.
PPROM is associated with the following risks:
-Preterm labor
-Cord prolapse
-Placental abruption
-Chorioamnionitis
-Fetal pulmonary hypoplasia and other features of prematurity
-Limb positioning defects
-Perinatal mortality
Diagnosis of women with PPROM, as usual, begins with history and physical exam. Fetal heart rate auscultation and monitoring (+/- CTG) is another important step. A speculum vaginal exam should be performed to exclude cord prolapse, to assess the cervical length, and visualize pooling of liquor in the posterior fornix. High and low vaginal swabs should be obtained. Samples should be taken for nitrazine test.
Ultrasound may be used for estimation of amniotic fluid volume if the diagnosis is not certain, as well as for assessment of fetal wellbeing, estimated fetal weight, location of placenta, etc.
Once the diagnosis is certain the following should be considered as components of a management plan:
1. Maternal corticosteroids
Corticosteroids are effective in reducing adverse perinatal outcomes such as respiratory distress syndrome, intraventricular hemorrhage, and necrotizing enterocolitis. In case respiratory distress develops, duration of neonatal respiratory support is significantly reduced if corticosteroids have been administered.
Corticosteroids are indicated in the following situations:
-Gestational age is between 23+0d and 34+6d weeks if preterm labor is a concern
-Preterm birth is planned or expected within the next 7 days
Recommended regimens are IM betamethasone in two doses of 11.4 mg, 24 hours apart, to the woman. If betamethasone is unavailable, IM dexamethasone in two doses of 12 mg, 24 hours apart, is given.
When the gestational age is less than 32 + 6d, a single repeat dose of corticosteroid may be given seven days or more after the first dose if the woman is still considered to be at risk of preterm labor. Up to 3 repeat doses might be considered.
**2. Antibiotics **
The rationale for antibiotic prophylaxis is that infection appears to be both a cause and consequence of PPROM and is related to preterm delivery. The objective of antibiotic therapy is to reduce the frequency of maternal and fetal infection and delay the onset of preterm labor.
Antibiotic prophylaxis is indicated in all women with PPROM after low and high vaginal swabs are taken. In the absence of chorioamnionitis recommended antibiotic prophylaxis of choice is erythromicin given for 10 days intravenously.
Chorioamnionitis is a feared complication of PPROM. The diagnosis of chorioamnionitis relies on the clinical presentation and may be difficult in its early manifestations. The clinical picture may include maternal fever with 2 or more of the following:
-Increased white cell count (> 15 x 109 / L)
-Maternal tachycardia (> 100bpm)
-Fetal tachycardia (>160bpm)
-Uterine tenderness
-Offensive smelling vaginal discharge
-C-Reactive Protein > 40
3. Tocolytics
Chorioamnionitis is an absolute contraindication to tocolysis, while PPROM in the absence of intrauterine infection is only relatively contraindicated. The rationale for PPROM being so is that it is often difficult to exclude infections in the presence of PPROM because at times uterine contractions can be the only presenting symptom.
However, where contractions are present, nifedipine, as a tocolytic, may be started to prolong pregnancy for 48 hours while corticosteroid cover is established if there are no other signs of chorioamnionitis.
If not in labor or labor is not anticipated immediately, transfer to a tertiary care facility should be considered if possible. This ensures prompt care of the premature neonate if preterm labor and delivery occurs.
—
In this woman with PPROM at 26 weeks gestation, administration of corticosteroids as outlined above is the most appropriate next step in management.
For the moment, there are no signs and symptoms suggesting intrauterine infection; nonetheless, prophylactic antibiotics should be started to prevent infections as well as to delay labor.
OPTION B : Tocolysis using nifedipine was indicated if the woman was in labor. Cessation of labor for at least 48 hours provides a window for corticosteroid to establish its effects. In the absence of uterine contractions suggesting labor, tocolysis is not needed.
OPTION C : Transferring this woman to a tertiary hospital is an appropriate option after first doses of corticosteroid and antibiotic are given. Being in a tertiary hospital ensures optimal neonatal care in case of premature delivery.
OPTION D : Discharging this woman home on oral antibiotics is not an appropriate option because she needs investigations and fetal monitoring as well as close observation for dvelopment of any signs of infection and preterm labor.
OPTION E : Admission to a primary care center with no neonatal ICU (NICU) adds no benefit to the outcome.
*https://www.ranzcog.edu.au/Statements-Guidelines/Obstetrics/Preterm-Prelabour-Rupture-of-Membranes
A 23-year-old woman presents to your clinic at 26 weeks gestation after a sudden gush of watery fluid while cooking. Examination establishes the diagnosis of preterm premature rupture of membranes (PPROM). Fetal assessment using ultrasound and cardiotocography (CTG) is reassuring. Vaginal examination using a speculum shows fluid in the posterior vaginal fornix and a 1cm dilated cervical os. Uterine contractions are absent. There are two hospitals in the vicinity: a primary hospital and a tertiary hospital, 50km and150 km away, respectively. After administration of antibiotics and corticosteroids, which one of the following would be the most appropriate step in management of this patient?
A. Give him the warning signs and send her home.
B. Send her to the secondary care hospital 50km away.
C. Send her to the tertiary care chospital 150km away.
D. Tocolysis.
E. Labor induction.
C. Send her to the tertiary care chospital 150km away
PROM is defined as rupture of fetal membranes before labor at any gestational age. If PROM occurs before, it is termed premature PROM (PPROM). The main concerns in PPROM are preterm labor, which is associated with neonatal complications due to prematurity if labor establishes, and intrauterine infections with chorioamnionitis being of greatest significance.
If PPROM occurs before 34+6d, corticosteroids should be given to the mother in an attempt to decrease neonatal complications such as respiratory distress syndrome, intraventricular hemorrhage, and necrotizing enterocolitis.
Intravenous prophylactic antibiotics are given in anticipation of chorioamnionitis as a major complication of PPROM. Currently, intravenous erythromicin is the common practice is Australia as it is safe and mildy tocolytic.
Women with PPROM are at significantly elevated risk of preterm labor. The premature baby is required to be cared for in a center with optimal neonatal care in a neonatal intensive care unit (NICU). This can be found in a tertiary hospital. Therefore, if a woman with PPROM is not in labor and the labor is not anticipated immediately, the most appropriate next step will be transferring her to a tertiary hospital with NICU.
OPTION A : Expectant management at home may be considered for all man after 72 hours of hospitalization if there is:
-No further fluid leakage.
-Singleton pregnancy AND
-Cephalic presentation > 23 weeks AND
-Easy access to the hospital
This woman just had PPROM and has not been intensively evaluated in a hospital. Moreover, the nearest tertiary hospital is 150km away. Therefore, she does not meet criteria for expectant home care.
OPTION B : Secondary care hospitals do not have NICU and add no benefit to the care of the patient in this regard.
OPTION D : If the patient was in labor, evident by presence of uterine contractions and a dilating cervix, use of tocolytic agents were indicated to stop labor for at least 48 hours. This could provide a window for corticosteroids to exert their effects, as well as providing time for transfer to a tertiary care facility. Tocolysis is contraindicated in the presence of intrauterine infection, but only relatively contraindicated in PPROM; therefore, in the absence of intrauterine infection benefits of tocolysis outweigh risks of prematurity in women with (P)PROM and with uterine comtractions.
OPTION E : Induction of labor is not indicated due to prematurity unless:
-Signs of chorioamnionitis are present
-There is anterpartum hemorrhage
-The woman requests so
-There is neonatal jeopardy.
- https://www.ranzcog.edu.au/Statements-Guidelines/Obstetrics/Preterm-Prelabour-Rupture-of-Membranes
Which one of the following drugs can potentially cause the most harmful neurological defects in a fetus if given during pregnancy?
A. Amphetamine.
B. Morphine.
C. Cocaine.
D. Amitriptyline.
E. Alcohol.
C. Cocaine
The role of cocaine in producing congenital malformations is controversial. Some studies suggest that congenital birth defects including those of face, nervous system and urinary system are possible. These problems, however, are very rare and somewhat negligible. Although cocaine is not associated with congenital anomalies, it has been shown, especially in late pregnancy, to be associated with increased risk of fetal intracranial hemorrhage with the worst possible outcome among the options.
The following features are definitely associated with maternal cocaine use during the pregnancy:
-IUGR
-Placental abruption
-Still birth
-Premature delivery
-Need for neonatal resuscitation -Developmental delay
-Intraventricular hemorrhage
OPTION A : The role of amphetamines in fetal malformations are controversial. While some studies have shown no teratogenic effects associated with amphetamines, other suggest that orofacial clefts are possible effects of amphetamine use during pregnancy; the incidence, however, has been exceedingly rare. Other complications of amphetamines during pregnancy include:
-Intrauterine growth restriction (IUGR)
-Preterm delivery
-Increased neonatal mortality and morbidity
-Hypertension and pre-eclampsia
OPTION B : Morphine or other opiates have a number of complications. One example is transitory CNS depression. This is not of major concern if promptly being taken care of.
OPTION D : Tricyclic antidepressants (e.g. amitriptyline) are not associated with congenital anomalies or concerning complications of the fetus or neonate and are rather safe for use during pregnancy.
OPTION E : Alcohol, if used in excess amounts can cause foetal alcohol syndrome which has developmental delays as part the complications spectrum; however, intracranial hemorrhage associated with cocaine use has a much worse outcome compared to alcohol.
TOPIC REVIEW
The following table lists the most important drugs associated with congenital malformations: (see photo)
1) An extremely rare congenital disorder presenting with limb malformation (dysmelia)
*http://www.merckmanuals.com/professional/gynecolog
* Llewellyn-Jones Fundamentals of Obstetrics and Gynaecology – Elsevier –Mosby - 9th Edition
A 22-year-old woman presents with complaint of amenorrhea of one year duration. Her menses were regular before. She has no other complaints or symptoms. Physical examination is inconclusive. Pelvis ultrasonography is unremarkable. Thyroid function tests show a TSH of 3.5mU/L (0.5-5mU’/L). Which one of the following is most likely to be the cause of her amenorrhea?
A. Mullerian agenesis.
B. Ovarian dysgenesis.
C. Hormonal dysfunction.
D. Hyperthyroidism.
E. Subclinical hyperthyroidism.
C. Hormonal dysfunction
Secondary amenorrhea is defined as cessation of periods after they have started. As opposed to primary amenorrhea is defined as lack of menstrual flow by age 14 and absence of secondary sexual characteristics or lack of menstrual flow by age 16 and presence of secondary sexual characteristics
Since this woman’s case is the cessation of previously present periods, she has secondary amenorrhea. Mullerian agenesis (option A) and ovarian dysgenesis (option B) result in primary amenorrhea. With previous menstruation, these two conditions are unlikely. Of the uterine causes, only Asherman’s syndrome can be the cause of secondary amenorrhea.
Hyperthyroidism can cause menstruation abnormalities, particularly amenorrhea or oligomenorrhea; however, a normal TSH excludes hyperthyroidism (option D) as the etiology of her amenorrhea. Subclinical hyperthyroidism is defined as a subnormal TSH while T3 and T4 are within normal range. This woman has a normal TSH level and does not have subclinical hyperthyroidism **(option E).
Of the options, the only explanation can be hormonal dysfunction. A hormonal dysfunction in this case could be due to conditions such as a pituitary tumor, hyperprolactinemia, ovarian insufficiency, or polycystic ovary syndrome (PCOS).
- UpToDate - Evaluation and management of secondary amenorrhea
A 37-year-old woman presents with secondary amenorrhea of 7 months duration. She has a body mass index (BMI) of 24. On laboratory studies, she has a FSH level of 55 U/L (2- 8U/L in luteal phase; >25U/L in menopause), LH of 54 U/L and estradiol the lower limit of normal. Serum prolactin level is also normal. A urine pregnancy test excludes pregnancy. On ultrasonography, each ovary contains 3-4 cysts. She desires to become pregnant in the future. Which one of the following would be the most appropriate management of her condition?
A. Progestin-only pills (POP).
B. Menopausal hormone replacement therapy (HRT).
C. Combined oral contraceptives (COCs).
D. Metformin.
E. Danazol.
B. Menopausal hormone replacement therapy (HRT)
In this woman, FSH and LH are within menopausal range (cut-off point> 40 U/L) and estradiol in the normal lower limit. Prolactin level is normal and excludes hyperprolactinaemia as a cause of her amenorrhea. The clinical picture and laboratory values are highly suggestive of ovarian failure.
Primary ovarian insufficiency (POI), commonly referred to as premature ovarian failure (POF), is defined as ovarian failure before 40 years of age. The incidence is 1 in 250 by age 35 and 1 in 100 by age 40.
In its full-blown presentation, ovarian failure is associated with amenorrhea, symptoms of estrogen deficiency, and increased gonadotropin levels (In the menopausal range) before the age of 40 years. The term ‘failure’ means that ovarian function is not normal, but it does not necessarily imply total cessation of ovarian function. Patients diagnosed with primary ovarian insufficiency may intermittently produce estrogen, ovulate or even conceive and have a normal pregnancy and delivery in 5-10% of cases.
Measuring serum FSH level is the core study to establish the diagnosis of POI/POF after pregnancy has been ruled out. By convention, two FSH levels in the menopausal range for the specific assay (>40 U/L by radioimmunoassay), measured at least 1 month apart, are diagnostic of POI/POF.
Measurement of serum LH is also important. In most cases of spontaneous POI/POF, FSH is higher than LH. If autoimmune oophoritis is present, FSH may be only mildly elevated, sometimes below the cutoff of 40 U/L, while LH is markedly elevated.
All women with POI/POF should be started on menopausal hormone replacement therapy (HRT) to prevent menopause complications and this should be continued at least until the average age of normal menopause (50-51 years). The main objective of early HRT is prevention of bone loss and early osteoporosis. Other measures to consider are supplementation of calcium and vitamin D.
Standard postmenopausal HRT does not provide effective contraception and spontaneous ovarian activity may resume; therefore, women with POI/POF, who require contraception, should use oral contraceptives for both hormone replacement and contraception.
This woman desires pregnancy; therefore, the most appropriate option for her is postmenopausal HRT. OCP would have been the correct answer if she also had asked for contraception.
- Medscape - Ovarian Insufficiency
- Clinical manifestations and diagnosis of spontaneous primary ovarian insufficiency (premature ovarian failure)
- Management of spontaneous primary ovarian insufficiency (premature ovarian failure)
Which one of the following foods should be avoided during pregnancy?
A. Liver.
B. Smoked salmon.
C. Beans.
D. Fruits.
E. Rice.
Smoked salmon
Liver, beans, fruits and rice can all be safely used during pregnancy. Salmon and other sea foods are generally safe to use, unless they are smoked and ready-to-use. Smoked salmon and trout are susceptible to colonization with Listeria monocytogenes even in the refrigeration temperature and should be avoided during pregnancy.
Soft cheese is another source of Listeria, unless it is cooked above 65°C and served hot.
*http://www.babycenter.com.au/c4209/food-and-drink-
*http://www.nutritionaustralia.org/sites/default/fi
*https://www.nhmrc.gov.au/_files_nhmrc/publications
Time spent:
Lisa is in your office at 8 weeks pregnancy for an antenatal visit. She asks you about foods that can be harmful to her pregnancy. Which one of the following she should avoid during pregnancy?
A. Soft cheese.
B. Tinned salmon.
C. Fish rich is iodine.
D. Lobster.
E. Liver.
Soft cheese
Of the options, soft cheese should be avoided due to the risk of listeriosis, unless it is thoroughly cooked and heated.
Listeria monocytogenes can cause invasive diseases including meningitis, meningoencephalitis, or bacteremia in immunosuppressed patients, individuals at the extremes of age including neonates and elderly adults, and pregnant women. Listeriosis can be associated with miscarriage and early neonatal sepsis.
Some foods, such as soft cheese, are more prone to contamination by Listeria, which can even grow at refrigeration temperature. Such foods should be avoided during pregnancy.
During pregnancy it is recommended that the following dairy products be avoided:
-Soft, semi-soft and surface ripened cheeses (e.g. brie, camembert, ricotta, feta and blue cheese)
-Soft serve ice cream
-Unpasteurized dairy foods - almost all dairy foods produced in Australia must be pasteurized, however some specialty imported cheeses may be unpasteurized.
NOTE - Soft cheese is safe if it is cooked above 65 ̊C Sand served hot e.g. ricotta and spinach cannelloni, cheese topping on pizza.
Ready-to-eat fish, such as smoked salmon or trout is also considered high-risk for Listeria contamination and should be avoided during pregnancy, but canned salmon or appropriately cooked salmon can be is used safely.
Liver, fish rich in iodine (if not smoked or ready-to-use), and lobster can be safely used if appropriately cooked and served.
*http://www.nutritionaustralia.org/sites/default/fi
*http://www.babycenter.com.au/c4209/food-and-drink
*https://www.nhmrc.gov.au/_files_nhmrc/publications
Kate is 22 years old and a patient in your clinic for the past 3 years. She was diagnosed with hypothyroidism 12 months ago, and was started on levothyroxine 100 mcg, daily. Today, she has presented with complaints of hot flushes and irregular periods for the past 4 months. Her periods were previously regular occurring every 28-30 days. Which one of the following hormonal assessments is more important to do next?
A. Estrogen.
B. Follicular stimulating hormone (FSH).
C. Luteinizing hormone (LH).
D. Thyroid stimulating hormone (TSH).
E. Prolactin.
D. Thyroid stimulating hormone (TSH)
The scenario represents a case of irregular periods and hot flushes. In approach to menstruation abnormalities, thyroid hormone problems should always be considered. In fact with Kate’s history, the diagnostic approach is completely clear. Kate is on thyroid hormones for treatment of hypothyroidism. Excess exogenous thyroid hormones either used for treatment can have led to hyperthyroidism, which in turn has resulted in menstruation abnormalities. The most common menstruation abnormalities associated with hyperthyroidism are oligomenorrhea and amenorrhea; however, menorrhagia is also possible.
Also, hyperthyroidism is a well-known cause of non-menopausal hot flushes. Hypertension is another cause.
Another possibility is that her hypothyroidism is still undertreated. Hypothyroidism, however, is most likely to present with menorrhagia. Additionally, the presence of hot flushes goes against hypothyroidism.
In this case, the next best step in approach to Kate’s problem, after taking a complete history, performing a thorough physical exam, and excluding pregnancy is to order a TSH.
Estrogen (option A), FSH (option B) , LH , and prolactin (option E) should be considered next if TSH is normal and thyroid problems are excluded because premature ovarian failure is another possibility to consider next.
- UpToDate - Overview of the clinical manifestations of hyperthyroidism in adults
Regarding oxytocin, which one of the following is true?
A. It has antidiuretic effect.
B. It is a steroid.
C. It is produced by the posterior pituitary.
D. It has an action on most involuntary muscles.
E. It causes milk production.
A. It has antidiuretic effect
Oxytocin has an antidiuretic effect through an unknown mechanism.
OPTION B and C : Oxytocin is a neurohypophysial peptide mostly produced in hypothalamus and stored in the posterior pituitary; however, recent studies showed that this hormone is also produced in uterus, placenta, amnion, corpus luteum, testis and heart.
OPTION D : The most prominent action of oxytocin is on smooth muscles of the uterus and breast. The most widely known effect of oxytocin on the uterine smooth muscles is uterine contraction. Its effect on smooth muscles of the breast is milk ejection during lactation. Oxytocin does not have an action on most of involuntary muscles such as those in digestive system or respiratory tract, or smooth muscles within the wall of vessels; at least not while within physiologic limits.
OPTION E : Although oxytocin is the main hormone in milk ejection process and milk-let-down reflex, it has no known role in milk production.
*http://www.ncbi.nlm.nih.gov/pmc/articles/PMC239673
*http://www.ncbi.nlm.nih.gov/pubmed/11274341
*http://physrev.physiology.org/content/81/2/629
Which one of the following is not an effect of oxytocin?
A. Mammalian glands stimulation.
B. Uterine stimulation.
C. Labor induction.
D. Progesterone release.
E. Antidiuretic effect.
D. Progesterone release
Oxytocin stimulates the smooth muscles of the uterus and breast. The most widely known effect of oxytocin on the uterine smooth muscles is uterine contraction and labor induction. Its effect on smooth muscles of the breast is milk ejection during lactation. It has no role in milk production.
Also, oxytocin has antidiuretic effect through an unknown mechanism.
Progesterone release is not an effect of oxytocin.
*http://www.ncbi.nlm.nih.gov/pmc/articles/PMC239673 *http://www.ncbi.nlm.nih.gov/pubmed/11274341
*http://physrev.physiology.org/content/81/2/629
A 17-year-old girl is brought to your practice for evaluation because her periods have not started yet. She also complains of abdominal pain every month and about the same time. On examination, she has fully developed breasts and axillary and pubic hair. She does not consent to a vaginal exam. Which one of the following could be the most likely diagnosis?
A. Imperforated hymen.
B. Absent vagina.
C. Mullerian agenesis.
D. Turner syndrome.
E. Normal variant.
A. Imperforated hymen
This girl has presented with primary amenorrhea as her chief complaint.
A girl is expected to have her periods until the age of 16 years. Primary amenorrhea is defined as absence of periods after the age 15-16 years in the presence of secondary sexual characteristics, or after the age of 13-14 years in the absence of such characteristics. The most common cause of primary amenorrhea is gonadal dysgenesis such as in Turner syndrome, followed by müllerian agenesis as the second most common cause. Patients with müllerian agenesis typically present with primary amenorrhea in the presence of normal growth and development in adolescence.
On physical examination, patients with müllerian agenesis have normal height, secondary sexual characteristics, body hair, and external genitalia, but a vagina is either absent, or present as a short blind-ended structure without a cervix at the vaginal apex. Patients with müllerian agenesis have a normal 46,XX karyotype and a normal hormonal profile. Most patients have a rudimentary nonfunctioning uterus.
In other words, cyclical endometrial shedding does not occur in most of women with müllerian agenesis; however, in 2 to 7% cases, there is a uterus with functioning endometrium. In such women, chronic or cyclic abdominopelvic pain occurs secondary to hematocolpos (accumulation of blood in the vagina), hematosalpinx (accumulation of blood in the ovarian tubes), hematometra (accumulation of blood in the uterus), or endometriosis.
Imperforated hymen can be another cause of primary amenorrhea. In contrast to most patients with müllerian agenesis, patients with an imperforate hymen will not have the typical fringe of hymenal tissue.
The patient with a low transverse vaginal septum will have a normal hymen with more proximal obstruction of the vaginal canal. In addition to presenting with primary amenorrhea, imperforated hymen and transverse vaginal septum are often associated with symptoms of cyclic abdominal or pelvic pain and a pelvic mass due to the obstructed outflow tract and associated hematocolpos.
With findings of fully developed sexual characteristics, absent periods, and cyclical pain, an imperforated hymen is the most likely diagnosis both clinically and statistically.
OPTION B and C : Absent vagina can be a presentation of a müllerian agenesis. Absent vagina /müllerian dysgenesis can present similar to imperforated hymen in 2 to 7% of cases in whom a functioning uterus exists. However, imperforated hymen is more likely statistically.
OPTION D : In Turner syndrome, usually no menstruation occurs. In fact, the gonads are dysplastic and malfunctioning (streak gonads). The patients often have secondary sexual characteristics (often delayed), but there is no cyclical abdominal pain because they do not menstruate. Short stature is commonly noted.
OPTION E : Absence of periods after the age of 16 years is always alarming and cannot be considered a normal variant.
- UpToDate - Evaluation and management of primary amenorrhea
A 53-year-old woman presents to your practice with vaginal bleeding after sexual intercourse 7 hours ago. She has not had any menses for the past 18 months. Her last cervical screening test, performed 12 months ago, was normal. She does not complain of any other symptoms and is otherwise healthy. Which one of the following could be the most likely underlying cause of postcoital bleeding in this woman?
A. Endometrial cancer.
B. Cervical cancer.
C. Cervical polyp.
D. Cervical ectropion.
E. Vaginal atrophy.
E. Vaginal atrophy
Postmenopausal vaginal bleeding, including post coital bleeding, can have many causes. Some menopausal women may experience ‘rouge ovulation’ which is an episode of ovulation and followed by menstruation after more than a year without episodes. Hormone replacement therapy (HRT) is another important cause of post-menopausal vaginal bleeding. Apart from those, vaginal bleeding in postmenopausal women should be taken very seriously and investigated promptly.
Fortunately, the most common causes of post-menopausal vaginal bleeding are benign and far less serious. Of these causes, atrophic vaginitis due to ongoing estrogen deficiency has been shown to be the most common cause of vaginal bleeding, especially post coital bleeding, in menopausal women.
It should be noted the although serious condition such as endometrial cancer or hypertrophy and cervical cancer are less common and comprise up to 20% of cases presenting with post-menopausal bleeding, these conditions should always considered first and excluded using prompt investigations.
Guidelines by the RACGP recommend that any post coital bleeding in a post-menopausal woman should be investigated with a co-test (HPV and LBC) and referral for gynecological assessment to rule out cervical cancer.
OPTION A : Although endometrial cancer can be another possibility, compared to atrophic vaginitis, it is not as common; nonetheless, assessment to exclude endometrial cancer should be undertaken with transvaginal ultrasound and endometrial biopsy if there is an abnormally thick endometrium.
OPTION B : With a normal pap smear 12 months ago, cervical cancer is very unlikely.
OPTION C and D : Cervical ectropion and cervical polyps can be other causes of postcoital bleeding. Cervical ectropion is more commonly a result of high estrogen levels such as during ovulatory phase in younger women, pregnancy and in those women on oral contraceptive pills. Cervical ectropion in postmenopausal women is not very common.
TOPIC REVIEW
Common causes of irregualr vaginal bleeding, including postcoital bleeding, are summarized in the following table: (see table)
- AAFP - Diagnosis and Treatment of Atrophic Vaginitis
Robert and Janet are in your clinic for consultation. Robert complains of malaise, mild headache and a generalized rash in form of both intact and unroofed vesicles, as well as macules and papules. Three weeks ago, Alan, their 7-year-old was diagnosed with chickenpox. Based on the history and the appearance of the rash, you establish the diagnosis of chickenpox for Robert. Janet is 13 weeks pregnant and is concerned if the infection can affect her and her baby too. Janet is not sure if she had chickenpox in childhood. Which one of the following is the most appropriate next step in management of Janet?
A. Start her on oral acicolvir.
B. Give her varicella zoster vaccine (Varivax).
C. Give her varicella zoster immunoglobulin (VZIG).
D. Arrange for checking her varicella zoster antibody (IgG) status.
E. Start her on oral famcilovir.
D. Arrange for checking her varicella zoster antibody (IgG) status
In approaching a pregnant woman, who has been exposed to varicella zoster virus (VZV) by contact to a case of chicken pox, the first question to be answered is whether the pregnant woman has had a history of chickenpox. If she remembers chickenpox infection, no further action is required. If she has had not the infection or the history is uncertain, the next step in management would be serologic tests for VZV (IgG). If the test is positive, the woman is immune. If not, management depends on the time elapsed since exposure:
- If the presentation is within the first 96 hours after exposure, passive immunization with VZV immunoglobulin (VZIG) is the most appropriate next step in management.
- If the presentation is after 96 hour, VZIG is of no use; however, the following women should still receive prophylaxis with the antiviral agents aciclovir (first line) or
valaciclovir:
*In the second half of pregnancy
*With a history of an underlying lung disease
*Immunocompromised
*Smokers
Since Janet is not certain about contracting chicken pox before and there is no history of vaccination against VZZ, the next best step for her would be checking her immunity status against VZV by VZV IgG (D is correct).
OPTION A : Oral aciclovir (or valaciclovir as a second option) is used for women with established VZV infection who present within the 24 hours after the rash appears or for those who present later and are at risk of complicated VZV infection indicated by: neurological symptoms, fever longer than 6 days, eruption of new pocks after 6 days, VZV pneumonia, or those who are immunocompromised.
OPTION B : Varivax contains live attenuated virus and is contraindicated throughout pregnancy.
OPTION C : VZIG would be the correct option if IgG levels show that Janet is not immune to VZV.
OPTION E : while there is adequate data that aciclovir and valaciclovir are safe to use during pregnancy, safety of famciclovir for use in a pregnant has yet to be established; hence, famciclovir is not as safe.
- Australian Society of Infectious Diseases - Management of Prenatal Infections
- South Australia Health Department - Varicella Zoster (chicken pox) in Pregnancy Clinical Guideline
A 35-year-old woman is brought to the emergency department after she sustaind ankle twisting. After initial management of her current problem, you realize that she is 10 weeks along her second pregnancy. On further questioning, she admits to heroin addiction. She also says that ‘Doc, I sometimes need to get high on meth. My favorite wings to fly are cocaine though, but since I cannot afford it, I a bit when I manage to crash a party’. She drinks a bottle of Whisky every day. During the past few weeks, she has started to worry about not being able to be a good mother. For this reason she is taking diazepam at night. She says that she managed to illegally get a bottle of these sleeping pills. Which one of the following in her history is most likely to cause fetal malformations?
A. Diazepam.
B. Alcohol.
C. Amphetamine.
D. Cocaine.
E. Heroine.
B. Alcohol
This woman is taking a bottle of Whisky every day. A small glass of Whisky (1.5 Oz) is equivalent of a standard drink. A bottle definitely exceeds 12 standard drinks; therefore this woman’s fetus would be at significant risk of fetal alcohol syndrome (FAS) which is associated with the following congenital malformations:
-Low-set ears
-Midfacial hypoplasia
-Elongated philtrum
-Upturned nose
-Microcephaly
-Skeletal and cardiac malformations
OPTION A : The health risks of benzodiazepines in pregnancy have not been clearly established. There have been inconsistent reports of teratogenic effects associated with fetal exposure to benzodiazepines. Regular benzodiazepine use in pregnancy may be associated with a neonatal abstinence syndrome, which may be of delayed onset..
OPTION C : Amphetamine use in controlled doses during pregnancy is unlikely to pose a substantial teratogenic risk, but women who use amphetamines are at higher risk of a range of obstetric complications such as reduced birth weight — many of these outcomes are not specific to amphetamines but influenced by other drug use and lifestyle factors in addition to amphetamine use. Exposure to amphetamines in utero may influence prenatal brain development, but the nature of this influence and potential clinical significance are not well established.
3,4- methylenedioxymethamphetamine (MDMA), commonly known as ecstasy, is an amphetamine derivative. Existing evidence suggests that use of ecstasyduring first trimester poses a potential teratogenic risk; therefore, it is strongly recommended that use of ecstasy be avoided during this dangerous period of organogenesis (between week two and week eight post conception or between week four and ten of last menstrual period)
OPTION D : The role of cocaine in congenital malformations is controversial, and the malformations caused by cocaine are extremely rare. However, if it leads to fetal intracranial haemorrhage the outcome would be devastating.
OPTION E : Opiate addiction carries the significant risk of several perinatal complications, but there is no associated congenital malformation.
- Llewellyn-Jones Fundamentals of Obstetrics and Gynaecology – Elsevier –Mosby - 9th Edition
*http://www.merckmanuals.com/professional/gynecol
A 54-year-old woman presents to your clinic, with complaint of a vulvar lesion associate with severe itching. Her problem started nearly 6 months ago and has progressed in severity. She has been menopausal since the age 50 years. On examination, atrophic white plaques are evident as well as excoriation marks over and around the vulva. The labia minora is slightly atrophic and there is interoitus stenosis. Vaginal examination is normal. The appearance of the vulva is shown in the accompanying photograph. Which one of the following is the most appropriate option for treatment of this patient?
A. Vaginal estrogen cream.
B. Clobetasol cream.
C. Hydrocortisone cream.
D. Topical antifungals.
E. Oral prednisone.
B. Clobetasol cream
The clinical findings of pruritic white vulvar plaques, in the absence of vaginal symptoms and signs, make genital lichen sclerosus (LS) the most likely diagnosis.
LS is a chronic inflammatory dermatosis of unknown etiology. The disease most commonly affects the genitalia, but it can occur at any skin site (extragenital lichen sclerosus). Balanitis xerotica obliterans is an outdated termed used to describe the LS of glans penis. Lichen sclerosus can occur in males or females of any age. In female, however, there are two peaks of onset during prepubertal and perimenopausal/postmenopausal ages, both of which are estrogen-deficient states; however, the relationship between LS and estrogen deficiency is unclear. In males it is more common between the puberty and age 60 years.
Typical clinical presentation is with white, often atrophic, plaques associated with pruritus and pain that result in genital scarring and adhesions. The most common complaint in vulvar lichen sclerosus is progressive pruritus that can be followed by dyspareunia, dysuria, or genital bleeding. Female genital lesions may be confined to the labia majora but usually involve, and eventually obliterate the labia minora and stenose the interoitus. Often, an hourglass, butterfly, or figure-8 pattern involves the perivaginal and perianal areas, with minimal involvement of the perineum in between.
Penile LS is associated with sexual/urinary dysfunction. It is usually preceded by pruritus but may present with sudden phimosis of previously retractable foreskin, and urinary obstruction can result.
Extragenital lichen sclerosus (LS) may be asymptomatic (approximately one third of cases) or it may itch or be tender.
NOTE - With genital LS in women, there is an increased risk of squamous cell carcinoma (<5%). Progression to malignancy in genital LS in men is rare.
First-line therapy for LS includes patient education and super-potent topical corticosteroids (e.g. clobetasol propionate). Prolonged use of potent corticosteroids does not result in atrophy of the labia minora and clitoris, owing to the resistant nature of their modified mucous membranes. However, perianal and hair-baring areas (e.g. labia majora) can atrophy within 2-3 weeks of use. Intralesional corticosteroid injections are also considered at occasions.
Second-line therapies include the calcineurin inhibitors, tacrolimus and pimecrolimus, which can be a helpful adjunct to topical corticosteroids for maintenance.
Third-line therapies that could be considered in treatment-resistant genital lichen sclerosus could include topical or oral retinoid, steroid injections, systemic cyclosporine (topical use has not shown effective), methotrexate, or hydroxyurea. For extragenital lichen sclerosus, phototherapy or methotrexate could be considered.
OPTION A : Vaginal estrogen cream is the first-line treatment option for women with atrophic vaginitis and not effective against LS. Normal vaginal exam excludes atrophic vaginitis as the diagnosis.
Although extensively used in the past, topical estrogen, topical testosterone, topical progesterone, and hormone replacement therapy are not recommended anymore due to lack evidence for effectiveness.
OPTION C : Hydrocortisone is a weak corticosteroid and not appropriate for treatment of LS where potent to very potent (super potent) topical corticosteroids are indicated.
OPTION D : Topical antifungal agents were the correct answer if the diagnosis would have been vulvovaginal candidiasis. Candidiasis is associated with inflamed skin and mucosa and cheesy- white vaginal discharge that are not present in this woman.
OPTION E : Oral (systemic) predniso(lo)ne is not used for treatment of LS due to their adverse effect profile that is not outweighed by the benefits.
- Medscape - Lichen Sclerosus
- Localized provoked vestibulodynia (vulvodynia): assessment and management
A 39-week pregnant woman is admitted to the Labor Unit due to commencement of labor pain and contractions. The woman is placed in left lateral decubitus position and started on supplemental oxygen by face mask. An intravenous line is established and isotonic fluids and cyntocinon drip is started according to the protocol. A CTG is performed that shows a fetal heart rate of 140 which drops to 70 bpm and returns back to the base line of 140 bpm in two minutes. Which one of the following is the most appropriate immediate management?
A. Stop cyntocinon.
B. Fetal scalp blood sampling.
C. Immediate cesarean section.
D. Titrate and increase the dose of cyntocinon.
E. Continuous CTG monitoring.
Stop cyntocinon
The scenario represents a case of prolonged fetal heart rate deceleration during labor. Prolonged deceleration is defined as a drop in the fetal heart rate below the baseline that lasts from 90 seconds up to five minutes. Prolonged bradycardia is defined as a fetal heart rate <100 bpm lasting more than five minutes.
The following are the possible causes of prolonged deceleration or prolonged bradycardia:
* Maternal hypotension
* Cord prolapse or compression
* Uterine hypertonia
* Scar dehiscence
* Placental abruption
* Rapid fetal descent
Management of prolonged decelerations or prolonged bradycardia includes the following steps:
-Reposition the woman – e.g. lateral position
-Checking maternal blood pressure and administration of bolus IV fluids in case of hypotension.
-Discontinuation of oxytocin or decreasing rate of infusion (if in progress)
-Check the maternal blood pressure.
-Checking the maternal pulse to differentiate maternal pulse rate from the fetal heart rate (FHR).
-If indicated, performing a vaginal exam to exclude cord prolapse or rapid cervical dilatation if the bradycardia persists.
-Consideration of applying a fetal scalp electrode.
-Assessment of abdominal tone to exclude a hypertonic uterus.
-Preparation e for assisted delivery (vacuum or forceps delivery) or emergency cesarean section if bradycardia does not resolve.
It is very likely that prolonged decelerations have been caused by excess uterine contraction as a result of oxytocin (cyntocinon) infusion; therefore, cessation of cyntocinon is the most important immediate action to consider. CTG monitoring option E should also be continued throughout the labor for assessment of fetal status.
OPTION B : Fetal scalp blood sampling is considered if measures such as repositioning the woman, cessation of cyntocinon, blood pressure correction or other measures fail to correct the situation. This will determine if there is fetal jeopardy necessitating immediate delivery.
OPTION C : Urgent cesarean section or assisted vaginal delivery may be required if fetal compromise persists despite more conservative measures.
Increasing the dose of cyntocinon , by increasing the contractions, will escalate the fetal condition and is an incorrect option.
- WA Health Department – Obstetrics and Gynaecology Guidelines
- RANZCOG Intrapartum Fetal Surveillance Clinical Guideline
A 38-year-old pregnant woman, G1P0, is brought to the emergency department after spontaneous rupture of membranes. She is admitted to the labor unit. After three hours, there is still no uterine contractions present; hence, she is started on cyntocinon. After four hours, reassessment shows that there are adequate uterine contractions. A CTG shows a base line fetal heart rate of 143 bpm dropping to 60 bpm and slowly returning to the baseline in four minutes. On vaginal exam, the fetal head is in LOA position, the station is 1+ and there is 1+ molding. The cervix is dilated at 9 cm. Which one of the following is the most appropriate next step in management?
A. Forceps delivery.
B. Vacuum delivery.
C. Addition of one liter of normal saline.
D. Cessation of cyntocinon.
E. Emergent cesarean delivery.
D. Cessation of cyntocinon
With adequate uterine contractions, head engagement and appropriate descent, the labor is progressing appropriately; however, with FHR dropping to 60 bpm (<100 bpm) there is prolonged decelerations that make the CTG abnormal.
CTG is considered abnormal if there are at least two of the following features on CTG:
* Baseline FHR is between 100-109 bpm or between 161-170 bpm
* Variability of FHR is reduced (3-5 bpm for >40 minutes)
* Decelerations are variable without complicating features
OR
There is any of the following:
* Baseline FHR is <100 bpm or >170bpm
* Variability is absent (<3 bpm)
* Decelerations are prolonged for >3 minutes OR late OR have complicated variables
When there is CTG abnormalities, the first steps to consider are reversing the possible causes of decelerations :
* Maternal hypotension
* Cord prolapse or compression
* Uterine hypertonia
* Scar dehiscence
* Placental abruption
* Rapid fetal descent
Management of prolonged decelerations or prolonged bradycardia includes the following steps:
-Reposition the woman – e.g. lateral position
-Checking maternal blood pressure and administration of bolus IV fluids in case of hypotension.
-Discontinuation of oxytocin or decreasing rate of infusion (if in progress)
-Check the maternal blood pressure.
-Checking the maternal pulse to differentiate maternal pulse rate from the fetal heart rate (FHR).
-If indicated, performing a vaginal exam to exclude cord prolapse or rapid cervical dilatation if the bradycardia persists.
-Consideration of applying a fetal scalp electrode.
-Assessment of abdominal tone to exclude a hypertonic uterus.
-Preparation e for assisted delivery (vacuum or forceps delivery) or emergency cesarean section if bradycardia does not resolve.
Uterine hyperstimulation and contractions due to excess cyntocinon is a common cause of prolonged deceleration; therefore, cessation of cyntocinon is the most appropriate immediate step once prolonged decelerations are encountered on CTG. It is very likely that prolonged decelerations have been caused by excess uterine contraction as a result of oxytocin (cyntocinon) infusion; therefore, cessation of cyntocinon is the most important immediate action to consider.
It is also important to check the maternal blood pressure as the mother’s hypotension is also a cause of prolonged deceleration. In case of hypotension, intravenous fluids should be administered (option C) to correct maternal hypotension. CTG monitoring should be continued for assessment of fetal status and response to treatment.
Assisted vaginal delivery using forceps or vacuum (option A and B) or emergent cesarean delivery (option E) are measures to consider if the above conservative steps fail to correct the underlying cause(s) of fetal compromise.
* RANZCOG Intrapartum Fetal Surveillance Clinical Guideline * WA Health Department – Obstetrics and Gynaecology Guidelines
A 24-year-old woman presents to a GP clinic with concerns about breastfeeding. She gave birth to a healthy male child 20 days ago through an uneventful vaginal delivery. She is worried that she is not producing milk to feed her baby and feels he is always hungry. Which one of the following conditions could be the most likely cause of decreased mild production?
A. Poor positioning of the baby.
B. Exhaustion of the mother.
C. Short duration of feeding at each session.
D. Decreased frequency of breastfeeding.
E. Insufficient fluid intake by the mother.
D. Decreased frequency of breastfeeding
Although most women are capable of producing more milk than their infants require, more than half of breastfeeding mothers perceive that their milk supply as inadequate. Results of an online survey of 12 World Health Organization Western Pacific offices revealed that the second most commonly reported reason women did not breastfeed was “not having enough milk” (17%).
For some mothers, this is a perceived rather than real issue. In such cases, providing information on what constitutes ‘normal’ newborn feeding behavior may provide reassurance. For example, it is normal for infants to have frequent but small amount feeds during a particular time of the day. A change to an infant’s feeding pattern, such as increased frequency of feeds, or a feeling of softer breasts, may concern the mother; however, these may be normal changes that are unrelated to decreased supply.
Of the causes that lead to genuine lowered milk production, decreased frequency of breastfeeding is the most common one. In the human mammary gland, lactation is under autocrine control, in which the frequency and degree of milk removal by expression regulate an inhibitory peptide present in the milk. This means that if the milk is not removed, this inhibitory peptide accumulates and subsequently decreases the synthesis of milk. If the milk is frequently removed, this inhibitory peptide does not accumulate, and milk synthesis increases.
*https://www.racgp.org.au/afp/2016/august/overcoming-challenges-faced-by-breastfeeding-mothers/
* https://emedicine.medscape.com/article/979458-overview#a4
Which one of the following women is most likely to develop pre-eclampsia during pregnancy?
A. A 40-year-old woman, gravida 6, para 5, with no previous history of any pregnancy-related complications.
B. A 16-year-old primigravida woman.
C. A 35-year-old woman with primary hypertension.
D. A 25-year-old woman who is gravida 4, para 3.
E. A 25-year-old primigarvida with family history of pre-eclampsia in her mother and sister.
C. A 35-year-old woman with primary hypertension
Pre-eclampsia is defined as hypertension and proteinuria after 20 weeks of pregnancy.
The following table summarizes the established risk factors for development of pre-eclampsia and their associated increased risks:
(See table below)
According to the above table, a woman with chronic hypertension is five time more likely to develop preeclampsia compared to other options.
OPTION A : In a 40-year-old woman with no previous obstetrical history, the risk of preeclampsia is almost doubled.
OPTION B : Nulliparity almost triples the risk of preeclampsia. Age 16 years or younger has been considered as a risk factor but recent studies failed to establish a meaningful relationship.
OPTION D : This woman has no known risk factor for preeclampsia.
OPTION E : In this woman, the family history of preeclampsia in her mother and sister, triples the risk of preeclampsia for her.
In the event of chord prolapse during labor, which one of the following findings is more likely on CTG monitoring?
A. Sinusoidal pattern.
B. Early decelerations.
C. Variable decelerations.
D. Late decelerations.
E. Fetal tachycardia.
C. Variable decelerations
The most common CTG abnormality associated with umbilical cord prolapse is variable decelerations. Other less common CTG findings include persistent fetal bradycardia and prolonged decelerations of over one minute.
- RCOG - Greentop Guidelines - Umbilical Cord Prolpase
Dorothy, 21 years of age, presents to your GP clinic for advice regarding commencement of combined oral contraceptive pills (COCPs). You take a careful medical and family history, which is significant for gastro-esophageal reflux disease (GERD) and premenstrual headaches. She has smoked 15-20 cigarettes a day for the past four years and drinks 2 to 3 glasses of wine every day. Her family history includes breast cancer of her mother diagnosed at the age of 53 years and hypertension in her father. On examination, her blood pressure is 130/78 mmHg. She has a body mass index (BMI) of 28. She wants to know if there is anything wrong with her taking OCPs. In counselling her, which one of the following will you discuss further as a possible absolute contraindication to COCPs?
A. Her drinking history.
B. Her family history of breast cancer.
C. Her premenstrual headaches.
D. Her BMI.
E. Her smoking history.
C. Her premenstrual headaches
According to the UK Medical eligibility criteria for contraceptive use, the following are considered absolute risk factors for COCPs:
- Breastfeeding and ≤6 weeks postpartum
- Smoker ≥35 year and ≥ 15 cigarettes/day
- Presence of multiple risk factors for CVD including older age, smoking, diabetes, hypertension
- Hypertension with systolic ≥160mmHg or diastolic ≥95mmHg
- Vascular disease
- Major surgery with prolonged immobilization
- Current or past history of venous thromboembolism (VTE)
- Known thrombogenic mutations (Factor V Leiden, Prothrombin mutation, Protein S, Protein C and Antithrombin deficiencies)
- Migraine with aura
- Current or past history of Ischemic Heart Disease (IHD);
- Complicated valvular heart disease
- Diabetes complicated by nephropathy, retinopathy or vascular disease
- Breast cancer
- Severe Liver disease including cirrhosis hepatocellular adenoma and hepatoma
- Raynaud’s with lupus anticoagulant
- SLE with antiphospholipid antibodies
In the history, Dorothy has mentioned headaches before her menses. The relation of the headaches to her periods makes migraine a possibility, as one of the triggers for migraine is hormonal changes in the premenstrual period. If that is the case, further scrutinizing the history is required for characteristic features of aura. If her headaches are associated with aura, COCPs will be absolutely contraindicated for her.
OPTION A : Alcohol use is not a contraindication to COCPs use.
OPTION B : While personal history of breast cancer is an absolute contraindication to COCPs, family history of breast cancer is not associated with increased risk of breast cancer in COCPs users; hence, the history of breast cancer in Dorothy’ mother does not preclude use of COCPs for her.
OPTION D : Overweightness and obesity alone are not contraindications to use of combines oral contraceptive pills, unless associated with other cardiovacular risk factors such as smoking, diabetes, etc.
OPTION E : COCPs are contraindicated in woman who are 35 years or older AND smoke ≥ 15 cigarettes/day. Although Dorothy smokes 15-20 cigarettes a day, she is 21 years; therefore, her smoking history is not an absolute contraindication.
- RANZCOG - Combined Hormonal Contraceptives
A 55-year-old woman presents to your GP practice with complaints of dyspareunia, dysuria, and vaginal itch. She has been menopausal since the age of 50 years. She relates that sexual intercourse has been becoming progressively painful for the past 6 months. She describes the pain as burning. The vaginal itching and dysuria has developed in the past few weeks. Her past medical history is unremarkable, and she is currently on no medications except for supplemental vitamins. She denies any hot flushes. On examination, atrophy of the labia minora is evident, but labia majora and other parts of the vulva are normal. Vaginal exam reveals a rather stenotic vagina with pale and atrophic walls but no vaginal or cervical inflammation or discharge. Which one of the following would be the most appropriate first-line treatment for her?
A. Estrogen cream.
B. Topical antifungal agents.
C. Conization.
D. Hydrocortisone cream.
E. Topical antibiotics.
A. Estrogen cream
This woman has atrophic changes of the vagina 5 years after menopause. For her, atrophic vaginitis is the most likely diagnosis.
Atrophic vaginitis results from estrogen deficiency and is experienced by almost 50% of postmenopausal women. Symptoms include itching, burning, dryness, and irritation, all of which can lead to dyspareunia. On the other hand, a decline in estrogen alters the vaginal flora, leading to bacterial overgrowth. This can cause bacterial vaginosis and vaginal discharge.
Declining estrogen also affects the urinary tract, leading to thinning of the bladder and urethral linings. This can result in chronic dysuria and an increased incidence of urinary tract infections. Estrogen creams are the mainstay of treatment for atrophic vaginitis. As this may take a while to come to effect, lubricants can be used to facilitate sexual intercourse.
OPTION B : Antifungal agents were the treatment of choice if vaginal candidiasis would have been the diagnosis. The condition most commonly presents with itching. Burning and dyspareunia can also be present; however, an inflamed vaginal with or without cheesy white discharge rather than an atrophic vagina would have been expected on examination.
OPTION C : Conization has no role in treatment of atrophic vaginitis. It is often used to treat premalignant and malignant cervical lesions.
OPTION D : Potent to very potent topical corticosteroids are used, as first-line therapy, for treatment of lichen sclerosus (LS). This woman is not likely to have LS, as LS does not involve vagina. On the other hand, hydrocortisone is a weak corticosteroid and not effective in treatment of LS.
OPTION E : Topical antibiotics (e.g., metronidazol) may be used for concomitant bacterial infection or vaginosis in atrophic vaginitis, but it is not effective for atrophic vaginitis itself.
- MJA – Management of common vulval conditions
- RACGP - Localized provoked vestibulodynia (vulvodynia): assessment and management
- Medscape – Treating Atrophic Vaginitis
Hanna is a 32-year-old patient of yours, who have decided to start combined oral contraceptive pills (COC) for contraception, but she is worried about the risk of cancer because she has heard from her friends and read on different websites that COC increase the risk of some cancers. In consulting her regarding long-term use COC, which one of the cancers will you mention to be of highest risk for her if she takes COC in the long run?
A. Endometrial cancer.
B. Colon cancer.
C. Breast cancer.
D. Cervical cancer.
E. Ovarian cancer.
D. Cervical cancer
COCs cease ovulation and is associated with decreased risk of ovarian cancer in women who have never used COCs. This protective effect is increased with the length of time COCs are used, and last up to 30 years after cessation. This effect is caused by reducing the number of ovulations a woman experiences in her lifetime, and consequently reducing exposure to naturally occurring female hormones as a risk factor.
The risk of colorectal cancers is also decreased by 15% to 20% in women who have ever taken COCs compared to women never on COC. This effect is caused by reduction in bile acids in women on COCs.
Another cancer with **decreased risk associated with COCs use is endometrial cancer **. There is at least 30% reduction in risk of developing endometrial cancer in women who have ever used COCs compared to those who have not. The risk reduces more with prolonged use of COCs. This reduction is even more significant in long-time users of oral COCs who are smokers, obese, or physically inactive. Such protective effect is caused by suppression of endometrial proliferation as a result of naturally occurring sex hormones when a woman uses COCs (synthetic sex hormones).
COCs, however, are associated with an increased risk of cervical cancer. Women who have used oral COCs for 5 or more years have a higher risk of cervical cancer than women who have never used COCs. The longer a woman uses COCs, the greater the increase in her risk of cervical cancer will be. One study found a 10% increased risk for less than 5 years of use, a 60% increased risk with 5–9 years of use, and a doubling of the risk with 10 or more years of use. This risk declines over time after cessation of COCs. Such effect is probably due to changes in the susceptibility of cervical cells to persistent infection with high-risk HPV types which are virtually the cause of all cervical cancers. Another hypothesis is that women on COCs have more frequent unprotected sex which increases the risk of HPV infection.
COCs are also associated with a slight increase in risk of breast cancer compared to women who have never taken COCs. The risk declines after cessation, and after 10 years of cessation, will be the same as that of women who have never used COCs. Generally, the benefits of preventing unwanted pregnancy and its harms outweighs the slightly increased risk of breast cancer associated with COCs.
In consulting Hanna, she should be told the main concern would be the increased risk of cervical cancer if COCs are taken longer than 5 years as the main concern, and the fact that there is a just slightly increased risk of breast cancer according to studies. COCs would not be appropriate for her in the presence of any personal history of breast cancer but even with breast cancer history in her first degree relatives, she can still take COCs.
It should be explained to her than in fact OCPs decrease her risk of developing ovarian, endometrial and colon cancers.
- National Cancer Institute – Oral contraceptives and cancer risk
- Cancer Council - Oral contraceptives
Miranda is a 33-year-old patient of your, who has presented for cervical screening test today. Her previous tests were all normal. She is married, has never been pregnant, and started her sexual relationships from the age of 17 years. She has been using oral contraceptive pills (OCPs) for the past 8 years as the means of contraception. She does not smoke but
drinks alcohol on social occasions. She is obese with a BMI of 31 kg/m2. In consulting her, which one of the following will you mention as the most significant risk factor for development of cervical cancer in the future?
A. Sexual relationship.
B. Alcohol.
C. Obesity.
D. Prolonged use of OCPs.
E. Nulliparity.
D. Prolonged use of OCPs
Some conditions are associated with increased risk of developing cervical cancer. Of these, the most important one is infection with high risk human papilloma virus (HPV) types, such as HPV type 16 and 18. Virtually, all the cases of cervical cancers are caused by HPV infection. In the absence of HPV infection, no cervical cancer occurs. Other risk factors predispose to the development of cervical cancer only in the presence of HPV infection.
All women, who have ever been sexually active, are at an average risk of developing cervical cancer through receiving the HPV infection either by sexual intercourse or skin to skin contact. However, sexual relationship without contracting HPV infection is not a risk factor for cervical cancer.
OCP use in another important risk factor for cervical cancer. Women, who have used oral OCPs for 5 years or more \, have a higher risk of cervical cancer than women who have never used OCPs.
The longer a woman uses oral contraceptives, the greater the increase in her risk of cervical cancer. One study found a 10% increased risk for less than 5 years of use, a 60% increased risk with 5–9 years of use, and a doubling of the risk with 10 or more years of use. Such effect is probably due to changes in the susceptibility of cervical cells to persistent infection with high-risk HPV types. Another hypothesis is that women on OCPs have more frequent unprotected sex, with consequent increased the risk of HPV infection. This should be mentioned to Miranda as the most significant risk factor among other options.
Alcohol (option B) have not shown to be associated with an increased risk of cervical cancer. Although obesity and overweightness have been implicated for increasing the risk of cervical cancer, such association is not significant.
Nulliparity (option E) is a protective factor rather than a risk factor. Studies suggest that giving birth to 5 or more children may slightly increase the cervical cancer for women who have HPV infection.
*TOPIC REVIEW
*
Important risk factor for cervical cancer are as follows:
*Persistent infection with high-risk HPV types (the most significant risk factor – without HPV infection, there is no risk of cervical cancer even in the presence of multiple other factors)
*Smoking
*Lack of regular cervical screening tests
*Age – cervical cancer risk increases after the age of 35 years Prolonged use of OCPs (more than 5 years) *Immunosuppression
*Previous screening abnormalities or previous cervical cancer
*Multiparity (5 or more)
*Exposure to diethylstilbestrol (DES)
- RACGP – Guidelines for preventive activities in general practice (The Red Book) – cervical cancer
- Cancer Australia – Cervical cancer: what are the risk factors for cervical cancer?
A 27-year-old woman comes to antenatal clinic at 10 weeks gestation. It is her first pregnancy. She smokes 2 packs of cigarettes a day and drinks 5-7 standard drinks every day. Recently, she has started using crack cocaine and heroin. She mentions that she is going through a lot these days because her employer found out about her addiction and fired her. She is very anxious and worried about her future and cannot sleep well. This is why she uses diazepam every night. Which one of the following in the history may have the worst effect on the fetus?
A. Cocaine.
B. Heroin.
C. Cigarette smoking.
D. Diazepam.
E. Alcohol.
A. Cocaine
Although cocaine is not associated with congenital anomalies, its use is associated with increased risk of fetal intracranial hemorrhage with the worst possible outcome if it occurs.
OPTION B : Heroine and other opiates can be associated with the following complications:
- Low birth weight usually associate with intrauterine growth restriction(IUGR)
- Preterm labor
- Drug withdrawal for the neonate (neonate abstinence syndrome)
- Increased risk of sudden infant death syndrome (SIDS)
OPTION C : Smoking is associated with multiple complications usch as IUGR, preterm labor, limb reduction, and GI malformations; however, none of them is as life-threatening as fetal intracranial hemorrahge should it occur.
OPTION D : The health risks of benzodiazepines in pregnancy have not been clearly established. There have been inconsistent reports of teratogenic effects associated with fetal exposure to benzodiazepines. Regular benzodiazepine use in pregnancy may be associated with a neonatal abstinence syndrome, which may be of delayed onset.
OPTION E : Although alcohol use is associated with fetal alcohol syndrome (FAS), the amount needed for occurrence of FAS often has to be more than 12 standard drinks (120gr) per day. With only 5 to 7 standard drinks per day, the likelihood of FAS is insignificant.
NOTE - There is a long list of therapeutic medications and recreational drugs that can cause neonatal malformations. Malformations are caused by teratogenic effects of these drugs. Bear in mind that malformations are different from perinatal complications. For example, marijuana can be associated with several perinatal risks, but there are no associated congenital malformations. Alcohol, on the other hand, not only causes complications such as intrauterine growth restriction (IUGR), but it use can also give rise to neonatal alcohol syndrome (FAS), characterized by congenital malformations such as midfacial hypoplasia, upturned nose, long philtrum and low-set ears.
- Llewellyn – Jones Fundamentals of Obstetrics and Gynaecology Elsevier – Mosby 9th Editionhttp://www.merckmanuals.com/professional/gynecology_and_obstetrics/drugs_in_pregnancy/drugs_in_pregnancy.html
- http://www.sahealth.sa.gov.au/wps/wcm/connect/fad9
A couple present to your clinic for evaluation of infertility because they have failed to conceive after one year of unprotected sexual intercourse in a timely fashion at mid-cycles. The female is 45 years old and has three children from her former spouse. She has regular periods at 30-day intervals. The male is 50 years old and has never fathered a child. They have been married for five years now. Which one of the following could be the most likely cause of the infertility?
A. Maternal age.
B. Paternal age.
C. Semen abnormalities.
D. Anovulatory cycles.
E. Pelvic adhesions from previous pregnancies.
C. Semen abnormalities
Infertility is defined by the World Health Organisation (WHO) as the inability of a couple to conceive after 12 months of regular unprotected intercourse in women less than 35 years of age; and after six months of regular unprotected intercourse in women 35 years of age or older; or the inability to carry pregnancies to live birth. Based on this definition, this couple are infertile.
Infertility can be caused by a male factor, female factor, or both. Causes of infertility can be categorized as follows:
Female infertility factors:
* Hormonal disorders
* Damaged or blocked fallopian tubes
* Endometriosis
* Excessively thick cervical mucus
Male infertility factors:
*Semen abnormalities (volume, PH, count, morphology, motility, vitality, etc.)
*Any obstruction in the course of sperms out and/or ejaculation failure.
In general, advances maternal age is the most important factor influencing infertility. This is followed by male infertility as the second most important factor.
In this case, the female partner has already given birth to three children via vaginal delivery. There has been no risk factor in the history, such as caesarean section or instrumentation, to make pelvic adhesions from previous pregnancies (option E) a likely explanation to the infertility.
She is 45 years old and, as mentioned before, advanced maternal age (option A) has the most important role in infertility due to factors such as poor follicle reserve and anovulatory cycles, decreased quality of the egg, and undesirable endometrium for the fertile egg to implant. This woman has regular periods at 30-day intervals indicating that she very likely to be still ovulating because anovulatory cycles are associated with irregular periods as the most common symptom.
Although having ovulatory cycles does not exclude problems such as poor-quality eggs or endometrium for fertility due to advanced age, the fact that the male partner has never fathered a child makes male-infertility, and more specifically semen abnormalities, the most likely explanation for infertility in this couple.
Paternal age (option B) is a risk factor for semen abnormalities, but again since he has never fathered a child before (even at younger ages), semen abnormalities independent of the paternal age could be a better and more likely explanation.
For this couple, the first investigation to consider should be a semen analysis. Once male infertility, as the most likely explanation in this scenario, is excluded, attention should be turned to female factors.
- RACGP – AFP – We’re having trouble conceiving
Mary, 27 years old, is admitted to the Maternity Ward after her labor pain started. After amniotomy, she is placed in the left lateral position and on supplemental oxygen by nasal canula, and intravenous fluids and Syntocinon® (oxytocin) infusion is started. A while later and during fetal heart auscultation, fetal heart rate (FHR) of 70 bpm is noted. CTG is applied which reveals a baseline fetal heart rate of 140 bpm dropping to 70 bpm periodically with each episode of bradycardia lasting approximately three minutes. Which one of the following is the most appropriate next step in management?
A. Fetal scalp blood sampling.
B. Stop Syntocinon.
C. Immediate cesarean delivery.
D. Continuous CTG monitoring.
E. Reposition to supine.
B. Stop Syntocinon
The FHR pattern on CTG is typically interpreted as reassuring or non-reassuring. The presence of a reassuring pattern indicates that there is a minimal possibility of fetal acidemia at that point in time. It does not predict the future fetal status because tracing pattern can change.
A CTG pattern is reassuring if all of the following are present:
- A baseline fetal heart rate of 110 to 160 bpm
- Absence of late or variable FHR decelerations
- Moderate FHR variability (6 to 25 bpm)
- Age-appropriate FHR accelerations
Any other CTG pattern except the above is considered non-reassuring and prompt action should be taken. When a non-reassuring CTG is encountered, management includes the following steps:
- Calling for assistance
- Administration of oxygen via a tight-fitting face mask
- Changing the maternal position to left lateral or knee chest (to remove the pressure of the gravid uterus from the inferior vena cava and improve maternal circulation and consequently placental circulation)
- Administration of fluids (bolus)
- Discontinuation of any uterine stimulant (e.g. oxytocin)
- Continuous CTG monitoring
- Performing a vaginal examination (to exclude cord prolapse) and fetal scalp stimulation
- Determination and correction of the cause of the non-reassuring CTG pattern if possible
- Considering tocolysis if indicated
- Determining whether operative intervention (e.g., cesarean delivery) is warranted and, if so, how urgently it is needed
In this scenario, the presence of fetal bradycardia (FHR<110 bpm) makes the CTG non-reassuring and expeditious action and investigation (as mentioned above) is warranted. Mary has already been placed in the left lateral position and is receiving oxygen and intravenous fluids. Of the options, cessation of oxytocin infusion is the most appropriate next step in management because it is likely that oxytocin has led to uterine hyperstimulation. Uterine hyperstimulation is defined as tachysystole (more than five active labor contractions in 10 minute) or uterine hypertonus (contractions lasting more than two minutes or contractions occurring within 60 seconds of each other) in the presence of fetal heart rate abnormalities.
OPTION A : Fetal scalp blood sampling is indicated to assess the presence and severity of fetal acidosis if CTG remains non-reassuring despite initial measures.
OPTION C : Immediate cesarean delivery is considered if there are persisting CTG abnormalities indicative of fetal jeopardy. Unless conditions dictate otherwise, a fetal scalp blood sample is performed to evaluate fetal acidemia prior to making such decision.
OPTION D : Continuous CTG monitoring is indicated in all cases of non-reassuring CTGs after initial measures are undertaken. CTG monitoring is a very important step in this case, but does not take precedence over cessation of oxytocin.
OPTION E : Placing Mary in supine position will cause the pregnant uterus to compress the inferior vena cava and reduce the venous return which will lead to decreased maternal cardiac output. This will result in decreased blood flow to the placenta and deteriorate the fetus’s condition.
- RANZCOG Intrapartum Fetal Surveillance Clinical Guideline
A 25-year-old woman presents at 20 weeks gestation for an antenatal check-up. Laboratory studies reveal a platelet count of 90,000/mm3 with no other abnormalities in test results. She has no symptoms and physical examination is unremarkable. Which one of the following could be the most likely diagnosis?
A. Idiopathic thrombocytopenia.
B. Immune thrombocytopenia.
C. Gestational thrombocytopenia.
D. Disseminated intravascular coagulation (DIC).
E. Preeclampsia.
C. Gestational thrombocytopenia
Thrombocytopenia is the second most common hematological finding during pregnancy after anemia. Thrombocytopenia can have many etiologies, some of which are not specific to pregnancy.
Causes of thrombocytopenia in pregnancy include:
- Pregnancy-related
* Acute fatty liver
* Gestational thrombocytopenia
* HELLP syndrome
* Hypertensive disorders such as preeclampsia/eclampsia - General
* Autoimmune conditions (SLE, antiphospholipid syndrome)
* Bone marrow disorders
* Disseminated intravascular coagulation
* Drugs
* Heparin induced thrombocytopenia
* Hypersplenism
* Inherited, Type IIB Von Willebrand disease
* B12, or folate deficiency
* Immune (idiopathic) thrombocytopenia
* Pseudothrombocytopenia
* Secondary immune thrombocytopenia due to viral infections (e.g., HIV, Hep C, CMV, EBV)
* Thrombotic microangiopathies ( TTP/HUS)
Of these, gestational thrombocytopenia is the most common cause of thrombocytopenia in pregnancy, followed by ITP as the second most common cause.
Gestational thrombocytopenia (incidental thrombocytopenia) occurs in approximately eight percent of all pregnancies and accounts for more than 70% of cases with thrombocytopenia in pregnancy. Although the pathophysiology of gestational thrombocytopenia is unknown, it is thought to be related to hemodilution, increased platelet consumption, and increased platelet aggregation (as a result of increased levels of thromboxane A2). Platelet count may be lower in women with twins perhaps due to a greater increase in thrombin generation.
In summary, gestational thrombocytopenia is defined by the following five criteria:
1. Mild and asymptomatic thrombocytopenia
2. No past history of thrombocytopenia (except possibly during a previous pregnancy) 3. Occurrence during late gestation (from mid-second trimester to third trimester)
4. No association with fetal thrombocytopenia
5. Spontaneous resolution after delivery
This woman has an isolated and asymptomatic mild thrombocytopenia. This picture is consistent with either gestational thrombocytopenia or ITP. Given the high prevalence of gestational thrombocytopenia, it could be the most likely diagnosis.
OPTION A and B : Immune thrombocytopenia and idiopathic thrombocytopenia are in fact two names for one condition characterized by isolated thrombocytopenia of autoimmune origin that can present similar to gestational thrombocytopenia; however, gestational thrombocytopenia is much more common during pregnancy and more likely to be the underlying cause of this woman’s presentation.
OPTION D : DIC is a serious life-threatening condition presenting with low-platelet count, intravascular coagulation, and thrombus formation potentially in every organ system. The two most common conditions associated with DIC during pregnancy are placental abruption and fetal demise. Laboratory studies show low levels of all of coagulation factors. An asymptomatic isolated thrombocytopenia is not a picture consistent with DIC.
OPTION E : Although thrombocytopenia is a common feature of pregnancy-related hypertensive disorders such as pre-eclampsia or HELLP syndrome. With pre-eclampsia other manifestations clinical or laboratory findings would be expected (e.g., proteinuria, hypertension, elevated liver enzymes, headache, abdominal pain, etc.) An isolated mild thrombocytopenia in a completely asymptomatic pregnant woman is unlikely to have been caused by pre-eclampsia.
NOTE - Compared to non-pregnant women with thrombocytopenia, pregnant women with thrombocytopenia tend to have fewer bleeding complications due to the procoagulant state induced by increased levels of fibrinogen, factor VIII and von Willebrand factor, suppressed fibrinolysis and reduced protein S activity. There are several other pregnancy-related conditions that can also lead to thrombocytopenia. Thrombocytopenia in pregnancy is a common reason for hematology consultation.
- Medscape – Thrombocytopenia in Pregnancy * UpToDate – Thrombocytopenia in pregnancy
A 31-year-old woman presents with complaints of 10 kg weight loss and heavy periods for the past six months. However, her periods were and still are regular occurring at intervals of 32 days lasting for five days with no increased pain or discomfort. The number of pads she uses indicates blood loss of more than 80 mL per cycle. She has no known medical condition. The rest of the physical examination, including pelvic exam is inconclusive. An office urine pregnancy test is negative. Which one of the following would be the most appropriate investigation to consider first?
A. Transvaginal ultrasonography.
B. Abdominal ultrasonography.
C. Thyroid stimulating hormone (TSH) level.
D. Prolactin level.
E. Follicle stimulating hormone (FSH) level.
A. Transvaginal ultrasonography
This woman has presented with an altered pattern of menstrual bleeding in from of increased menstrual flow at regular intervals, namely ‘menorrhagia’. Menorrhagia is a form of abnormal uterine bleeding (AUB).
Normal menstrual periods last 3–6 days and is associated with blood loss of up to 80 ml. Menorrhagia is defined as menstrual periods lasting more than 7 days and/or involving blood loss greater than 80 mL.
AUB affects 9-14% of women. AUB generally can be divided into anovulatory and ovulatory patterns. Ovulatory pattern of AUB is characterized by abnormal volume or length of blood loss at regular intervals, whereas in anovulatory pattern the bleeding is irregular and unpredictable (no regular periods)
Ovulatory pattern is usually caused by uterine problems such as leiomyomas, endometriosis, adenomyosis, polyps, etc. as opposed to anovulatory pattern that is a result of hormonal problems such as polycystic ovarian syndrome (PCOS), hypothyroidism, hyperthyroidism, hyperprolactinemia, or Cushing syndrome.
The approach to women with ovulatory versus anovulatory pattern is different. It is recommended that if the pattern cannot be specified, the patient be assessed as having irregular bleeding because this pattern includes investigation for endometrial hyperplasia/cancer for more diagnostic safety.
In approach to every woman of reproductive age presenting with AUB, the most important to consider and exclude first is pregnancy regardless of the pattern. If anemia is suspected based on the history or clinical findings, a full blood exam (FBE) is indicated. A serum TSH to exclude thyroid disease is indicated if the history and/or physical examination raise suspicion against thyroid disease, especially hypothyroidism. Routine use of TSH in every woman presenting with AUB is not recommended.
Other investigations depend on the pattern and whether it is ovulatory or anovulatory. Since this woman has menorrhagia, indicating ovulatory AUB and the high possibility of structural anomalies such as fibroids, adenomyosis, polyps, etc., a transvaginal ultrasound is very likely to pick up the etiology and is the most important initial diagnostic approach to consider first and go with. Abdominal ultrasound is not as accurate as transvaginal ultrasound for this purpose yet can be used if transvaginal ultrasound is not possible (e.g., adolescent girls).
NOTE - A very significant finding in the history is the weight loss. For significant weight loss in this woman in the presence of AUB, the two most important differential diagnoses to think of are hyperthyroidism and cancer. Of these two, the greatest concern is cancer, especially endometrial cancer. Even if cancer is the case, transvaginal ultrasound is the initial option of choice for assessment of endometrial thickness and other abnormalities of the uterine cavity and adjacent structures. In the presence of any endometrial abnormality, the patient will require endometrial curettage and biopsy. Hyperthyroidism is more likely to be associated with amenorrhea or oligomenorrhea than menorrhagia; however, thyroid function tests can be considered as well once cancer as a main concern is safely excluded.
Hormonal assay like FSH (option E), LH, and prolactin (option D) are investigations to consider in patients with anovulatory AUB, where hormonal derangements are the most important underlying etiology to consider and think of.
- Cancer Australia - Abnormal Vaginal Bleeding in Pre- and Post-menopausal Women
- Clinical Care Standard - Heavy Menstrual Bleeding Clinical Care Standard
Angelina, accompanied by her mother, is in your office for consultation. She is 15 years old, and is concerned because she has not started her periods yet. She says: “I am about the same height as everybody else in my class at school. Their periods have started but mine not.” Which one of the following is the most appropriate initial question to ask her?
A. When she had her breast buds developed.
B. When she had her pubic hair.
C. Whether she gets cyclic abdominal pain.
D. Whether she had accelerated growth.
E. When she had her axillary hair.
A. When she had her breast buds developed
The complaint presented in the scenario is primary amenorrhea. Primary amenorrhea is defined as the absence of menses after the age of 16 in the presence of normal growth and secondary sexual characteristics, or 13-14 in the absence of secondary sexual characteristics. In case breast budding occurs before the age of 10, amenorrhea is defined as absent periods within the next 5 years.
Puberty in girls start with breast development followed by the growth spurt, growth of axillary and pubic hair, and finally menstruation. Breast above Tanner II stage indicates the commencement of puberty and exposure to estrogen.
In this case, it is very important to know whether puberty has started and if so, how long has she been through puberty. This can be known by simply asking if she has had any breast developments as the most appropriate question. Other question are relevant if puberty has already begun indicated by breast development.
- NSW Health – Amenorrhoea
- RACP – Investigating primary and secondary amenorrhoea
A 26-year-old woman with history of chronic immune thrombocytopenic purpura (ITP) presents to your clinic and seeks advice regarding pregnancy. She has a platelet count of 70000/mm3. Which one of the following is the correct statement in counselling her?
A. She should have splenectomy before pregnancy.
B. She should avoid pregnancy in the next 2 years.
C. She can become pregnant.
D. If she becomes pregnant, the mode of delivery should be cesarean section.
E. She cannot become pregnant while the platelet count is below 70000/mm3.
C. She can become pregnant
While uncommon, immune thrombocytopenia (ITP) is an important cause of thrombocytopenia in pregnant women either as a pre-existing condition or occurring at any time during pregnancy.
ITP developed in pregnancy is important to be distinguished from ‘incidental’ or gestational thrombocytopenia, which, is responsible for up to 80% of cases of thrombocytopenia in pregnant women, or from more serious conditions such as HELLP syndrome.
Gestational thrombocytopenia usually causes an approximately 10% decrease in platelet counts and is usually characterized by counts that are above 70,000/mm3. When platelets are lower than that, it is mostly ITP.
A diagnosis of ITP is usually made in 1-4% of cases of thrombocytopenia. ITP, either pre-exiting or developed during pregnancy, does not prevent a woman from becoming pregnant or safely delivering a healthy baby. The best advice for this woman is that she can become pregnant and ITP does not preclude vaginal delivery; however, the pregnancy is categorized as high risk and she will need extra care and probably treatment, especially around the delivery.
In pregnant women with ITP, treatment is usually not necessary as long as the platelet count is above 30,000, unless there is bleeding or easy bruising. For pregnant women with symptomatic platelet counts of above 30,000, or women with platelet count of below 30,000 regardless of symptoms, treatment either with prednisolone or intravenous immunoglobulin (IVIG) is required due to significant concern of bleeding, especially uterine bleeding.
It is important to maintain a platelet count that allows the pregnant woman with the condition to go through delivery. This safe count is at least 50,000. For regional anesethesia, a count between 70,000 and 100,000 is often demanded.
- Medpage Today - Clinical Challenges: ITP in Pregnancy
- Immune Thrombocytopenia in Pregnancy
Adele is a 23 years old primigravida woman who has a spontaneous abortion at 12 weeks gestation 2 weeks ago. She has presented to you because she wants to have another pregnancy as soon as possible as she thinks this could help her husband and her get over with their loss. She asks you when she can become pregnant again. Which one of the following would be the best advice?
A. She should not become pregnant for at least 6 months and should be on OCPs for now.
B. She can start trying to conceive again immediately if she feels fit for that.
C. She can conceive after 12 months.
D. She can conceive after 3 months.
E. She can conceive after 2 menstrual cycles.
B. She can start trying to conceive again immediately if she feels fit for that
Despite the emotional pain of pregnancy loss, most couples go onto having another pregnancy. It is recommended that women with pregnancy loss wait for one menstrual period after a pregnancy loss and then try the next pregnancy when they are feeling emotionally strong and ready.
For some, this could be 2 months, for others 2 years. It is generally recommended that women with pregnancy loss can conceive immediately after they feel fit both physically and emotionally to have another pregnancy. For this woman, next pregnancy can be planned once she feels emotionally ready.
The following are recommendations for women who are planning conceiving again after a pregnancy loss:
- Reducing the chance of another pregnancy loss by starting the next pregnancy as healthy as possible. Some measures are quitting smoking and alcohol, achievement of a healthy weight through exercise and diet and adoption of healthy lifestyle in general.
- Checking on any long term health problems such as depression, high blood pressure, or diabetes. Medicines might need to be altered or more emphasis placed on non- medical treatments after appropriate advice from health professionals.
- Having a health check-up before the next pregnancy.
For couples with a lost pregnancy, peer support can be invaluable. Other mothers who have suffered pregnancy loss can have a real understanding of a woman’s experience as she goes through another pregnancy.
- RANZCOG – Pregnancy loss
A 27-year-old woman is in labor at 39 weeks gestation when passage of meconium is noted. A cardiotocography (CTG) is arranged that shows a fetal heart rate (FHR) of 149 bpm, a beat-to-beat variability of 15, no acceleration, and no deceleration. Which one of the following should be the next best step in management?
A. Fetal scalp blood sampling as there is a 10% chance of hypoxia.
B. Fetal scalp blood sampling as there is a 50% chance of hypoxia.
C. Fetal scalp blood sampling as there is 75% chance of hypoxia.
D. Emergency cesarean section.
E. Close monitoring until delivery as there is no abnormality.
E. Close monitoring until delivery as there is no abnormality
Baseline fetal heart rate (FHR) is the mean level of the FHR when this is stable, excluding accelerations and decelerations. It is determined over a period of 5-10 minutes, expressed as beats per minute (bpm). Preterm fetuses tend to have values towards the upper end of the normal range.
Baseline variability is the minor fluctuation in baseline FHR. It is assessed by estimating the difference between the highest peak and lowest trough of fluctuation in one-minute segments of the fetal hear rate trace. Baseline variability is categorized as follows:
* Normal variability: 6-25 beats per minute
* Reduced variability: 3-5 beats per minute
* Absent variability: <3 beats per minute
* Increased (salutatory) variability: > 25 beats per minute.
Accelerations are transient increases in FHR of 15 bpm or more above the baseline that last at least 15 seconds. Accelerations in preterm fetuses may be of lesser amplitude and shorter duration.
Decelerations are transient episodes of decreased FHR below the baseline of more than 15 bpm lasting at least 15 seconds. The specific features of the deceleration inform the classification. Decelerations should never be described as ‘unprovoked’; the fetus will not decelerate its heart rate without physiological provocation. Uterine activity, even in its mildest form, will result in decelerations in a fetus whose oxygenation is already compromised.
Decelerations are categorized as:
- Early decelerations - benign and associated with the sleep cycle and often in the range of 4-8 cm of cervical dilatation. They are caused by head compression and in general are a normal physiological response to a mild increase in intracranial pressure. Importantly they are uniform in shape and start and finish with the contraction. They may be said to mirror the contraction.
- Variable decelerations - repetitive or intermittent decreasing of FHR with rapid onset and recovery. Time relationships with contraction cycle may be variable but most commonly occur simultaneously with contractions. The significance of variable decelerations depends on the overall clinical picture and specific features of the decelerations themselves, as well as other features of the CTG. Variable decelerations in association with other non-reassuring or abnormal features change the category of the deceleration to ‘complicated’.
- Prolonged decelerations - a decrease of FHR below the baseline of more than 15 bpm for longer than 90 seconds but less than 5 minutes.
- Late decelerations - defined as uniform, repetitive decreasing of FHR with, usually, slow onset mid to end of the contraction and nadir more than 20 seconds after the peak of the contraction and ending after the contraction. Late decelerations are caused by contractions in the presence of hypoxia. This means that they will occur with each contraction and the fetus is already hypoxic. There will be no features of a well oxygenated fetus, like early or typical variable decelerations, normal baseline variability or shouldering. They start after the start of the contraction and the bottom of the deceleration is more than 20 seconds after the peak of the contraction. Importantly, they return to the baseline after the contraction has finished. In the hypoxic fetus, this will include decelerations of less than 15 bpm (and occasionally less than 5 bpm).
A CTG is considered abnormal and demands further action or investigations if :
There is at least two of the following:
* Baseline FHR is between 100-109 bpm or between 161-170 bpm
* Variability of FHR is reduced (3-5 bpm for >40 minutes)
* Decelerations are variable without complicating features
OR
There is any of the following:
* Baseline FHR is <100 bpm or >170bpm
* Variability is absent (<3 bpm)
* Decelerations are prolonged for >3 minutes OR late OR have complicated variables
This CTG has none of the above and is completely normal; therefore, close monitoring until the delivery is all that is needed at this stage.
- WA Health Department – Obstetrics and Gynaecology Guidelines
- RANZCOG Intrapartum Fetal Surveillance Clinical Guideline